core conditions Flashcards

1
Q

what does an acute subdural haemorrhage and an extradural haemorrhage look like on a CT scan (i.e. how can you tell the difference between the 2)?

A

SDH: imaging will show a hyperdense (bright), crescenteric collection surrounding the brain that is not limited by suture lines EDH: imaging will show a hyperdense (bright), biconvex (or lentiform) collection around the surface of the brain that is limited by suture lines

How well did you know this?
1
Not at all
2
3
4
5
Perfectly
2
Q

generally speaking, what are some causes of aneurysms?

A
  • Atherosclerosis. - Vasculitis (e.g. Kawasaki disease). - Syphilis. - Infective (may be due either to fungi or bacteria invading the vessel wall and may be due to distant spread from infective endocarditis or from localised spread from adjacent structures). - Penetrative or blunt trauma. - Congenital (e.g. berry aneurysm). - Cocaine use has been associated with cerebral, aortic, visceral and peripheral aneurysms
How well did you know this?
1
Not at all
2
3
4
5
Perfectly
3
Q

generally, how should delirium be managed?

A
  • Investigate underlying cause and treat this (e.g. UTI, polypharmacy etc) - Supportive management: > Involve family and carers > Have clock in room > Have familiar items in room > Have consistent staff see the patient - Haloperidol or olanzapine may be used if the patient is a risk to themselves or others (but weigh up pros/cons of adding additional medications)
How well did you know this?
1
Not at all
2
3
4
5
Perfectly
4
Q

what drug class is tramadol?

A

opioid

How well did you know this?
1
Not at all
2
3
4
5
Perfectly
5
Q

how would you expect the CSF from a lumbar puncture to appear (look like) in: - a normal sample - bacterial meningitis - viral meningitis?

A
  • normal: clear - bacterial meningitis: cloudy/turbid - viral meningitis: clear
How well did you know this?
1
Not at all
2
3
4
5
Perfectly
6
Q

very very briefly, how should infective endocarditis be managed?

A

Initiate empirical abx while awaiting blood culture results (use trust guidelines) Some may require surgery (there’s a number of possible indications).

How well did you know this?
1
Not at all
2
3
4
5
Perfectly
7
Q

if morning stiffness is present in OA, how long would you expect it to last?

A

<30 mins

How well did you know this?
1
Not at all
2
3
4
5
Perfectly
8
Q

what is the name of the fraction of gluten responsible for the immunological response seen in coeliac disease?

A

gliadin

How well did you know this?
1
Not at all
2
3
4
5
Perfectly
9
Q

what blood result is taken to indicate hepatotoxicity in the context of paracetamol overdose?

A

ALT >1000 IU/L

How well did you know this?
1
Not at all
2
3
4
5
Perfectly
10
Q

what are some of the possible causes of thrombophilia?

A
  • Inherited: Factor V Leiden, Protein C/Protein S deficiency - Acquired: Antiphospholipid syndrome, malignancy, pregnancy, myeloproliferative disorders (e.g. polycythaemia vera, essential thrombocytosis), malignancy, acute inflammatory states (e.g. IBD, connective tissue diseases), nephrotic syndrome, DIC
How well did you know this?
1
Not at all
2
3
4
5
Perfectly
11
Q

how should superficial thrombophlebitis be managed?

A

General: - The affected extremity should be elevated if possible and large, warm compresses may be applied - exercise reduces pain and the possibility of DVT. Pharmacological: - Topical analgesia with non-steroidal anti-inflammatory creams applied locally to the superficial vein thrombosis/superficial thrombophlebitis area controls symptoms - LMWH heparin or fondaparinux may be prescribed (depending on trust guidelines)

How well did you know this?
1
Not at all
2
3
4
5
Perfectly
12
Q

if it is decided that compression stockings are to be used in the management of someone with varicose veins (note that NICE doesn’t recommend them unless interventional options are not appropriate), what must you first do?

A

rule out the possability of peripheral arterial disease (in which compression stockings would be contraindicated)

How well did you know this?
1
Not at all
2
3
4
5
Perfectly
13
Q

describe the pathophysiology involved in hyperosmolar hyperglycaemic state

A
  • HHS is usually precipitated by a trigger e.g. infection, poor diabetes control etc - A relative insulin deficiency leads to a serum glucose that is usually higher than 33 mmol/L (600 mg/dL), and a resulting serum osmolarity that is greater than 320 mosmol/kg - This causes excessive urination, leading to volume depletion and hemoconcentration that causes a further increase in blood glucose level. - Ketosis is absent because the presence of some insulin inhibits hormone-sensitive lipase mediated fat tissue breakdown.
How well did you know this?
1
Not at all
2
3
4
5
Perfectly
14
Q

how is a ‘chronic migraine’ defined?

A

patients have headaches at least 15 days a month, with at least eight episodes where their headaches and associated symptoms meet diagnostic criteria for migraine

How well did you know this?
1
Not at all
2
3
4
5
Perfectly
15
Q

what is the most common feature of a focal seizure arising from the parietal lobe?

A

Contralateral paraesthesia

How well did you know this?
1
Not at all
2
3
4
5
Perfectly
16
Q

what is the antidote for opioid overdose?

A

naloxone (0.4-2mg IV/IM)

How well did you know this?
1
Not at all
2
3
4
5
Perfectly
17
Q

how should a patient be instructed to take a bisphosphonate and why?

A

They may cause oesophageal irritation and so should be taken by the patient on an empty stomach (so usually morning), sitting up with plenty of water. The patient should then stay sat upright and not eat or take other oral medication for at least 30 minutes afterward

How well did you know this?
1
Not at all
2
3
4
5
Perfectly
18
Q

how should septic arthritis be managed?

A
  • IV abx (e.g. flucloxacillin or vancomycin) started empirically - Analgesia - Joint immobilisation (and begin to mobilise ~5 days later/ after infection has been treated) - Joint drainage considered if antibiotics not effective - Urgent referral to orthopaedics
How well did you know this?
1
Not at all
2
3
4
5
Perfectly
19
Q

what is the biggest concern with being on a DMARD?

A

People on DMARDs are more prone to infections and complications of infections due to myelosuppression

How well did you know this?
1
Not at all
2
3
4
5
Perfectly
20
Q

what are some of the causes/ contributing factors to orthostatic hypotension?

A
  • Ageing: healthy ageing is associated with decreased autonomic buffering capacity - Medication: antihypertensive medication, alpha-blockers, diuretics, tricyclic antidepressants - Conditions causes autonomic dysfunction: diabetes (peripheral neuropathy), Parkinsons - Anaemia, dehydration, hypovolaemia (e.g. secondary to blood loss)
How well did you know this?
1
Not at all
2
3
4
5
Perfectly
21
Q

how should a DVT be managed?

A

Start low molecular weight heparin or fondaparinux in confirmed DVT and those with a strong clinical suspicion - the LMWH or fondaparinux should be continued for at least 5 days or until the international normalised ratio (INR) is 2.0 or above for at least 24 hours, whichever is longer Offer oral anticoagulant (warfarin or NOAC) within 24 hours of diagnosis and continue for 3 months

How well did you know this?
1
Not at all
2
3
4
5
Perfectly
22
Q

which clotting factors are deficient in haemophilia type A and B?

A

A: factor VIII B: factor IX

How well did you know this?
1
Not at all
2
3
4
5
Perfectly
23
Q

which medication is is most commonly associated with osteonecrosis of the jaw (although is nonetheless a rare complication)?

A

bisphosphonates

How well did you know this?
1
Not at all
2
3
4
5
Perfectly
24
Q

what is the most common ECG indication for a permanent pacemaker insertion?

A

complete (3rd degree) heart block

How well did you know this?
1
Not at all
2
3
4
5
Perfectly
25
Q

a patient in SVT is haemodynamically unstable, how should they be managed?

A

If haemodynamically unstable, the most effective and rapid means of terminating any tachycardia is direct current (DC) cardioversion

How well did you know this?
1
Not at all
2
3
4
5
Perfectly
26
Q

a patient is found to have a 4.3cm AAA on abdominal ultrasound, how should they be followed up?

A

should have an aortic ultrasound every year to monitor (3-4.4 cm = monitor yearly, 4.5-5.4 cm = monitor every 3 months, >5.5cm = surgery adviced)

How well did you know this?
1
Not at all
2
3
4
5
Perfectly
27
Q

once the defib arrives at a crash call, how should it be used (not including what drugs are given when, just the procedure with the defib)?

A
  • Continue to give chest compressions and ventilation while defib pads are applied (one applied below the right clavicle, the other in the midaxillary line (V6 position)) - Pause CPR for rhythm analysis (this should be for no-longer than 5 seconds) > Shockable rhythms are ventricular fibrillation and pulseless ventricular tachycardia > Non-shockable rhythms are pulseless electrical activity and asystole - If shockable rhythm, resume chest compressions while the defib is charging. Everyone else should move away - When the defib is charged, stop chest compressions and have everyone stand away. When safe, give shock - Immediately resume compressions and ventilation following shock and continue for another 2 minutes - After 2 mins, do another rhythm check and follow steps as above (if rhythm was unshockable, continue CPR and do another rhythm analysis in 2 mins time)
How well did you know this?
1
Not at all
2
3
4
5
Perfectly
28
Q

what are some of the features of critical limb ischaemia?

A

features include: rest pain (which is not acute, and can often be eased by the patient hanging their leg out of bed at night), ulceration and gangrene remember: it is not the same as acute limb ischaemia (as it is a chronic condition) and is technically a sub-type of PAD (it’s basically just severe stage of PAD)

How well did you know this?
1
Not at all
2
3
4
5
Perfectly
29
Q

in which 3 leads is it normal to get T wave inversion?

A

aVR, III and V1

How well did you know this?
1
Not at all
2
3
4
5
Perfectly
30
Q

what are some causes of cardiogenic shock?

A
  • Most common: MI (particularly anterior) - Other causes: arrythmias (e.g. AF, VT), chest trauma, acute valvular incompetence, myocardial contusion (usually due to hitting steering wheel in car accident), severe PE, suppression of myocardial contractility by drugs (e.g. beta-blockers)
How well did you know this?
1
Not at all
2
3
4
5
Perfectly
31
Q

who should be referred to a fast-track cancer list when found to have IDA, regardless of any other symptoms?

A

anyone over 60

How well did you know this?
1
Not at all
2
3
4
5
Perfectly
32
Q

what are some of the possible causes of a trigeminal nerve lesion?

A

Sensory lesion: trigeminal neuralgia, herpes zoster Motor lesion: acoustic neuroma

How well did you know this?
1
Not at all
2
3
4
5
Perfectly
33
Q

if an adult attends A&E after sustaining a head injury, what features would indicate that a CT head should be performed in less than an hour of arrival?

A

CT head should be conducted in <1 hour if any of the following are present: - GCS < 13 on initial assessment (or <14 for children) - GCS < 15 at 2 hours after injury on assessment in the emergency department - Suspected open or depressed skull fracture - Any sign of basal skull fracture - Post-traumatic seizure - Focal neurological deficit - More than one episode of vomiting since the head injury

How well did you know this?
1
Not at all
2
3
4
5
Perfectly
34
Q

what should you tell patients who are undergoing investigations for coeliac disease?

A

Any test for CD is accurate only if a gluten-containing diet is eaten during the diagnostic process.

How well did you know this?
1
Not at all
2
3
4
5
Perfectly
35
Q

what most commonly causes cholecystitis?

A

In most cases (90%), it is caused by complete cystic duct obstruction usually due to an impacted gallstone in the gallbladder neck or cystic duct, which leads to inflammation within the gallbladder wall.

How well did you know this?
1
Not at all
2
3
4
5
Perfectly
36
Q

what are the 2 non-shockable rhythms in a cardiac arrest?

A

pulseless electrical activity asystole

How well did you know this?
1
Not at all
2
3
4
5
Perfectly
37
Q

anti-CCP (anti-cyclic citrullinated peptide) is most commonly seen in which condition?

A

rheumatoid arthritis (positive in about 70% of patients)

How well did you know this?
1
Not at all
2
3
4
5
Perfectly
38
Q

how should DIC be managed?

A
  • Treat underlying condition - A platelet transfusion should be considered when the platelet count is <20 x 10^9/L or <50 x 10^9/L with active bleeding - Fresh frozen plasma (FFP) is the preferred agent for replacement of coagulation factors and coagulation inhibitors when significant bleeding is present or when fibrinogen levels are <2.94 micromol/L
How well did you know this?
1
Not at all
2
3
4
5
Perfectly
39
Q

how is a pathological q wave usually defined and what do their presence indicate?

A
  • usually defined as duration ≥ 0.04 s or ≥ 25% of R-wave amplitude, although exact criteria can vary - They are a sign of a previous MI. They are not an early sign of myocardial infarction, but generally take several hours to days to develop.
How well did you know this?
1
Not at all
2
3
4
5
Perfectly
40
Q

very briefly, what causes Parkinson’s disease (i.e. the pathophysiology)?

A

It is caused by degeneration of the dopaminergic pathways in the substantia nigra

How well did you know this?
1
Not at all
2
3
4
5
Perfectly
41
Q

what is Uhthoff’s phenomenon and in which condition is it most associated with?

A

the worsening of symptoms when the body is overheated (e.g. hot weather, exercise, saunas). it is most associated with MS

How well did you know this?
1
Not at all
2
3
4
5
Perfectly
42
Q

what degree of heart block is mobitz type 2 and what does it look like on an ECG?

A

second degree 2 or 3 P waves occur successively, without a QRS complex following each (e.g. 2:1 or 3:1 ratio of P waves to QRS complexes)

How well did you know this?
1
Not at all
2
3
4
5
Perfectly
43
Q

what investigations might you consider in someone with suspected reactive arthritis?

A
  • ESR and CRP are usually very high. - FBC: normocytic normochromic anaemia, mild leukocytosis and thrombocytosis during the acute phase. - HLA-B27 is positive in the majority of those affected. Rheumatoid factor and antinuclear antibodies are absent. - Joint aspiration may be required to rule out septic or crystalline arthritis - Culture of stools, throat and urogenital tract samples in order to identify the causative organism
How well did you know this?
1
Not at all
2
3
4
5
Perfectly
44
Q

how does NICE define the different stages of hypertension?

A

Stage 1 hypertension: BP in surgery/clinic is ≥140/90 mm Hg and ambulatory blood pressure monitoring (ABPM) or home blood pressure monitoring (HBPM) ranges from 135/85 mm Hg to 149/94 mm Hg. Stage 2 hypertension: BP in surgery/clinic is ≥160/100 mm Hg but less than 180/120 mm Hg and ABPM or HBPM is ≥150/95 mm Hg. Stage 3 or severe hypertension: systolic BP in surgery/clinic is 180 mm Hg or higher or diastolic BP is 120 mm Hg or higher.

How well did you know this?
1
Not at all
2
3
4
5
Perfectly
45
Q

where is B12 absorbed?

A

in the distal ileum (so certain conditions, such as Crohn’s, or an ileal resection can affect absorption)

How well did you know this?
1
Not at all
2
3
4
5
Perfectly
46
Q

how should a suspected PE be investigated?

A

if PE is suspected, use the two-level PE Wells’ score to estimate the clinical probability of PE: - If score 4 or more, PE is likely and immediate computed tomography pulmonary angiogram (CTPA) should be offered - If score is less than 4 but PE still suspected, conduct D-dimer test. If positive, offer immediate CTPA other investigations: O2, FBC, U&Es, LFTs, baseline clotting screen, ECG investigations to consider: ABG, CXR , echo

How well did you know this?
1
Not at all
2
3
4
5
Perfectly
47
Q

what is the most common cause of a pansystolic murmur, heard best at the apex and often described as ‘blowing’?

A

mitral regurg

How well did you know this?
1
Not at all
2
3
4
5
Perfectly
48
Q

which joint does gout usually affect?

A

50% of all attacks and 70% of first attacks affect the first MTP.

How well did you know this?
1
Not at all
2
3
4
5
Perfectly
49
Q

what medications are recommended to be used in the long-term after an MI?

A
  • Aspirin should be given to all patients, unless contra-indicated. The addition of clopidogrel has been shown to reduce morbidity and mortality - Beta-blocker (a rate-limiting calcium channel blocker such as verapamil may be considered if beta-blockers are contraindicated) - ACE inhibitor - Nitrates for angina - Statins may help prevent a recurring cardiac event
How well did you know this?
1
Not at all
2
3
4
5
Perfectly
50
Q

what does the h.pylori eradication regime consist of?

A

2 abx + 1 PPI NICE currently recommend: - A PPI (e.g. omeprazole) and - Clarithromycin and - Amoxicillin or metronidazole

How well did you know this?
1
Not at all
2
3
4
5
Perfectly
51
Q

what is the most common type of atrioventricular re-entry tachycardia (AVRT)?

A

Wolff-Parkinson-White syndrome is the most common type of AVRT. (Possible ECG features include a short PR interval and wide QRS complexes with a slurred upstroke - ‘delta wave’)

How well did you know this?
1
Not at all
2
3
4
5
Perfectly
52
Q

what are the first-line medications used in partial and generalised seizures?

A
  • partial: carbamazepine (followed by sodium valproate and lamotrigine) - generalised: sodium valproate (or lamotrigine in women of childbearing age)
How well did you know this?
1
Not at all
2
3
4
5
Perfectly
53
Q

what scoring system is used to determine the most appropriate initial investigation in suspected PE?

A

the two-level PE Wells’ score

How well did you know this?
1
Not at all
2
3
4
5
Perfectly
54
Q

what is the incidence of complications in diverticulitis and what kind of complications can occur?

A

One third of patients with diverticulitis will develop complications such as: - perforation, abscess, fistula, stricture/obstruction - diverticular bleeding is a common cause of lower GI bleeding and is painless for most. Will resolve spontaneously in 70-80% of people

How well did you know this?
1
Not at all
2
3
4
5
Perfectly
55
Q

what is the daily requirement of: 1. water 2. potassium, sodium, chloride 3. glucose for a healthy adult?

A
  1. 25-30ml/kg/day 2. 1mmol/kg/day 3. 50-100g/day
How well did you know this?
1
Not at all
2
3
4
5
Perfectly
56
Q

when should someone be offered an immediate CTPA is suspected PE?

A

if they have a Well’s score of 4 or more

How well did you know this?
1
Not at all
2
3
4
5
Perfectly
57
Q

toxicity of which medication puts you at increased risk of SVT?

A

digoxin toxicity

How well did you know this?
1
Not at all
2
3
4
5
Perfectly
58
Q

what nerve is most likely to be affected by an anterior shoulder dislocation and what are the most likely affects of this?

A
  • axillary nerve may result in: - impaired arm abduction - flatted deltoid (muscle atrophy) - loss of sensation in regimental badge area
How well did you know this?
1
Not at all
2
3
4
5
Perfectly
59
Q

what are some of the possible features of a PE?

A
  • Sudden onset of symptoms - Dyspnoea - Pleuritic and/or retrosternal chest pain - Cough/haemoptysis - Dizziness/syncope - Hypoxia - Tachycardia/tachypnoea - Signs of DVT
How well did you know this?
1
Not at all
2
3
4
5
Perfectly
60
Q

what is the function of the abducens nerve?

A

controls the lateral rectus muscle (which enables eye to look laterally)

How well did you know this?
1
Not at all
2
3
4
5
Perfectly
61
Q

what are some of the clinical features of polymyalgia rheumatica?

A
  • Age over 50 years and duration of symptoms more than two weeks - Bilateral shoulder or pelvic girdle aching, or both (bilateral upper arm tenderness usually accompanies shoulder pain) - Shoulder/hip girdle stiffness lasting >1 hour. (note: shoulder stiffness may make it difficult to raise arms to brush hair, dress, brush teeth etc) - Evidence of an acute-phase response (raised ESR/CRP) and/or rapid response to steroids - may be flu-like features at the onset
How well did you know this?
1
Not at all
2
3
4
5
Perfectly
62
Q

what are some possible causes of third nerve palsy?

A

diabetes (note, is usually pupil sparing so pupil is not dilated) GCA subarachnoid haemorrhage

How well did you know this?
1
Not at all
2
3
4
5
Perfectly
63
Q

what is the investigation of choice in suspected acute mesenteric ischaemia?

A

CT

How well did you know this?
1
Not at all
2
3
4
5
Perfectly
64
Q

when do alcohol withdrawal symptoms usually start to present and when do they usually peak?

A

usually start around 8 hours after last alcoholic drink. Symptoms usually peak roughly by day 2

How well did you know this?
1
Not at all
2
3
4
5
Perfectly
65
Q

what is the most common clinical signs seen in individuals with chronic myeloid leukaemia?

A

splenomegaly

How well did you know this?
1
Not at all
2
3
4
5
Perfectly
66
Q

what is the blood loss (both in ml and %) in: 1. Class 1 2. Class 2 3. Class 3 4. Class 4 haemorrhagic shock?

A
  1. <750ml, 10-15% blood loss 2. 750-1500ml, 15-30% blood loss 3. 1500ml-2000ml, 30-40% blood loss 4. >2000ml, >40% blood loss
How well did you know this?
1
Not at all
2
3
4
5
Perfectly
67
Q

what condition is defined as a deficiency of factor IX?

A

haemophilia type B

How well did you know this?
1
Not at all
2
3
4
5
Perfectly
68
Q

what are some of the negative prognostic features for rheumatoid arthritis?

A
  • rheumatoid factor positive - poor functional status at presentation - HLA DR4 - X-ray: early erosions (e.g. after < 2 years) - extra articular features e.g. nodules - insidious onset - anti-CCP antibodies
How well did you know this?
1
Not at all
2
3
4
5
Perfectly
69
Q

what are some causes of non-meglabolastic macrocytic anaemia?

A
  • Alcohol abuse is a common cause. There may also be folate deficiency due to a poor diet although beer is a good source of folate. - Liver disease. - Severe hypothyroidism. - Reticulocytosis. - Other blood disorders including aplastic anaemia, red-cell aplasia, myelodysplastic syndromes, myeloid leukaemia. - Drugs that affect DNA synthesis, such as azathioprine
How well did you know this?
1
Not at all
2
3
4
5
Perfectly
70
Q

acetylcysteine is used in the management of the overdose of which medication?

A

paracetamol

How well did you know this?
1
Not at all
2
3
4
5
Perfectly
71
Q

how would you expect hydrocephalus to present (in both an adult and an infnat)?

A

Adult patients with hydrocephalus present with symptoms due to raised intracranial pressure, which include: - Headache (typically worse in the morning, when lying down and during valsalva) - Nausea and vomiting - Papilloedema - Coma (in severe cases) As infants have skull sutures that are not yet fused, the pathological rise in ICP causes: - Rapid increase in head circumference - The skull sutures separate, the anterior fontanelle bulges and the scalp veins become distended - An advanced sign is fixed downward gaze or ‘setting-sun’ sign

How well did you know this?
1
Not at all
2
3
4
5
Perfectly
72
Q

what is Kernig’s sign and what condition is it seen in?

A

with patient supine and hip fully flexed, there is resistance and pain on passive extension of knee seen in meningitis

How well did you know this?
1
Not at all
2
3
4
5
Perfectly
73
Q

generally speaking, what is the non-invasive ventilation of choice in: 1. type 1 respiratory failure patients 2. type 2 repspiratory failure patients

A
  1. CPAP 2. BiPAP
How well did you know this?
1
Not at all
2
3
4
5
Perfectly
74
Q

what causes the lub dub sound on auscultation?

A

lub= closing of mitral and tricuspid valves (signifying start of systole) dub= closing of aortic and pulmonary valves (signficying start of diastole)

How well did you know this?
1
Not at all
2
3
4
5
Perfectly
75
Q

what is the difference in the epidemiology between Hodgkins and Non-Hodgkins Lymphoma?

A

HL: there is a peak in incidence in young adults aged 20-34 years, with a further peak observed over 70 years NHL: Low-grade NHL is most commonly seen in >50s, but high-grade NHLs are one of the more common cancers seen in children and young adults

How well did you know this?
1
Not at all
2
3
4
5
Perfectly
76
Q

describe how you should perform chest compression on an adult patient?

A
  • Give 30 chest compressions followed by 2 ventilations. - The correct hand position for chest compression is the middle of the lower half of the sternum - Depth of 5–6 cm - Rate of 100–120 compressions min-1 - Allow the chest to recoil completely after each compression
How well did you know this?
1
Not at all
2
3
4
5
Perfectly
77
Q

what condition is a third heart sound (S3) associated with?

A

heart failure (note that in young healthy people it can be a normal phenomena)

How well did you know this?
1
Not at all
2
3
4
5
Perfectly
78
Q

how is pancreatitis traditionally diagnosed?

A

Serum amylase 3x or more than the normal is traditional way of diagnosing (although lipase levels are more sensitive and specific)

How well did you know this?
1
Not at all
2
3
4
5
Perfectly
79
Q

what signs/symptoms would you expect a patient with a subdural haematoma to present with?

A

Symptoms: - Headache (progressively getting worse) - Nausea/vomiting (may be sign of increased ICP) - May be a sign of increased intracranial pressure or increasing midline shift Signs: - Evidence of head trauma - Decreased/loss of consciousness (a sign of midline shift and herniation) - Abnormal eye examination: Anisocoria (unequal pupil size) may be a sign of brainstem herniation. Papilloedema may indicate raised ICP - May develop seizures

How well did you know this?
1
Not at all
2
3
4
5
Perfectly
80
Q

what are some of the possible complications of GCA?

A
  • Inflammation may affect blood supply to eye resulting in permanent visual damage (occurs in 20%) - Aneurysms, dissections and stenotic lesions of the aorta and its major branches. - Complications of long-term steroid use e.g. osteoporosis
How well did you know this?
1
Not at all
2
3
4
5
Perfectly
81
Q

what medications should be prescribed/considered in someone with newly diagnosed stable angina?

A
  • Prescribe sublingual glyceryl trinitrate (GTN) for the rapid relief of symptoms of angina. Advice to people: - Offer either a beta blocker or a calcium channel blocker as first-line treatment for stable angina. Decide which drug to use based on comorbidities, contraindications and the person’s preference - Consider aspirin 75 mg daily for people with stable angina, taking into account the risk of bleeding and comorbidities - Offer statin
How well did you know this?
1
Not at all
2
3
4
5
Perfectly
82
Q

describe how an IV insulin infusion should be given in DKA management

A

soluble insulin should be diluted (and mixed thoroughly) with sodium chloride 0.9% intravenous infusion to a concentration of 1 unit/mL; infuse at a fixed rate of 0.1 units/kg/hour.

How well did you know this?
1
Not at all
2
3
4
5
Perfectly
83
Q

what does the T-score have to be on DEXA scan to diagnose osteopenia?

A

T score less than -1 but above -2.5= hip BMD between 1 and 2.5 SD below the young adult reference mean (note <2.5= osteoporosis)

How well did you know this?
1
Not at all
2
3
4
5
Perfectly
84
Q

If a palate rises asymmetrically when patient is asked to say ‘aaahh’, what cranial nerve is affected?

A

vagus nerve

How well did you know this?
1
Not at all
2
3
4
5
Perfectly
85
Q

what is the best imaging for gallstones?

A

ultrasound

How well did you know this?
1
Not at all
2
3
4
5
Perfectly
86
Q

briefly, how should a subdural haematoma be managed?

A
  • Stabilise patient + urgent referral to neurology - Prophylactic antiepileptics - Stop/reverse (if possible) any anticoagulation - Manage raised ICP e.g. IV Mannitol - Surgery (such as burr hole craniotomy) is needed if there are focal signs, deterioration, a large haematoma, raised intracranial pressure or midline shift
How well did you know this?
1
Not at all
2
3
4
5
Perfectly
87
Q

what would you expect to see in a patient with a radial nerve injury?

A

-wrist drop - reduced sensation to the dorsal aspect of the hand (over the thumb and the proximal phalanx of the index, middle and the medial part of the ring finger)

How well did you know this?
1
Not at all
2
3
4
5
Perfectly
88
Q

what is Reiter’s triad and in with what condition is it associated with?

A

urethritis, conjunctivitis, and arthritis associated with reactive arthritis (but not all patients with reactive arthritis will have all 3 features of reiter’s triad) (also note: Reiter was a freakin NAZI so Reiter’s triad and Reiter’s syndrome (now reactive arthritis) or phrases which aren’t really used anymore)

How well did you know this?
1
Not at all
2
3
4
5
Perfectly
89
Q

what condition is the ‘waiter’s tip posture’ seen in?

A

Erb’s palsy

How well did you know this?
1
Not at all
2
3
4
5
Perfectly
90
Q

what are possible symptoms of AF?

A

chest pain/discomfort, palpitations, dyspnoea, dizziness, irregularly irregular pulse

How well did you know this?
1
Not at all
2
3
4
5
Perfectly
91
Q

what is the normal water requirement for a healthy adult?

A

25-30ml/kg/day

How well did you know this?
1
Not at all
2
3
4
5
Perfectly
92
Q

what are some of the possible complications of taking a bisphosphonate?

A
  • GI upset (e.g. heartburn, oesophagitis) - Hypophosphataemia - Osteonecrosis of the jaw. - Atypical femoral fracture
How well did you know this?
1
Not at all
2
3
4
5
Perfectly
93
Q

what is the definition of a subarachnoid haemorrhage?

A

An abnormal collection of blood between the arachnoid and pia mater, which normally contains CSF

How well did you know this?
1
Not at all
2
3
4
5
Perfectly
94
Q

what are the Canadian C-Spine rules? (i.e. what is their use, not what specifically are the rules)

A

The Canadian C-Spine Rule is a decision making tool used to determine when radiography should be utilized in patients following trauma where cervical spine injury is a concern

How well did you know this?
1
Not at all
2
3
4
5
Perfectly
95
Q

what should be done in primary care to help manage a patient with peripheral arterial disease?

A
  • lifestyle modifications e.g. smoking cessation, increased exercise - Control risk factors e.g. hypertension, hyperlipidaemia - Start on an antiplatelet (usually aspirin, can be clopidogrel)
How well did you know this?
1
Not at all
2
3
4
5
Perfectly
96
Q

briefly describe the pathophysiology involved in DKA

A
  • A lack of insulin in the blood (which may be due to diabetes or in response to a starvation state) results in glucose not being taken up into the cells - This stimulates the hormone glucagon, which stimulates glycolysis (the breakdown of glycogen), as well as glucaneo-genesis - This lack of cellular glucose also stimulates lipolysis (breakdown of fat). This causes the release of free fatty acids. These fatty acids are broken down in the liver to produce ketone bodies
How well did you know this?
1
Not at all
2
3
4
5
Perfectly
97
Q

how can you look for dilated bowel loops on an abdo x-ray? (i.e. what ‘rule’ can you use to help look for dilation)

A

use the 3-6-9 rule: small bowel should be <3cm, large bowel <6cm, ceacum <9cm

How well did you know this?
1
Not at all
2
3
4
5
Perfectly
98
Q

what is the max dose of paracetamol that should be taken in 24hours, and what is generally considered to be a lethal dose?

A

max dose= 4g lethal dose= >12g

How well did you know this?
1
Not at all
2
3
4
5
Perfectly
99
Q

what would you expect to see on the ECG of someone with unstable angina?

A

In unstable angina, the ECG typically shows T-wave inversion or ST-segment depression’ however, the ECG may be normal if some time has elapsed since the last episode of pain

How well did you know this?
1
Not at all
2
3
4
5
Perfectly
100
Q

what is the epidemiology for hodgkins lymphoma?

A

there is a peak in incidence in young adults aged 20-34 years, with a further peak observed over 70 years

How well did you know this?
1
Not at all
2
3
4
5
Perfectly
101
Q

how should an acute ischaemic limb be managed?

A
  • Urgent admission + inform vascular surgery - Heparinisation is needed immediately (this may double the limb salvage rate), and provide analgesia - If the occlusion is embolic, the options are surgical embolectomy or local thrombolysis e.g. tissue plasminogen activator - If the occlusion is due to thrombotic disease the options are intra-arterial thrombolysis, angioplasty or bypass surgery. If due to thrombosis of an arterial graft, then thrombolysis is the first step - If a limb is irreversibly ischaemic, amputation will be required
How well did you know this?
1
Not at all
2
3
4
5
Perfectly
102
Q

what are some of the clinical features septic arthritis?

A
  • Joint is usually swollen, warm, tender and exquisitely painful on movement - Fever and rigors often present - Bacteraemia is common and may cause prostration, vomiting or hypotension - Effusion may be obvious
How well did you know this?
1
Not at all
2
3
4
5
Perfectly
103
Q

what are some of the most common features of a focal seizure arising from the frontal lobe?

A
  • Head/leg movements - Dystonic posturing (‘fencing posture’) - Post-ictal weakness - usually last <60 seconds
How well did you know this?
1
Not at all
2
3
4
5
Perfectly
104
Q

briefly, how should chronic heart failure be managed?

A
  • Lifestyle changes - First-line treatment for all patients is an ACE inhibitor and a beta-blocker (ideally start separately) - Diuretics should be given for fluid overload - Vaccines: > Annual flu vaccine > One-off pneumococcal vaccine
How well did you know this?
1
Not at all
2
3
4
5
Perfectly
105
Q

what are some of the possible causes of spinal cord compression?

A
  • Trauma (including car accidents, falls and sports injuries) - Tumours, both benign or malignant - A prolapsed intervertebral disc - An epidural or subdural haematoma - Inflammatory disease, especially rheumatoid arthritis - infection - Cervical spondylitic myelopathy
How well did you know this?
1
Not at all
2
3
4
5
Perfectly
106
Q

what investigations should you consider in someone with suspected subarachnoid haemorrhage?

A
  • CT detects >90% of SAH within the first 48 hours - CT angiography to determine source of bleed - Lumbar puncture if CT -ve but clinical suspicion (xanthochromia= bilirubin in CSF, diagnostic of SAH). Should be done at least 12 hours after onset of symptoms
How well did you know this?
1
Not at all
2
3
4
5
Perfectly
107
Q

what is the classical triad of signs/symptoms seen in someone with cardiac tamponade?

A

hypotension, muffled heart sounds, jugular venous distention

How well did you know this?
1
Not at all
2
3
4
5
Perfectly
108
Q

with what signs and symptoms does Alzheimer’s dementia present with (early, later and advanced stages)?

A
  • Onset is insidious and usually progresses slowly over 7-10 years - In early stages: short-term memory loss, dysphasia (difficulty finding words), spatial disorientation - Later stages: confusion, personality disintegration (e.g. aggression, psychosis), apraxia (inability to perform particular purposive actions), agnosia (inability to recognise objects/people/sounds etc) - Advanced stages: apathy, wandering/immobility, incontinence, psychiatric symptoms (e.g. hallucinations, delusions)
How well did you know this?
1
Not at all
2
3
4
5
Perfectly
109
Q

how should a haemodynamically stable patient in VT be managed?

A
  • IV amiodarone - Direct current cardioversion 2nd line
How well did you know this?
1
Not at all
2
3
4
5
Perfectly
110
Q

what antibodies should be checked when suspecting coeliac disease?

A

Total immunoglobulin A (IgA) and IgA tissue transglutaminase (tTG) as the first choice. Endomysial antibodies (EMA) if IgA-tTG is unavailable

How well did you know this?
1
Not at all
2
3
4
5
Perfectly
111
Q

simply put, what usually causes hyperuricaemia (which can lead to gout)?

A

Hyperuricaemia is due to renal under-excretion of urate in 90% of cases and to over-production in 10%, although there is often an overlap of both

How well did you know this?
1
Not at all
2
3
4
5
Perfectly
112
Q

what usually happens to blood pressure in TACO and TRALI?

A

TACO: often rapidly increases TRALI: often decreases (with tachycardia)

How well did you know this?
1
Not at all
2
3
4
5
Perfectly
113
Q

how common is it to get secondary generalisation in a partial seizure?

A

occurs in roughly 2/3rds of patients who have a partial seizure

How well did you know this?
1
Not at all
2
3
4
5
Perfectly
114
Q

how can a migraine be managed acutely?

A

Moderate symptoms: NSAIDs or aspirin +/- anti-emetic (e.g. metoclopramide or prochlorperazine) Severe symptoms: triptan (e.g. sumatriptan) +/- NSAID +/- anti-emetic

How well did you know this?
1
Not at all
2
3
4
5
Perfectly
115
Q

how should TRALI be managed?

A

seek senior advice but largely supportive management with fluids and oxygen (may require ventilation + ICU admission if severe)

How well did you know this?
1
Not at all
2
3
4
5
Perfectly
116
Q

what bloods should you consider ordering in a patient with confusion?

A
  • FBC: look for markers of infection - B12 and folate: deficiency can cause confusion - TFTs: hypothyroidism can cause confusion - U&Es: renal failure may cause cognitive problems - LFTs: may indicate underlying pathology or alcohol problems - Calcium: hypo and hypercalcaemia can cause cognitive changes
How well did you know this?
1
Not at all
2
3
4
5
Perfectly
117
Q

what is the most common sign of trochlear nerve palsy/lesion?

A

diplopia (+ often head tilt to attempt to compensate)

How well did you know this?
1
Not at all
2
3
4
5
Perfectly
118
Q

what is the definition of: - cholelithiasis - cholecystitis - choledocholithiasis - cholangitis?

A
  • Cholelithiasis= stones in the gallbladder (so get colicky pain) - Cholecystitis= inflammation of gallbladder (caused by cystic duct obstruction by stones) - Choledocholithiasis= common bile duct obstruction by stone (so get jaundice) - Cholangitis= infection of biliary tree (caused by choledocholithiasis)
How well did you know this?
1
Not at all
2
3
4
5
Perfectly
119
Q

what does the mnemonic CRABBI stand for in the context of the main features of myeloma?

A
  • Calcium: Hypercalcaemia occurs as a result of increased osteoclast activity within the bones. This leads to constipation, nausea, anorexia and confusion - Renal: Monoclonal production of immunoglobulins results in light chain deposition within the renal tubules. This causes renal damage which presents as dehydration and increasing thirst - Anaemia: Bone marrow crowding suppresses erythropoiesis leading to anaemia. This causes fatigue and pallor - Bleeding: bone marrow crowding also results in thrombocytopenia which puts patients at increased risk of bleeding and bruising - Bones: Bone marrow infiltration by plasma cells and cytokine-mediated osteoclast overactivity creates lytic bone lesions. This may present as pain (especially in the back) and increases the risk of fragility fractures - Infection: a reduction in the production of normal immunoglobulins results in increased susceptibility to infection
How well did you know this?
1
Not at all
2
3
4
5
Perfectly
120
Q

what are the most important antibodies to test for in suspected rheumatoid arthritis?

A

Rheumatoid factor (RF) and anti-cyclic citrullinated peptide (anti-CCP) are the most important antibodies to test for. Anti-CCP is slightly more specific than RF

How well did you know this?
1
Not at all
2
3
4
5
Perfectly
121
Q

describe a typical aura which precedes a migraine

A

typical aura are visual, progressive, last 5-60 minutes and are characterised by transient hemianopic disturbance or a spreading scintillating scotoma (spot of flickering light near or in the centre of the visual field)

How well did you know this?
1
Not at all
2
3
4
5
Perfectly
122
Q

what are some of the features of an atonic seizure?

A

sudden, brief loss of muscle tone in the body causing person to become limp and collapse. There is usually no LOC and recovery is swift

How well did you know this?
1
Not at all
2
3
4
5
Perfectly
123
Q

what is a delta wave and what does it suggest if seen on an ECG?

A

The Delta wave is a slurred upstroke in the QRS complex often associated with a short PR interval This may be a sign that the ventricles are being activated earlier than normal from a point distant to the AV node. Associated with Wolff-Parkinson-White syndrome

How well did you know this?
1
Not at all
2
3
4
5
Perfectly
124
Q

a patient presents with chest pain and new onset LBBB, how should they be managed?

A

MI can present with new LBBB (without ST changes) so manage as MI

How well did you know this?
1
Not at all
2
3
4
5
Perfectly
125
Q

with what signs and symptoms would you expect small and large bowel obstructions to present with?

A

Small bowel: nausea, vomiting (associated more with proximal obstructions), faeculent vomiting (due to reverse peristalsis or fistula formation), constipation, crampy abdominal pain, fever, history of prior abdominal surgeries Large bowel: abdominal distension, severe or complete constipation, colicky abdominal pain, nausea and vomiting (not as severe or common as small bowel), tenesmus, palpable abdo mass Both: dehydration, active high-pitched ‘tinkling’ bowel sounds

How well did you know this?
1
Not at all
2
3
4
5
Perfectly
126
Q

how can hydrocepahlus be categorised and what are some of the causes of the 2 categories?

A

Obstructive (‘non-communicating’) hydrocephalus: this is due to a structural pathology blocking the flow of cerebrospinal fluid. Causes include: - Tumours - Acute haemorrhage (SAH or intraventricular haemorrhage) - Congenital malformations e.g. aqueduct stenosis, Chiari malformation Non-obstructive (‘communicating’) hydrocephalus: this is due to an imbalance of CSF production/absorption. Causes include: - Choroid plexus tumour (causes increased production of CSF- very rare however) - Failure or reabsorption at the arachnoid granulations, which may be due to meningitis or post-haemorrhage

How well did you know this?
1
Not at all
2
3
4
5
Perfectly
127
Q

besides from an early diastolic murmur, what other signs may be elicited on examination of someone with aortic regurgitation? (2)

A
  • Collapsing pulse - Wide pulse pressure (wide gap between systolic and diastolic BP)
How well did you know this?
1
Not at all
2
3
4
5
Perfectly
128
Q

how should an acute attack of gout be managed?

A
  • NSAIDs e.g. diclofenac, naproxen - If NSAIDs are not tolerated or are contraindicated, prescribe colchicine - If on allopurinol at the time of attack, do not stop taking, but do not start in acute attack (Note: will resolve spontaneously within 3-15 days but require treatment as extremely painful)
How well did you know this?
1
Not at all
2
3
4
5
Perfectly
129
Q

what are some of the possible risk factors for hyperuricaemia/gout?

A
  • male, middle-aged/elderly - Diet: meat, seafood, alcohol - Diuretics, chemotherapy - Obesity, diabetes, hypertension, CKD
How well did you know this?
1
Not at all
2
3
4
5
Perfectly
130
Q

how might someone who has taken an opioid overdose present?

A

Opioid overdose triad: - Pinpoint pupils - Unconsciousness - Respiratory depression Other symptoms include: hypotension, tachycardia, nausea/vomiting

How well did you know this?
1
Not at all
2
3
4
5
Perfectly
131
Q

what are some causes of iron overload?

A
  • Hereditary haemochromatosis: a genetic condition where individuals show increased iron absorption and organ deposition - Repeated red blood cell transfusions - Other causes: liver disease, excess iron intake
How well did you know this?
1
Not at all
2
3
4
5
Perfectly
132
Q

a pericardial friction rub is associated with what condition?

A

pericarditis

How well did you know this?
1
Not at all
2
3
4
5
Perfectly
133
Q

where abouts in the GI tract are diverticula most commonly seen?

A

Diverticula can occur throughout the gastrointestinal tract, but are seen most commonly in the sigmoid and descending colon

How well did you know this?
1
Not at all
2
3
4
5
Perfectly
134
Q

what is the prophylactic management of gout?

A
  • Lifestyle advice: limit alcohol, reduce meat/seafood, weight reduction, avoid dehydration - Manage risk factors: e.g. review diuretics, manage hypertension - Allopurinol, to be started at least 1-2 weeks after acute attack and titrate up while monitoring serum uric acid level
How well did you know this?
1
Not at all
2
3
4
5
Perfectly
135
Q

what colour do gram negative vs gram positive bacteria stain?

A

gram positive: purple gram negative: pink

How well did you know this?
1
Not at all
2
3
4
5
Perfectly
136
Q

which condition is the Framingham criteria used to help diagnose?

A

congestive heart failure

How well did you know this?
1
Not at all
2
3
4
5
Perfectly
137
Q

what are the 2 main diagnostic tests for myeloma?

A

Bone marrow aspiration and trephine biopsy: confirms the diagnosis if the number of plasma cells is significantly raised

How well did you know this?
1
Not at all
2
3
4
5
Perfectly
138
Q

a patient’s ECG shows tachycardia and a broad complex QRS, what arrythmia are they in?

A

ventricular tachycardia

How well did you know this?
1
Not at all
2
3
4
5
Perfectly
139
Q

lip smacking is a common feature in which kind of focal seizure?

A

those arising from the temporal lobe

How well did you know this?
1
Not at all
2
3
4
5
Perfectly
140
Q

during what time frame do acute transfusion reactions occur?

A

within 24 hours of the transfusion

How well did you know this?
1
Not at all
2
3
4
5
Perfectly
141
Q

how should status epilepticus be managed in a community setting?

A
  • Protect head + secure airway - Use buccal midazolam or rectal diazepam as first-line treatment for prolonged or repeated seizures in the community - Call for ambulance
How well did you know this?
1
Not at all
2
3
4
5
Perfectly
142
Q

what would you expect to see on the ECG of someone having an inferior STEMI and which artery is most likely to be involved?

A

ST elevation in leads II, III and aVF right coronary artery is the most likely occluded vessel

How well did you know this?
1
Not at all
2
3
4
5
Perfectly
143
Q

how would you expect someone with myasthenia gravis to present?

A

The key feature is muscle fatigability - muscles become progressively weaker during periods of activity and slowly improve after periods of rest: - extraocular muscle weakness: diplopia - proximal muscle weakness: face, neck, limb girdle - ptosis - dysphagia - may experience a myasthenic crisis: a complication of MG characterised by worsening muscle weakness resulting in respiratory failure that requires intubation and mechanical ventilation

How well did you know this?
1
Not at all
2
3
4
5
Perfectly
144
Q

what 2 enzyme defects most commonly cause haemolytic anaemia?

A

G6PD deficiency (most common), pyruvate kinase deficiency

How well did you know this?
1
Not at all
2
3
4
5
Perfectly
145
Q

what condition is the Philadelphia chromosome associated with?

A

chronic myeloid leukaemia (seen in 90% of cases)

How well did you know this?
1
Not at all
2
3
4
5
Perfectly
146
Q

what is the usual treatment regime in someone with newly diagnosed RA?

A

In people with newly diagnosed active RA, offer DMARD monotherapy (usually methotrexate but can be leflunomide or sulfasalazine), plus short-term glucocorticoids as first-line treatment as soon as possible, ideally within three months of the onset of persistent symptoms (note, used to be dual DMARD therapy but NICE guidelines recently changed)

How well did you know this?
1
Not at all
2
3
4
5
Perfectly
147
Q

what does the hypoglossal nerve control?

A

muscles of the tongue

How well did you know this?
1
Not at all
2
3
4
5
Perfectly
148
Q

what is the epidemiology for non-hodgkins lymphoma?

A

Low-grade NHL is most commonly seen in >50s, but high-grade NHLs are one of the more common cancers seen in children and young adults. White people have a greater risk than black or Asian.

How well did you know this?
1
Not at all
2
3
4
5
Perfectly
149
Q

which is more common, an anterior or posterior shoulder dislocation? and what is the most common mechanism of the injury?

A

anterior is far more common (~95%) most commonly caused by a FOOSH

How well did you know this?
1
Not at all
2
3
4
5
Perfectly
150
Q

how might a lesion of the trigeminal nerve present?

A
  • Loss of sensation - Loss of corneal reflex - Weakness of jaw clenching or side-to-side movements (depending on site of lesion, may not have all these features)
How well did you know this?
1
Not at all
2
3
4
5
Perfectly
151
Q

what basic investigations could be done in primary care to look for evidence of end-organ damage in someone with hypertension?

A
  • Urine dip for haematuria. - Arrange measurement of: > Urine albumin:creatinine ratio (to test for the presence of protein in the urine). > HbA1C (to test for diabetes). > Electrolytes, creatinine, and estimated glomerular filtration rate (to test for chronic kidney disease). - Examine the fundi (for the presence of hypertensive retinopathy). - Arrange for a 12-lead electrocardiograph to be performed (to assess cardiac function and detect left ventricular hypertrophy). - Consider the need for specialist investigations in people with signs and symptoms suggesting target organ damage or a secondary cause of hypertension.
How well did you know this?
1
Not at all
2
3
4
5
Perfectly
152
Q

what is the main diagnostic investigation for GCA?

A

temporal artery biopsy (characterised by a predominance of mononuclear cell infiltration or granulomatous inflammation, usually with multinucleated giant cells)

How well did you know this?
1
Not at all
2
3
4
5
Perfectly
153
Q

a patient describes experiencing an ‘aura’ before their seizure. Based on this alone, what is the most likely seizure that the patient has experienced?

A

a focal seizure from the temporal lobe (note: is often, but not always, from the temporal lobe)

How well did you know this?
1
Not at all
2
3
4
5
Perfectly
154
Q

what is Murphy’s sign and which condition is it used to help diagnose?

A

Murphy’s sign: lay two fingers over the RUQ. Ask the patient to breathe in. This causes pain and arrest of inspiration as the inflamed GB impinges on your fingers. The sign is only positive if a similar manoeuvre in the left upper quadrant does not cause pain Classically seen cholecystitis

How well did you know this?
1
Not at all
2
3
4
5
Perfectly
155
Q

what are some of the red flag features which may point towards malignancy in a patient with back pain?

A
  • history of cancer (particularly breast, prostate, lung (most common cancers causing bone mets) or myeloma) - Pain that remains when lying down, aching night-time pain that disturbs sleep - Constitutional symptoms, such as fever, chills, or unexplained weight loss. - Point tenderness over the vertebral body
How well did you know this?
1
Not at all
2
3
4
5
Perfectly
156
Q

what is the aetiology of peptic ulcers?

A
  • H.pylori (90% of duodenal ulcers, ~80% of gastric ulcers) - Drugs: NSAIDs, steroids, SSRIs - Lifestyle: alcohol, smoking, stress - Zollinger-Ellison syndrome: rare cause characterised by excessive levels of gastrin, usually from a gastrin secreting tumour
How well did you know this?
1
Not at all
2
3
4
5
Perfectly
157
Q

what would you expect to see on the CT head of a patient with a subdural haematoma (both acute and chronic)?

A

Acute SDH: imaging will show a hyperdense (bright), crescenteric collection surrounding the brain that is not limited by suture lines Chronic SDH: imaging will show a hypodense (dark), crescenteric collection around the surface of the brain that is not limited by suture lines

How well did you know this?
1
Not at all
2
3
4
5
Perfectly
158
Q

what investigations should be considered in suspected acute diverticulitis?

A
  • Bloods (raised WBC and inflammatory markers) - CXR with the patient upright can aid detection of pneumoperitoneum. - Abdominal X-rays may demonstrate small or large bowel dilation or ileus, pneumoperitoneum, bowel obstruction, or soft tissue densities suggesting abscesses. - CT scanning with intravenous, oral or rectal contrast: sensitivities and specificities for CT are significantly better than for contrast enemas. (note: endoscopy is generally avoided in the initial assessment due to risk of perforation)
How well did you know this?
1
Not at all
2
3
4
5
Perfectly
159
Q

when should levodopa be offered in patients with Parkinson’s disease?

A

Levodopa should be offered to people in the early stages of Parkinson’s disease whose motor symptoms impact on their quality of life

How well did you know this?
1
Not at all
2
3
4
5
Perfectly
160
Q

what are some euvolaemic causes of hyponatraemia?

A
  • Severe hypothyroidism - Normal physiological change of pregnancy - Syndrome of inappropriate antidiuretic hormone secretion (SIADH)
How well did you know this?
1
Not at all
2
3
4
5
Perfectly
161
Q

what criteria must be met for a stroke to be classified as a ‘partial anterior circulation stoke’ (PACS)?

A

Two of the following need to be present for a diagnosis of PACS: - Unilateral weakness (and/or sensory deficit) of the face, arm and leg - Homonymous hemianopia - Higher cerebral dysfunction (dysphasia, visuospatial disorder)

How well did you know this?
1
Not at all
2
3
4
5
Perfectly
162
Q

which autoantibody are vasculitides most commonly associated with?

A

ANCA (anti-neutrophilic cytoplasmic antibodies) note that ANCA is not specific to vasculitides however

How well did you know this?
1
Not at all
2
3
4
5
Perfectly
163
Q

with what features would you expect someone in acute left ventricular failure to present with?

A
  • dyspnoea: > Exertional (ask about exercise tolerance) + at rest > Orthopnoea (ask about no. of pillows) and paroxysmal nocturnal dyspnoea - Cough (may be worse at night and associated with pink/frothy sputum) - Fatigue - Signs: cyanosis, tachycardia, displaced apex beat, bibasal crackles (classically- may also be heard as a wheeze)
How well did you know this?
1
Not at all
2
3
4
5
Perfectly
164
Q

what is the most important investigation for septic arthritis?

A

Urgent joint aspiration for synovial fluid microscopy and culture

How well did you know this?
1
Not at all
2
3
4
5
Perfectly
165
Q

what does injury to the long thoracic nerve classically cause?

A

winged scapula

How well did you know this?
1
Not at all
2
3
4
5
Perfectly
166
Q

what are some of the risk factors for stoke?

A

Age, hypertension, smoking, diabetes, heart disease, post-TIA, combined oral contraceptive, carotid bruit (carotid artery occlusion), hyperlipidaemia, excess alcohol, clotting disorders

How well did you know this?
1
Not at all
2
3
4
5
Perfectly
167
Q

what condition will basically all individuals with Down’s syndrome eventually develop?

A

Alzheimer’s dementia (usually beginning around the age of 40)

How well did you know this?
1
Not at all
2
3
4
5
Perfectly
168
Q

what are some of the possible causes of vestibulocochlear nerve palsy/lesion?

A
  • Paget’s disease of the bone - Meniere’s disease - Herpes zoster - Acoustic neuroma - Aminoglycosides e.g. gentamicin
How well did you know this?
1
Not at all
2
3
4
5
Perfectly
169
Q

how long should a normal QRS complex be on an ECG?

A

roughly 0.12s (3 small squares)

How well did you know this?
1
Not at all
2
3
4
5
Perfectly
170
Q

into what 3 categories can causes of hyponatraemia be divided into and what are some causes which fall under each category?

A

Hypovolaemic: - Vomiting - Diarrhoea - Burns - Diuretic use - Prolonged exercise/sweating - Addison’s disease (insufficient production of cortisol and aldosterone) Euvolaemic: - Severe hypothyroidism - Normal physiological change of pregnancy - Syndrome of inappropriate antidiuretic hormone secretion (SIADH) Hypervolemic: - Cirrhosis - Nephrotic syndrome - Congestive heart failure

How well did you know this?
1
Not at all
2
3
4
5
Perfectly
171
Q

which is more common- non-hodgkins lymphoma or Hodgkins lymphoma?

A

NHL is 5 times more common than HL

How well did you know this?
1
Not at all
2
3
4
5
Perfectly
172
Q

what signs/symptoms would you expect to see in a patient with a AAA (both un-ruptured and ruptured)?

A

Unruptured AAA: - Most are asymptomatic - May be found incidentally on examination (expansile abdominal aorta- pulsatile is normal) or scan (ultrasound, CT, MRI, abdominal x-ray) - Pain in the back, abdomen, loin or groin Ruptured AAA: - Ruptured AAA presents with a classical triad of pain in the flank or back (may be sudden and severe), hypotension and a pulsatile abdominal mass; however, only about half have the full triad - Syncope, shock or collapse - Retroperitoneal haemorrhage may cause Grey Turner’s sign, i.e. flank bruising - Patient may feel cold, sweaty and faint

How well did you know this?
1
Not at all
2
3
4
5
Perfectly
173
Q

how is osteoporosis diagnosed using a DEXA scan? (i.e. how to interpret the score)

A
  • Osteoporosis= hip BMD 2.5 SD or more below the young adult reference mean (T score ≤-2.5). - Severe osteoporosis= hip BMD 2.5 SD or more below the young adult reference mean in the presence of one or more fragility fractures (T score ≤-2.5 PLUS fracture) where BMD= bone mineral density
How well did you know this?
1
Not at all
2
3
4
5
Perfectly
174
Q

briefly, how should viral meningitis be managed?

A

Viral meningitis usually requires only supportive therapy (e.g. fluids, antiemetics etc)

How well did you know this?
1
Not at all
2
3
4
5
Perfectly
175
Q

what chromosome is chronic myeloid leukaemia associated with?

A

Philadelphia chromosome (shortened chromosome 22)

How well did you know this?
1
Not at all
2
3
4
5
Perfectly
176
Q

how should hyperkalaemia be managed?

A

Protect cardiac membrane: - Give 10 ml 10% calcium gluconate (calcium chloride is an alternative ideally given via central access) which will improve ECG changes within 1-3 minutes; however, this effect only has a transient effect of 30-60 minutes - This reduces risk of arrythmias Shift potassium into cells: - Insulin-glucose infusion: usually 10 units of soluble insulin are added to 25 g of glucose and administered by IV infusion (potassium moves with glucose, glucose therefore given to prevent hypoglycaemia). - Capillary blood glucose needs to be checked before, during and after. - Potassium will decrease (0.6-1.0 mmol/L) in 15 minutes and the reduction lasts for 60 minutes - 10-20mg nebulised salbutamol is often also given to promote the movement of potassium into cells (note that there are other methods to remove potassium from the body, unlike the methods above which move potassium from extracellular to intracellular, but these are slower acting methods so should only be used in mild-moderate hyperkalaemia)

How well did you know this?
1
Not at all
2
3
4
5
Perfectly
177
Q

what are some of the most common causes of AF?

A

The most common causes of AF are coronary heart disease, hypertension, valvular heart disease and hyperthyroidism

How well did you know this?
1
Not at all
2
3
4
5
Perfectly
178
Q

what condition would you expect to see a positive Phalen’s test and Tinel’s sign and how are these elicited?

A

seen in carpal tunnel syndrome Positive Phalen test: flexing the wrist for 60 seconds causes pain or paraesthesia in the median nerve distribution. Positive Tinel’s sign: tapping lightly over the median nerve at the wrist causes a distal paraesthesia in the median nerve distribution

How well did you know this?
1
Not at all
2
3
4
5
Perfectly
179
Q

which tumours most commonly metastasize to the brain?

A
  • lung (most common) - breast - bowel - skin (namely melanoma) - kidney (note that brain metastases are the most common type of brain tumour)
How well did you know this?
1
Not at all
2
3
4
5
Perfectly
180
Q

what do the 4Hs and 4Ts refer to and what are they?

A

they refer to the reversible causes of cardiac arrest - 4 Hs: hypoxia, hypothermia, hypovolaemia, hyperkalaemia (+ hypokalaemia, hypoglycaemia, hypocalcaemia, acidaemia and other metabolic disorders) - 4 Ts: cardiac tamponade, tension pneumothorax, thrombosis (coronary or pulmonary), toxins

How well did you know this?
1
Not at all
2
3
4
5
Perfectly
181
Q

what should be prescribed in an acute attack of gout if NSAIDs are contraindicated?

A

colchicine

How well did you know this?
1
Not at all
2
3
4
5
Perfectly
182
Q

what are some causes of hyperkalaemia?

A

Renal causes: - AKI - CKD Excessive release from cells: - Acidosis e.g. DKA - Tissue damage (e.g. rhabdomyolysis, burns, trauma) Medication: - ACEi, ARBs, NSAIDs, potassium-sparing diuretics (e.g. spironolactone), beta-blockers, heparin, trimethoprim Iatrogenic: - Blood transfusion - Over-replacement of K+ with intravenous fluids Addison’s disease (lack of aldosterone causes more potassium to be retained in kidneys)

How well did you know this?
1
Not at all
2
3
4
5
Perfectly
183
Q

what is the cornerstone of treatment for ankylosing spondylitis?

A

NSAIDs

How well did you know this?
1
Not at all
2
3
4
5
Perfectly
184
Q

what are some of the genetic causes of haemolytic anaemia?

A
  • Red cell membrane abnormalities: hereditary spherocytosis, elliptocytosis. - Haemoglobin abnormalities: sickle cell anaemia, thalassaemia. - Enzyme defects: G6PD deficiency, pyruvate kinase deficiency
How well did you know this?
1
Not at all
2
3
4
5
Perfectly
185
Q

what is the most common cancer in children?

A

acute lymphoblastic leukaemia

How well did you know this?
1
Not at all
2
3
4
5
Perfectly
186
Q

overdose of aspiring causes what? (i.e. what type of poisoning)

A

salicylate poisoning

How well did you know this?
1
Not at all
2
3
4
5
Perfectly
187
Q

how long should LMWH or fondaparinux be continued for in management of PE?

A

Start as soon as possible and continue for at least 5 days or until INR is 2 or above for at least 24 hours (note: this is the same as DVT)

How well did you know this?
1
Not at all
2
3
4
5
Perfectly
188
Q

what are the 2 most common causes of acute pancreatitis and at roughly what age do they present?

A

alcohol (average age of onset is 38) and gallstones (average age of onset is 69) point is that alcohol-related AP usually occurs in younger patients than gallstone-related AP

How well did you know this?
1
Not at all
2
3
4
5
Perfectly
189
Q

how should a PE be managed (in a haemodynamically stable patient)?

A

Initial resuscitation: - Oxygen 100% - Obtain IV access - Analgesia if necessary (e.g. morphine) Anticoagulation therapy: - Offer low molecular weight heparin or fondaparinux in confirmed cases unless contraindicated. Start as soon as possible and continue for at least 5 days or until INR is 2 or above for at least 24 hours - Rivaroxaban (or other NOAC/warfarin) is recommended for at least 3 months following in patients with risk factors

How well did you know this?
1
Not at all
2
3
4
5
Perfectly
190
Q

how is appendicitis diagnosed and what investigations should be considered?

A

appendicitis is usually a clinical diagnosis investigations should be conducted to rule out alternative diagnoses e.g. - urinalysis +/- pregnancy test - imaging (CT is generally preferred over ultrasound)

How well did you know this?
1
Not at all
2
3
4
5
Perfectly
191
Q

groin pain that is exacerbated by movement should make you think of what?

A

hip OA

How well did you know this?
1
Not at all
2
3
4
5
Perfectly
192
Q

what are some causes of lymphoedema?

A

Possible causes include: lymph node removal (e.g. in breast cancer or other cancers), radiotherapy, cellulitis, trauma, DVT, obesity etc

How well did you know this?
1
Not at all
2
3
4
5
Perfectly
193
Q

what are some of the common causative organisms of bacterial meningitis in: - neonates - children - adults

A

Neonates: group B streptococci, E.coli Young children: Neisseria meningitidis, streptococcus pneumoniae, haemophilus influenzae Adults: Neisseria meningitidis, streptococcus penumoniae

How well did you know this?
1
Not at all
2
3
4
5
Perfectly
194
Q

how does severe haemophilia classically present? (in terms of presenting symptoms)

A

Recurrent spontaneous bleeding into joints (haemarthroses) and muscles without history of significant trauma

How well did you know this?
1
Not at all
2
3
4
5
Perfectly
195
Q

during what time frame does TRALI usually occur?

A

within 6 hours of the transfusion

How well did you know this?
1
Not at all
2
3
4
5
Perfectly
196
Q

what drug does NICE recommend if acetcycholinesterase inhibitors are contraindicated/not tolerated or if the patient has advanced Alzheimer’s?

A

Memantine (a N-methyl-D-aspartate (NMDA) antagonist)

How well did you know this?
1
Not at all
2
3
4
5
Perfectly
197
Q

what is the definition of an aneurysm?

A

An aneurysm is a permanent and irreversible dilatation of a blood vessel by ≥50% of its normal expected diameter

How well did you know this?
1
Not at all
2
3
4
5
Perfectly
198
Q

how should peripheral arterial disease be managed (both in primary and secondary care)?

A

In primary care: - lifestyle modifications e.g. smoking cessation, increased exercise - Control risk factors e.g. hypertension, hyperlipidaemia - Start on an antiplatelet (usually aspirin, can be clopidogrel) Referral: deteriorating or uncontrolled symptoms or the presence of features resembling critical limb ischaemia should prompt a referral to secondary care Secondary care intervention (usually only done in CLI or near CLI): - Endovascular revascularisation options include stents and percutaneous transluminal angioplasty (PTA). These are usually the options of choice when lesions are short and proximal - Bypass surgery is usually the option of choice when lesions are longer or multi-segmental. (ideally use vein grafts rather than prosthetic)

How well did you know this?
1
Not at all
2
3
4
5
Perfectly
199
Q

what are some signs of a basal skull fracture? (4)

A
  • haemotympanum (blood in tympanic cavity of middle ear) - ‘panda’ eyes (bruising around the eyes) - CSF leakage (ears or nose) - Battle’s sign (bruising which sometimes occurs behind the ear in cases of basal skull fracture).
How well did you know this?
1
Not at all
2
3
4
5
Perfectly
200
Q

why should you always examine the hip if someone presents with knee pain?

A

as knee pain may be referred pain from the hip (e.g. in OA)

How well did you know this?
1
Not at all
2
3
4
5
Perfectly
201
Q

how does gout usually present?

A
  • Tender, swollen, erythematous joint, with pain which reaches its crescendo over a 6- to 12-hour period - May have joint stiffness which may limit function - 50% of all attacks and 70% of first attacks affect the first MTP. Other sites include: wrist, ankle, knee, midtarsal joints
How well did you know this?
1
Not at all
2
3
4
5
Perfectly
202
Q

between DKA and HHS, which is more commonly seen in: 1. type 1 diabetics 2. type 2 diabetics

A
  1. DKA 2. HHS
How well did you know this?
1
Not at all
2
3
4
5
Perfectly
203
Q

what are some of the signs and symptoms of hypovolaeic shock?

A

Symptoms: - The individual may feel cold, unwell, anxious, faint and short of breath. - There may be faintness on standing or even on sitting up, due to postural hypotension. - There may be symptoms related to the cause of the hypovolaemia, such as pain from a bleeding ulcer, dissecting aneurysm, ruptured ectopic pregnancy, trauma or burns. - Gut ischaemia can lead to nausea and vomiting but the significance is often overlooked. Signs: - The patient may look pale and sweaty. - There may be tachypnoea. - The periphery may be cold from poor perfusion, and capillary refill time may be prolonged. However, this can be a poor indicator of hypovolaemia. - There may be tachycardia and a fall in blood pressure or postural hypotension. Tachycardia and cold peripheries from vasoconstriction may occur before a fall in BP, especially in children and young adults. - Young people may show little rise in pulse rate and no fall in BP despite significant exsanguination. It is very easy to underestimate the severity of loss in a young person. - Late features include confusion or even coma.

How well did you know this?
1
Not at all
2
3
4
5
Perfectly
204
Q

what condition is dermatitis herpetiformis associated with?

A

it is the skin manifestation of coeliac disease, with 75% of individuals who have the rash having villous atrophy

How well did you know this?
1
Not at all
2
3
4
5
Perfectly
205
Q

what does ‘P Pulmonale’ refer to and what causes it?

A

P wave amplitude is >2.5mm due to right atrial enlargement (which is usually secondary to pulmonary valve stenosis or pulmonary hypertension)

How well did you know this?
1
Not at all
2
3
4
5
Perfectly
206
Q

what are some features on a resting ECG which may be indicative of coronary artery disease?

A

Changes on a resting 12-lead ECG that are consistent with CAD include: - Pathological Q waves. - Left bundle branch block (LBBB). - ST-segment and T-wave abnormalities (eg, flattening or inversion).

How well did you know this?
1
Not at all
2
3
4
5
Perfectly
207
Q

what are the features of a tension-type-headache?

A
  • Typically described as a tightness/pressure in a band around the head. The headache is not pulsatile/throbbing - There may be an identifiable trigger e.g. stress - Not associated with photophobia, vomiting (may get mild nausea), not exacerbated by movement - Neurological examination is normal
How well did you know this?
1
Not at all
2
3
4
5
Perfectly
208
Q

what would you expect to see on the MRI of someone with MS? (hard q)

A

95% of patients have periventricular lesions and over 90% show discrete white matter abnormalities. Areas of focal demyelination can also be seen as plaques in the optic nerve, brainstem and spinal cord

How well did you know this?
1
Not at all
2
3
4
5
Perfectly
209
Q

what are some of the risk factors for DVT?

A

Age, past DVT, family history, cancer, immobilisation (long-distance travel, hospital stay, bed bound), smoking, obesity, male, thrombophilia, pregnancy, combined oral contraceptive, HRT

How well did you know this?
1
Not at all
2
3
4
5
Perfectly
210
Q

A patient presents with weakness on one side of their face. They are unable to use all muscles of facial expression on this side, including not being able to raise their eyebrows. Is this an UMN or LMN lesion of the facial nerve? And what is the most likely cause?

A

LMN lesion Bell’s palsy is the most common cause of LMN facial palsy (UMN lesions, seen in patients with strokes or tumours, are forehead sparing)

How well did you know this?
1
Not at all
2
3
4
5
Perfectly
211
Q

what constitutes Charcot’s triad and in what condition is it seen?

A

Charcot’s triad: fever, jaundice, RUQ pain Seen in cholangitis

How well did you know this?
1
Not at all
2
3
4
5
Perfectly
212
Q

how does pancytopenia present?

A

present with features of anaemia, thrombocytopenia and leukopenia: - Anaemia: fatigue, weakness, pallor, headaches etc - Thrombocytopenia: epistaxis, bleeding gums, petechiae, spontaneous bruising - leukopenia: fever, chills, frequent infections

How well did you know this?
1
Not at all
2
3
4
5
Perfectly
213
Q

what is pernicious anaemia and what kind of anaemia does it cause?

A

autoimmune condition in which the immune system attacks cells in stomach which produce intrinsic factor, meaning B12 cannot combine with intrinsic factor to be absorbed in the distal ileum It causes megaloblastic macrocytic anaemia

How well did you know this?
1
Not at all
2
3
4
5
Perfectly
214
Q

what are some of the possible symptoms you would expect to see in myeloma?

A
  • Bone pain (particularly backache) - pathological fractures - spinal cord/nerve root compression - lethargy (due to anaemia) - anorexia, dehydration (due to proximal tubule dysfunction from light-chain precipitation) - recurrent bacterial infection - bleeding and/or bruising - features suggesting amyloidosis (e.g. cardiac failure, nephrotic syndrome) - signs and symptoms of hypercalcaemia (e.g. thirst, constipation, nausea, confusion).
How well did you know this?
1
Not at all
2
3
4
5
Perfectly
215
Q

what are some causes of RBBB?

A
  • normal variant - more common with increasing age - right ventricular hypertrophy - chronically increased right ventricular pressure - e.g. cor pulmonale - pulmonary embolism - myocardial infarction - atrial septal defect (ostium secundum) - cardiomyopathy or myocarditis
How well did you know this?
1
Not at all
2
3
4
5
Perfectly
216
Q

what time-span does one small square and one large square on an ECG represent?

A

small square= 0.04s large square= 0.2 seconds

How well did you know this?
1
Not at all
2
3
4
5
Perfectly
217
Q

what are the 4 main mechanisms which cause iron deficiency anaemia?

A

1- blood loss 2- insufficient intake 3- insufficient absorption 4- excessive requirement

How well did you know this?
1
Not at all
2
3
4
5
Perfectly
218
Q

which tumour marker is most specific for pancreatic cancer?

A

CA 19-9

How well did you know this?
1
Not at all
2
3
4
5
Perfectly
219
Q

what is the epidemiology for acute myeloid leukaemia?

A

AML is the most common acute leukaemia in adults. Median age of onset is 67

How well did you know this?
1
Not at all
2
3
4
5
Perfectly
220
Q

what are some of the driving rules regarding seizures/epilepsy?

A

generally patients cannot drive for 6 months following a seizure. For patients with established epilepsy they must be fit free for 12 months before being able to drive

How well did you know this?
1
Not at all
2
3
4
5
Perfectly
221
Q

what is the definition of status epilepticus?

A

a convulsive seizure which continues for a prolonged period (longer than five minutes, note that it used to be 30 minutes), or when convulsive seizures occur one after the other with no return of consciousness between.

How well did you know this?
1
Not at all
2
3
4
5
Perfectly
222
Q

what are some risk factors associated with subarachnoid haemorrhage?

A

Hypertension, smoking, excess alcohol, cocaine, bleeding disorders, family history Conditions associated with berry aneurysms: polycystic kidneys, coarctation of the aorta, Ehlers-Danlos syndrome, neurofibromatosis type 1

How well did you know this?
1
Not at all
2
3
4
5
Perfectly
223
Q

what kind of anaemia would you expect to see in thalassaemia?

A

hypochromic microcytic anaemia

How well did you know this?
1
Not at all
2
3
4
5
Perfectly
224
Q

what drugs can be used in rate control of AF?

A

Either offer a beta-blocker or a rate-limiting calcium channel blocker (e.g. diltiazem, verapamil etc). Digoxin may also be used but is not first-line

How well did you know this?
1
Not at all
2
3
4
5
Perfectly
225
Q

what 2 investigations are most relevant/important in someone with suspected peripheral arterial disease?

A
  • ABPI: normal =1, intermittent claudication = 0.6-0.9, rest pain = 0.3-0.6, impending gangrene = <0.3 - Duplex ultrasound is best way to confirm diagnosis (should also do bloods + ECG)
How well did you know this?
1
Not at all
2
3
4
5
Perfectly
226
Q

what does left and right axis deviation look like on an ECG?

A

Right axis deviation: lead III has the most positive deflection and lead I is negatively deflected. Commonly seen in right ventricular hypertrophy. Left axis deviation: lead I has the most positive deflection and leads II and III are negative. Seen in individuals with heart conduction defects, may also be caused by left ventricular hypertrophy.

How well did you know this?
1
Not at all
2
3
4
5
Perfectly
227
Q

a patient with severe renal impairment presents with symptoms highly suspicious of a PE (Well’s score >4), what investigation should be done?

A

V/Q scan (if patient has renal impairment and therefore cannot have contrast, investigation of choice is V/Q scan rather than CTPA)

How well did you know this?
1
Not at all
2
3
4
5
Perfectly
228
Q

what would the FBC and blood film of someone with sickle cell anaemia typically show?

A

the haemoglobin level is in the range 6-8 g/dL with a high reticulocyte count of 10-20%; the blood films may show sickled erythrocytes and features of hyposplenism. (remember, most people will be diagnosed by heal prick/Guthrie test, not FBC)

How well did you know this?
1
Not at all
2
3
4
5
Perfectly
229
Q

what blood test can be useful in confirming a diagnosis of gout but what is the caveat?

A

serum uric acid level BUT should be obtained at least 2 weeks after the attack resolves, as it may be falsely low or normal during the attack

How well did you know this?
1
Not at all
2
3
4
5
Perfectly
230
Q

what is the difference between stable and unstable angina?

A

stable= pain is precipitated by predictable factors, usually exercise unstable= pain occurs at any time/ is unpredictable. should be managed as an ACS

How well did you know this?
1
Not at all
2
3
4
5
Perfectly
231
Q

what is seen on an ECG of someone in first degree heart block?

A

PR interval >0.2s (1 large square)

How well did you know this?
1
Not at all
2
3
4
5
Perfectly
232
Q

what is GCA/temporal arteritis?

A

GCA is a systemic immune-mediated vasculitis affecting medium-sized and large-sized arteries, particularly the carotid artery and its extracranial branches

How well did you know this?
1
Not at all
2
3
4
5
Perfectly
233
Q

what investigations should you consider in someone you suspect to have an acute ischaemic limb?

A
  • Bloods (FBC, ESR, glucose, lipids, thrombophilia screen) - Hand-held Doppler ultrasound scan may help demonstrate any residual arterial flow - ABPI: 1= normal, 0.6-0.9= claudication 0.3-0.6= rest pain, <0.3= impending gangrene - Investigations to find source of embolism e.g. ECG, echocardiogram - If diagnosis is in doubt, do an urgent arteriography
How well did you know this?
1
Not at all
2
3
4
5
Perfectly
234
Q

which coagulation test is most commonly abnormal in people with haemophilia?

A

Activated partial thromboplastin time (APTT) - usually prolonged but can be normal in mild disease

How well did you know this?
1
Not at all
2
3
4
5
Perfectly
235
Q

which cancer are people with Down’s syndrome particularly more likely to develop?

A

leukaemia

How well did you know this?
1
Not at all
2
3
4
5
Perfectly
236
Q

after being diagnosed with hypertension, what tool should be used to help determine their risk of a cardiovascular event?

A

QRISK2 (or 3 now?)

How well did you know this?
1
Not at all
2
3
4
5
Perfectly
237
Q

describe the pathophysiology behind varicose veins

A

VVs are due to incompetent valves that connect the deep and superficial venous systems. Leakage in a valve causes retrograde flow back into the vein. Unlike deep veins which are thick-walled and confined by fascia, superficial veins cannot withstand high pressure and eventually become dilated and tortuous. The failure of one valve puts pressure on its neighbours and may result in retrograde flow - and hence varicosity - of the entire local superficial venous network.

How well did you know this?
1
Not at all
2
3
4
5
Perfectly
238
Q

what might an x-ray of someone with RA show?

A

soft tissue swelling, periarticular osteopenia, loss of joint space, erosions and deformity (MUST know difference between RA and OA x-ray findings)

How well did you know this?
1
Not at all
2
3
4
5
Perfectly
239
Q

if there is a lesion of the hypoglossal nerve, which side will the tongue deviate towards?

A

towards the site of the lesion

How well did you know this?
1
Not at all
2
3
4
5
Perfectly
240
Q

when should medication be considered in someone experiencing seizures?

A

at least after the 2nd seizure

How well did you know this?
1
Not at all
2
3
4
5
Perfectly
241
Q

which type of antibodies are involved in anaphylaxis (type 1 hypersensitivity reactions)?

A

IgE antibodies

How well did you know this?
1
Not at all
2
3
4
5
Perfectly
242
Q

how would you expect a facial nerve lesion to present?

A
  • Facial weakness: > In LMN lesion, forehead is paralysed > In UMN lesion, forehead is spared - Taste loss
How well did you know this?
1
Not at all
2
3
4
5
Perfectly
243
Q

how should salicylate poisoning be managed?

A
  • Activated charcoal if ingested <1 hour ago - Aggressive rehydration - urinary alkalinization with intravenous sodium bicarbonate - enhances elimination of aspirin in the urine - haemodialysis in severe cases
How well did you know this?
1
Not at all
2
3
4
5
Perfectly
244
Q

what condition is Zollinger-Ellison syndrome most commonly associated with?

A

peptic ulcer disease

How well did you know this?
1
Not at all
2
3
4
5
Perfectly
245
Q

what does the Oxford/Bamford Classification System classify? And into what 4 groups does it classify them?

A

Ischaemic strokes Classified into: - Total Anterior Circulation Strokes (TACS) - Partial Anterior Circulation Strokes (PACS) - Posterior circulation syndrome (POCS) - Lacunar Syndrome (LACS)

How well did you know this?
1
Not at all
2
3
4
5
Perfectly
246
Q

what is Virchow’s triad (both its purpose and what it makes up)?

A

Virchow’s triad describes the three broad categories of factors that are thought to contribute to thrombosis. These categories are: - vessel wall damage (e.g. catheterisation, intravenous drugs, sclerotherapy, inflammatory vascular diseases) - stasis (e.g. varicose veins, immobilisation) - hypercoagulability (e.g. oral contraceptive medicines, inherited or acquired thrombophilia)

How well did you know this?
1
Not at all
2
3
4
5
Perfectly
247
Q

what are some of the common side effects of iron supplementation?

A

constipation, black stools, epigastric pain, nausea, diarrhoea (note, these often reduce with time and may be improved by taking iron supplements with food)

How well did you know this?
1
Not at all
2
3
4
5
Perfectly
248
Q

how should postural hypotension be managed?

A

General: - Review medication - Give advice on safe standing (e.g. go in to sitting position first, take time etc) - Encourage high-dietary salt Medication (only given if symptoms persist despite the above): - Fludrocortisone is recommended as first-line drug monotherapy

How well did you know this?
1
Not at all
2
3
4
5
Perfectly
249
Q

what are Cullen’s and Grey Turner’s sign?

A

Cullen’s sign= periumbilical bruising due to intraperitoneal hemorrhage Grey Turner’s sign= bruising of flanks due retroperitoneal hemorrhage Both can be seen in severe acute pancreatitis, as well as other conditions (such as ruptured ectopic)

How well did you know this?
1
Not at all
2
3
4
5
Perfectly
250
Q

what does the T-score have to be on a DEXA scan to diagnose osteoporosis?

A

T score ≤-2.5=. hip BMD 2.5 SD or more below the young adult reference mean

How well did you know this?
1
Not at all
2
3
4
5
Perfectly
251
Q

what is the maximum recommended alcohol units for men and women?

A

14 units

How well did you know this?
1
Not at all
2
3
4
5
Perfectly
252
Q

what is the QRISK2 tool used for?

A

used to estimate the likelihood of a patient having a cardiovascular event within the next 10 years (usually done after a patient is diagnosed with a risk factor e.g. hypertension) can also be used to help determine appropriate treatment. i.e. is risk is >10%, then offer a statin

How well did you know this?
1
Not at all
2
3
4
5
Perfectly
253
Q

briefly describe the pathophysiology behind anaphylaxis?

A

An anaphylactic reaction occurs when an allergen reacts with specific IgE antibodies (so the individual must be pre-sensitised so IgE antibodies have already developed) on mast cells and basophils (type 1 hypersensitivity reaction), triggering the rapid release of stored histamine and the rapid synthesis of newly formed mediators. These cause capillary leakage, mucosal oedema and ultimately shock and asphyxia

How well did you know this?
1
Not at all
2
3
4
5
Perfectly
254
Q

what are some of the possible causes of thrombocytopenia? (there are many)

A
  • Decreased production (disorders of bone marrow): malignancy (e.g. leukaemia, lymphoma, myeloma), viral infections (e.g. EBV, CMV, herpes simplex, varicella zoster), chemotherapy, alcohol - Decreased platelet survival: idiopathic thrombocytopenia purpura (ITP), medication (e.g. heparin, carbamazepine, ibuprofen, quinidine, quinine, rifampin, sulfamethoxazole, trimethoprim and vancomycin), DIC, sepsis, rheumatological disorders (e.g. SLE, rheumatoid arthritis), antiphospholipid
How well did you know this?
1
Not at all
2
3
4
5
Perfectly
255
Q

what is the classical presentation of an extradural haematoma?

A

The classical presentation is of a patient who initially loses, briefly regains and then loses again consciousness after a low-impact head injury. The brief regain in consciousness is termed the ‘lucid interval’ and is lost eventually due to the expanding haematoma and brain herniation.

How well did you know this?
1
Not at all
2
3
4
5
Perfectly
256
Q

a positive Trendelenburg sign is seen in the injury of which nerve?

A

superior gluteal nerve

How well did you know this?
1
Not at all
2
3
4
5
Perfectly
257
Q

what are 2 important pieces of advice to give to patients on Parkinson’s medication?

A

May cause day-time sleepiness (so should inform DVLA) Must be advised to avoid abrupt withdrawal due to the risk of neuroleptic malignant syndrome

How well did you know this?
1
Not at all
2
3
4
5
Perfectly
258
Q

what signs/symptoms would you expect to see in a patient with cannula-related phlebitis?

A

The commonest symptoms of any form of phlebitis are erythema and swelling along the venous track, leading to hardened, cord -like veins. The area can feel warm and patients may experience pain or discomfort during drug administration (and administration itself may also be difficult). Other possible symptoms include pus/exudate, pyrexia and, if allowed to progress, may lead to sepsis

How well did you know this?
1
Not at all
2
3
4
5
Perfectly
259
Q

what are the two types of second degree heart block and what do they look like on an ECG?

A

Mobitz type 1: PR interval gets progressively bigger until there is a dropped QRS complex, at which point the cycle restarts Mobitz type 2: 2 or 3 P waves occur successively, without a QRS complex following each (e.g. 2:1 or 3:1 ratio of P waves to QRS complexes)

How well did you know this?
1
Not at all
2
3
4
5
Perfectly
260
Q

how would you expect someone with Lewy Body Dementia to present?

A

Core features: - Fluctuating cognition (attention & alertness) - Spontaneous motor features of Parkinsonism e.g. tremor, rigidity, poverty of facial expression, festinating gait. Occurs in roughly 70% - Visual hallucinations (~70%) Additional Features: - Sleep (REM) disorder - Neuroleptic (antipsychotic) sensitivity - systematised delusions (70%)

How well did you know this?
1
Not at all
2
3
4
5
Perfectly
261
Q

what are the 2 characteristic features of ventricular tachycardia on an ECG?

A

tachycardia + broad QRS complex (>0.12s)

How well did you know this?
1
Not at all
2
3
4
5
Perfectly
262
Q

what condition often precedes myeloma?

A

monoclonal gammopathy of undetermined significance (MGUS)

How well did you know this?
1
Not at all
2
3
4
5
Perfectly
263
Q

how should first degree heart block be managed?

A

if asymptomatic, just requires monitoring (at low risk of progressing to higher degrees in block). If symptomatic, review medications (e.g. beta blockers), or refer to cardiology is symptoms cannot be controlled

How well did you know this?
1
Not at all
2
3
4
5
Perfectly
264
Q

what rules can you use to help differentiate small from large bowel on an abdominal x-ray?

A
  • small bowel usually lies more centrally, with the large bowel framing it around the periphery. - The small bowel’s mucosal folds are called valvulae conniventes and are seen across the full width of the bowel. - The large bowel wall features pouches or sacculation that protrude into the lumen that are known as haustra (in between which are the semilunar folds). The haustra are thicker than the valvulae conniventes of the small bowel. They also commonly do not appear to completely traverse the bowel.
How well did you know this?
1
Not at all
2
3
4
5
Perfectly
265
Q

roughly how long after the onset of acute ischaemic limb will necrosis begin to occur (i.e. how long do you have to salvage the limb)?

A

roughly 6 hours

How well did you know this?
1
Not at all
2
3
4
5
Perfectly
266
Q

what are some of the clinical features of acute myeloid leukaemia?

A
  • anaemia: pallor, lethargy, weakness - neutropenia: whilst white cell counts may be very high, functioning neutrophil levels may be low leading to frequent infections etc - thrombocytopenia: bleeding and petechiae (usually on lower limbs) - splenomegaly - bone pain
How well did you know this?
1
Not at all
2
3
4
5
Perfectly
267
Q

simply speaking, what is the role of vitamin D in the regulation of calcium?

A

vitamin D helps with calcium absorption, so vitamin D deficiency can cause hypocalcaemia

How well did you know this?
1
Not at all
2
3
4
5
Perfectly
268
Q

what conditions are associated with myasthenia gravis?

A
  • thymomas in 15% - autoimmune disorders: pernicious anaemia, autoimmune thyroid disorders, rheumatoid, SLE - thymic hyperplasia in 50-70%
How well did you know this?
1
Not at all
2
3
4
5
Perfectly
269
Q

which leads on a 12-lead ECG represent an anteroseptal of the heart?

A

V1, V2 (more septal view) + V3, V4, (more anterior view)

How well did you know this?
1
Not at all
2
3
4
5
Perfectly
270
Q

what are the 2 mechanisms by which haemolysis can occur?

A
  1. Extravascular (most common and occurs in the reticuloendothelial system): red cells are removed from the circulation by the mononuclear-phagocytic system either because they are intrinsically defective or because of the presence of bound immunoglobulins to their surfaces 2. Intravascular (in the circulation): due to complement fixation, trauma, or other extrinsic factors
How well did you know this?
1
Not at all
2
3
4
5
Perfectly
271
Q

what are some of the risk factors for gallstone formation?

A

Increasing age, female, family history, sudden weight loss (e.g. after obesity surgery), loss of bile salts (e.g. ileal resection, Crohn’s disease), diabetes, pregnancy, oral contraception

How well did you know this?
1
Not at all
2
3
4
5
Perfectly
272
Q

what would you expect to see in the CSF of someone with MS? (hard q)

A

oligoclonal bands and elevated CSF immunoglobulin G (IgG) and IgG synthesis rates are present in 80% of MS cases

How well did you know this?
1
Not at all
2
3
4
5
Perfectly
273
Q

what are partial/focal seizures and what are the 3 different types?

A

partial seizures= focal onset with features referable to a part of one hemisphere. Often seen with underlying structural disease types are: - simple partial seizure - complex partial seizure - partial seizure with secondary generalisation

How well did you know this?
1
Not at all
2
3
4
5
Perfectly
274
Q

how can heart rate be calculated using an ECG tracing?

A

If heart rate is regular, it can be calculated by counting the number of large squares in an R-R interval, and then dividing 300 by this number e.g. 4 squares in an R-R interval; 300/4= 75bpm If rhythm is irregular, count the number of QRS complexes on the rhythm strip (each is 10s long) and multiply by 6

How well did you know this?
1
Not at all
2
3
4
5
Perfectly
275
Q

what is the imaging of choice in suspected MS?

A

MRI brain and spine

How well did you know this?
1
Not at all
2
3
4
5
Perfectly
276
Q

what is the difference between de-novo acute heart failure and decompensated acute heart failure and what are some of the causes of each?

A

de-novo AHF: new-onset heart failure in people without known cardiac dysfunction acute decompensation: occurs in people with a background of heart failure and is more common than de-novo AHF de-novo causes: Usually as a result of ischaemia. Other causes include: viral myopathy, toxins and valve dysfunction decompensated causes: Acute coronary syndrome, hypertensive crisis, acute arrhythmia, valvular disease

How well did you know this?
1
Not at all
2
3
4
5
Perfectly
277
Q

what are the 2 most important investigations to consider when you suspect a patient of having peptic ulcer disease (or have suspicion of other differentials)?

A

Upper GI endoscopy to rule out malignancy if suspected (>55 with ALARMS symptoms) Test for H.pylori: carbon-13 urea breath test, stool antigen test, blood antibody test (less common)

How well did you know this?
1
Not at all
2
3
4
5
Perfectly
278
Q

what signs and symptoms might someone with infective endocarditis present with?

A

Symptoms: - Fever/chills - Weight loss/ loss of appetite - Fatigue, myalgia Signs: - Heart murmur: most commonly of aortic regurgitation - Splinter haemorrhages - Osler’s nodes: small tender red-to-purple nodules on the pulp of the terminal phalanges of the fingers and toes - Janeway lesions: irregular painless erythematous macules on the thenar and hypothenar eminence (usually with acute IE and S. aureus) - Clubbing: only 10% of cases and usually in long-standing subacute IE - Roth’s spots: retinal haemorrhages with pale centres - splenomegaly

How well did you know this?
1
Not at all
2
3
4
5
Perfectly
279
Q

ESR is a useful prognostic factor in which cancer?

A

Hodgkin’s lymphoma (>70= poor prognosis)

How well did you know this?
1
Not at all
2
3
4
5
Perfectly
280
Q

what is the name of the criteria used to help determine whether a tonsillitis case is due to a bacterial source and what do the criteria consist of?

A

centor criteria the criteria are: - temp >38 - tender anterior cervical lymphadenopathy - exudate present on tonsils - absence of cough a bacterial infection is likely if at least 3/4 features are present

How well did you know this?
1
Not at all
2
3
4
5
Perfectly
281
Q

which viral infection is particularly associated with lymphoma?

A

Ebstein-Barr virus (glandular fever)

How well did you know this?
1
Not at all
2
3
4
5
Perfectly
282
Q

what is the most common cause of sciatica?

A

90% are caused by spinal disc herniation pressing on lumbar or sacral nerve roots

How well did you know this?
1
Not at all
2
3
4
5
Perfectly
283
Q

what is Schober’s test (how is it done) and in when should it be done (what condition can it be used to help diagnose)?

A

Done if suspecting ankylosing spondylitis While the patient is in a standing position the examiner makes a mark approximately at the level of L5. Two points are marked: 5 cm below and 10 cm above this point (for a total of 15 cm distance). Then the patient is asked to touch his/her toes while keeping the knees straight. If the distance of the two points do not increase by at least 5 cm (with the total distance greater than 20 cm), then this is a sign of restriction in the lumbar flexion

How well did you know this?
1
Not at all
2
3
4
5
Perfectly
284
Q

when should direct current cardioversion be used first line inn a patient with VT?

A

when the patient is haemodynamically unstable

How well did you know this?
1
Not at all
2
3
4
5
Perfectly
285
Q

a 65 year old has been diagnosed with hypertension. He was started on ramipril and then amlodipine was added when this was insufficient to bring his BP down. His BP is still not controlled. What is the next medication you should consider?

A

thiazide-diuretic (e.g. indapamide or bendroflumethiazide)

How well did you know this?
1
Not at all
2
3
4
5
Perfectly
286
Q

what is the normal lifespan of RBCs?

A

roughly 120 days

How well did you know this?
1
Not at all
2
3
4
5
Perfectly
287
Q

what is an aplastic sickle cell crisis often precipitated by?

A

infection with parvovirus B19 (slapped cheek syndrome)

How well did you know this?
1
Not at all
2
3
4
5
Perfectly
288
Q

how does hyponatraemia usually present?

A
  • Hyponatraemia is often asymptomatic if it is mild to moderate - possible symptoms include: confusion, lethargy, anorexia, nausea, agitation, dizziness, disorientation, headache, seizures, coma, raised ICP
How well did you know this?
1
Not at all
2
3
4
5
Perfectly
289
Q

what does ‘P Mitrale’ refer to and what causes it?

A

bifid P wave seen in lead II and enhanced negative deflection seen in V1 due to left atrial enlargement (which is usually secondary to mitral stenosis).

How well did you know this?
1
Not at all
2
3
4
5
Perfectly
290
Q

what are some of the features of an essential tremor?

A
  • usually a distal symmetrical postural tremor of the upper limbs, usually of low amplitude with a fairly rapid frequency of 8-10 Hz - Present during actions and may be worse with arms outstretched - Commonly improved with rest and alcohol - Often strong family history
How well did you know this?
1
Not at all
2
3
4
5
Perfectly
291
Q

what kind of anaemia is G6PD deficiency associated with?

A

haemolytic anaemia

How well did you know this?
1
Not at all
2
3
4
5
Perfectly
292
Q

adenosine should be avoided in patients with which common condition?

A

asthma

How well did you know this?
1
Not at all
2
3
4
5
Perfectly
293
Q

what is the prognosis like for patients with reactive arthritis?

A
  • Reactive arthritis is usually self-limiting with resolution of symptoms by 3-12 months, but symptoms may persist for 12 months or more - There is a high incidence of recurrence, especially in those who are HLA-B27-positive
How well did you know this?
1
Not at all
2
3
4
5
Perfectly
294
Q

what are the different sub-types of delirium?

A
  • Hypoactive subtype: apathy and quiet confusion are present and easily missed. This type can be confused with depression. - Hyperactive subtype: agitation, delusions and disorientation are prominent and it can be confused with schizophrenia. - Mixed subtype: patients vary from hypoactive to hyperactive
How well did you know this?
1
Not at all
2
3
4
5
Perfectly
295
Q

what are some of the signs and symptoms of aortic stenosis?

A

Symptoms: dyspnoea on exertion, angina, dizziness, syncope Signs: - Murmur: ejection systolic crescendo-decrescendo murmur, heard loudest at the right upper sternal border, and often described as a rough, low-pitch sound. The murmur radiates to the carotids. - Narrow pulse pressure (small gap between systolic and diastolic BP) - Slow rising pulse - Soft/absent S2 - A fourth heart sound indicates left ventricular hypertrophy (LVH) in severe AS

How well did you know this?
1
Not at all
2
3
4
5
Perfectly
296
Q

how would you expect someone in anaphylaxis to present (early and later findings)?

A
  • Initially, patients usually develop skin symptoms, including generalised itching, urticaria and erythema, rhinitis, conjunctivitis and angio-oedema - Signs that the airway is becoming involved include itching of the palate or external auditory meatus, dyspnoea, laryngeal oedema (stridor) and wheezing (bronchospasm). - General symptoms include palpitations and tachycardia, nausea, vomiting and abdominal pain, feeling faint - with a sense of impending doom; and, ultimately, collapse and loss of consciousness - Airway swelling, stridor, breathing difficulty, wheeze, cyanosis, hypotension, tachycardia and reduced capillary filling suggest impending severe reaction
How well did you know this?
1
Not at all
2
3
4
5
Perfectly
297
Q

in a community setting, if you suspected someone had bacterial meningitis, what should you give them?

A

IM benzylpenicillin (if doesn’t delay transfer to hospital) exact dose depends on age but in adults is 1.2g

How well did you know this?
1
Not at all
2
3
4
5
Perfectly
298
Q

what is the epidemiology like for GCA?

A
  • more common in women - almost exclusively occurs in >50s - european descent (uncommon in people of African or Asian origin)
How well did you know this?
1
Not at all
2
3
4
5
Perfectly
299
Q

what degree of heart block is mobitz type 1 and what does it look like on an ECG?

A

second degree PR interval gets progressively bigger until there is a dropped QRS complex, at which point the cycle restarts

How well did you know this?
1
Not at all
2
3
4
5
Perfectly
300
Q

what is the definition of paroxysmal AF? (the paroxysmal part, not the AF part)

A

AF with spontaneous termination within seven days and most often within 48 hours.

How well did you know this?
1
Not at all
2
3
4
5
Perfectly
301
Q

what medications should be prescribed for long-term use after an MI?

A
  • Aspirin should be given to all patients, unless contra-indicated. The addition of clopidogrel has been shown to reduce morbidity and mortality - Beta-blocker (a rate-limiting calcium channel blocker such as verapamil may be considered if beta-blockers are contraindicated) - ACE inhibitor - Nitrates for angina - Statins may help prevent a recurring cardiac event
How well did you know this?
1
Not at all
2
3
4
5
Perfectly
302
Q

what are some of the clinical features of acute lymphoblastic leukaemia?

A
  • anaemia: lethargy and pallor - neutropaenia: frequent or severe infections - thrombocytopenia: easy bruising, petechiae - bone pain (secondary to bone marrow infiltration) - splenomegaly - hepatomegaly - fever is present in up to 50% of new cases (representing infection or constitutional symptom) - testicular swelling
How well did you know this?
1
Not at all
2
3
4
5
Perfectly
303
Q

what criteria must be met for a stoke to be classified as a ‘total anterior circulation stroke’ (TACS)?

A

All three of the following need to be present for a diagnosis of TACS: - Unilateral weakness (and/or sensory deficit) of the face, arm and leg - Homonymous hemianopia - Higher cerebral dysfunction (dysphasia, visuospatial disorder)

How well did you know this?
1
Not at all
2
3
4
5
Perfectly
304
Q

explain the pathophysiology involved in paracetamol overdose

A
  • When taken in normal therapeutic doses, paracetamol is converted through conjugation into non-toxic metabolites: sulfate and glucuronide. - A small proportion (~5%) is oxidised by cytochrome P450 into N-acetyl-p-benzoquinone imine (NAPQI) which is toxic to the liver. However, this toxic metabolite is usually rapidly broken down by glutathione, preventing harm. - In overdose, the sulfate and glucuronide pathways become saturated and more NAPQI is produced. The supplies of glutathione become depleted and thus eventually there is widespread hepatocyte damage leading to liver necrosis - Toxicity is increased in patients with induction of the P450 system through drugs such as rifampicin, phenobarbital, phenytoin, carbamazepine and alcohol
How well did you know this?
1
Not at all
2
3
4
5
Perfectly
305
Q

which leads on a 12-lead ECG represent a lateral view of the heart?

A

lead I, aVL, V5, V6

How well did you know this?
1
Not at all
2
3
4
5
Perfectly
306
Q

what is the most common type of brain tumour in children?

A

astrocytomas

How well did you know this?
1
Not at all
2
3
4
5
Perfectly
307
Q

what is the epidemiology of acute lymphoclastic leukaemia?

A

most common cancer in children, usually presenting 2-4 years of age, rare in adults

How well did you know this?
1
Not at all
2
3
4
5
Perfectly
308
Q

when can adrenaline and amiodarone be given in a cardiac arrest and what doses?

A

in a shockable rhythm: - give 1mg IV adrenaline and 300mg IV amiodarone after the 3rd shock. The adrenaline can be repeated roughly every 3-5 mins (or after alternate shocks) in a non-shockable rhythm: - Give 1mg IV adrenaline as soon as IV access is achieved. This can be repeated roughly every 3-5 mins (or after alternate shocks)

How well did you know this?
1
Not at all
2
3
4
5
Perfectly
309
Q

what is the toxic metabolite involved in paracetamol overdose?

A

N-acetyl-p-benzoquinone imine (NAPQI)

How well did you know this?
1
Not at all
2
3
4
5
Perfectly
310
Q

what are the 2 most common causative organisms of bacterial meningitis in adults?

A

Neisseria meningitidis and streptococcus pneumoniae

How well did you know this?
1
Not at all
2
3
4
5
Perfectly
311
Q

what are the different types/classifications of a subdural haematoma?

A
  • An acute SDH. - A subacute SDH (this phase begins 3-7 days after the initial injury). - A chronic SDH (this phase begins 2-3 weeks after the initial injury
How well did you know this?
1
Not at all
2
3
4
5
Perfectly
312
Q

what are some of the signs and symptoms someone with salicylate (aspirin) poisoning may present with?

A

Symptoms: nausea, vomiting, tinnitus, lethargy, dizziness, restlessness, sweating Signs: tachypnoea, hyper and hypoglycaemia, fever, seizures

How well did you know this?
1
Not at all
2
3
4
5
Perfectly
313
Q

what investigations should you consider doing for someone presenting with tricyclic antidepressant overdose?

A
  • serum cyclic antidepressant level does not correlate well with severity of toxicity and is a poor predictor of clinical outcome - ABG for evaluation of acidosis or hypoxia - Bloods - ECG: > Sinus tachycardia > Wide QRS complex > PR and QT interval prolonged
How well did you know this?
1
Not at all
2
3
4
5
Perfectly
314
Q

what is the diagnostic investigation for acute myeloid and acute lymphblastic leukaemia and what does it show?

A

Bone marrow biopsy: Presence of blast cells in ≥20% of the bone marrow cells confirms the diagnosis (myeloblasts in AML and lymphoblasts in ALL)

How well did you know this?
1
Not at all
2
3
4
5
Perfectly
315
Q

what are some of the features of fibromyalgia?

A
  • chronic pain: at multiple site, sometimes ‘pain all over’ - lethargy - cognitive impairment: ‘fibro fog’ (difficulty with concentration, memory loss or confusion) - sleep disturbance, headaches, dizziness are common - far more common in women than men
How well did you know this?
1
Not at all
2
3
4
5
Perfectly
316
Q

what are 2 examples of rate-limiting calcium channel blockers?

A

diltiazem and verapamil

How well did you know this?
1
Not at all
2
3
4
5
Perfectly
317
Q

in what situations should a paracetamol level be taken as soon a patient arrives with paracetamol overdose?

A
  • if the paracetamol was ingested more than 4 hours ago - if the overdose was staggered
How well did you know this?
1
Not at all
2
3
4
5
Perfectly
318
Q

what are some hypervolaemic causes of hyponatraemia?

A
  • Cirrhosis - Nephrotic syndrome - Congestive heart failure
How well did you know this?
1
Not at all
2
3
4
5
Perfectly
319
Q

what investigations should you consider in a patient presenting with paracetamol overdose?

A
  • Paracetamol level: take paracetamol level four hours post-ingestion, or as soon as the patient arrives if: > Time of overdose is greater than four hours. > Staggered overdose (in staggered overdoses, the level is not interpretable except to confirm ingestion) - U&E, creatinine - to look for renal failure and have a baseline. - LFTs: may be normal if the patient presents early but may rise to ALT >1000 IU/L. This is the enzyme level taken to indicate hepatotoxicity. - Glucose: hypoglycaemia is common in hepatic necrosis and capillary blood glucose should be checked hourly. - Clotting screen: prothrombin time is the best indicator of severity of liver failure and the INR should be checked 12-hourly. - Arterial blood gas: acidosis can occur at a very early stage, even when the patient is asymptomatic. It is seen in up to 10% of patients with ALF
How well did you know this?
1
Not at all
2
3
4
5
Perfectly
320
Q

in the context of seizures, what is an ‘aura’ and what kind of seizure does it most commonly precede?

A

An ‘aura’ is part of the seizure in which the patient is aware, and may precede its other manifestations. The aura may be a gut feeling, or déjà vu, or strange smells etc. It implies a partial (focal) seizure, often, but not necessarily from, the temporal lobe

How well did you know this?
1
Not at all
2
3
4
5
Perfectly
321
Q

what can thalidimide be used to help manage?

A

multiple myeloma

How well did you know this?
1
Not at all
2
3
4
5
Perfectly
322
Q

what are some possible causes of mitral regurgitation?

A
  • Coronary artery disease/ post-MI - Mitral valve prolapse - Infective endocarditis - Rheumatic fever - Congenital heart disease - Connective tissue diseases: Marfan’s syndrome, Ehlers-Danlos syndrome
How well did you know this?
1
Not at all
2
3
4
5
Perfectly
323
Q

a 12-lead ECG shows tachycardia and a narrow QRS complex, what is this?

A

supraventricular tachycardia

How well did you know this?
1
Not at all
2
3
4
5
Perfectly
324
Q

what gene is ankylosing spondylitis most commonly associated with?

A

A strong association with HLA-B27 exists, particularly in white western European populations.

How well did you know this?
1
Not at all
2
3
4
5
Perfectly
325
Q

what are the 2 categories that tension-type-headaches can be divided into?

A
  • Episodic TTH. This occurs on fewer than 15 days each month. It can evolve into the chronic variety. - Chronic TTH. This occurs on more than 15 days each month and has all the features of the episodic TTH. Chronic type is more likely to be medication-induced and to be associated with comorbidities such as depression
How well did you know this?
1
Not at all
2
3
4
5
Perfectly
326
Q

what are some of the causes of acute limb ischaemia?

A
  • Thrombosis (~40%): most cases of leg ischaemia result from the presence of thrombus at sites of atherosclerotic narrowing - Embolism (~38%): for example, left atrial thrombus in patients in atrial fibrillation - Graft/angioplasty occlusion (~15%) - Trauma - Compartment syndrome
How well did you know this?
1
Not at all
2
3
4
5
Perfectly
327
Q

what drugs can cause B12 deficiency?

A

metformin, colchicine, PPIs (these are rare causes)

How well did you know this?
1
Not at all
2
3
4
5
Perfectly
328
Q

what ABPI reading would expect in a patient: 1. with no PAD 2. with intermittent claudication 3. with pain at rest 4. with impending gangrene?

A
  1. 1 2. 0.6-0.9 3. 0.3-0.6 4. <0.3
How well did you know this?
1
Not at all
2
3
4
5
Perfectly
329
Q

how does pseudogout present?

A
  • Presents very similarly to gout, but symptoms are generally milder - Knee is most commonly affected joint, but can affect any joint
How well did you know this?
1
Not at all
2
3
4
5
Perfectly
330
Q

describe intermittent claudication

A
  • aching or burning in the leg muscles (can be calf, thigh or buttocks) following walking. Pain comes on more rapidly when walking uphill than on the flat - patients can typically walk for a predictable distance before the symptoms start (should obtain claudication distance in history) - can be present in both legs, but one is usually worse than the other - usually relieved within minutes of stopping - not present at rest (if it is, this indicates critical limb ischaemia!)
How well did you know this?
1
Not at all
2
3
4
5
Perfectly
331
Q

how long should a PR interval be on a normal ECG?

A

Should be 3-5 small squares (0.12-0.2s) (if prolonged, should consider 1st degree heart block)

How well did you know this?
1
Not at all
2
3
4
5
Perfectly
332
Q

which valve is most commonly affected by infective endocarditis?

A

the mitral valve is most commonly affected, followed by the aortic valve

How well did you know this?
1
Not at all
2
3
4
5
Perfectly
333
Q

what is neutropenia?

A

Neutropenia means a low neutrophil count. The normal range for neutrophils is 2.5-7.5 x 109/L. Moderate neutropenia is defined as a neutrophil count of 0.5-1.0 x 109/L. Severe neutropenia is a count of <0.5 x 109/L.

How well did you know this?
1
Not at all
2
3
4
5
Perfectly
334
Q

what would an ECG look like of someone in ventricular fibrillation?

A

ECG shows rapid, bizarre, irregular waves of widely varying amplitude and frequency (basically it’s all over the place)

How well did you know this?
1
Not at all
2
3
4
5
Perfectly
335
Q

what is the ‘wearing-off effect’ and ‘on-off effect’ associated with levodopa use?

A

A major issue with levodopa is the wearing-off effect, where patients symptoms worsen towards the end of the dosage interval. Increasing the dosage/frequency may overcome this, however patients may then start to get dyskinesia at the beginning of a dosage. When this occurs together, it is called the on-off effect.

How well did you know this?
1
Not at all
2
3
4
5
Perfectly
336
Q

how long would you expect someone to experience morning stiffness in both: osteoarthritis rheumatoid arthritis

A

OA: <30 mins (if any) RA: >1 hour

How well did you know this?
1
Not at all
2
3
4
5
Perfectly
337
Q

how should someone with suspected metastatic cord compression be managed?

A
  • 16mg IV dexamethasone stat + MRI spine within 1 hour - Consider neurosurgical intervention
How well did you know this?
1
Not at all
2
3
4
5
Perfectly
338
Q

briefly describe cardiogenic shock (i.e. the definition)

A

Cardiogenic shock occurs when there is failure of the pump action of the heart, resulting in a decrease in cardiac output causing reduced end-organ perfusion. This leads to acute hypoperfusion and hypoxia of the tissues and organs, despite the presence of an adequate intravascular volume

How well did you know this?
1
Not at all
2
3
4
5
Perfectly
339
Q

how would you expect someone in ACS to present?

A
  • Prolonged (>20mins) central or epigastric chest pain which may radiate to left arm, shoulder, neck or jaw. > Remember that certain patients (e.g. diabetics, elderly) may not have pain (silent MI) - Pain may be described as substernal pressure/crushing pain - Chest pain may be associated with sweating, nausea, vomiting, fatigue, shortness of breath and palpitations - Low-grade fever, pale and cool, clammy skin.
How well did you know this?
1
Not at all
2
3
4
5
Perfectly
340
Q

what happens to preload and afterload in: 1. hypovolaemic shock 2. cardiogenic shock 3. neurogenic shock

A
  1. preload: decreases, afterload: increases 2. preload: increases, afterload: increases 3. preload: decreases, afterload: decreases
How well did you know this?
1
Not at all
2
3
4
5
Perfectly
341
Q

what 4 x-ray changes are classically seen in OA?

A
  • Narrowing of joint space - Osteophytes forming at joint margins - subchondral cysts - subchondral sclerosis
How well did you know this?
1
Not at all
2
3
4
5
Perfectly
342
Q

how should B12 deficiency be managed (including dietary advice)?

A
  • Dietary advice: eggs, cereals, meat, fish - Hydroxocobalamin 1g IM injection 3 times a week for 2/52, then 1 mg every 2–3 months (if no neurological involvement)
How well did you know this?
1
Not at all
2
3
4
5
Perfectly
343
Q

what are the different doses of adrenaline that are used in anaphylaxis (3 different doses depending on age)?

A

0-6 years: 150mcg 6-12 years: 300mcg >12: 500mcg

How well did you know this?
1
Not at all
2
3
4
5
Perfectly
344
Q

which lobe do the majority of partial seizures arise from?

A

temporal

How well did you know this?
1
Not at all
2
3
4
5
Perfectly
345
Q

what is thrombophilia?

A

Thrombophilia refers to a predisposition to thromboembolism. In practice, the term is used to describe patients who are at significantly increased long-term risk of venous thromboembolism

How well did you know this?
1
Not at all
2
3
4
5
Perfectly
346
Q

if a patient has right-sided homonymous hemianopia, what does this mean and where is the lesion?

A

the right visual field has been lost in BOTH eyes lesion is at the left optic tract

How well did you know this?
1
Not at all
2
3
4
5
Perfectly
347
Q

what is the most important/significant possible complication of AF?

A

stroke

How well did you know this?
1
Not at all
2
3
4
5
Perfectly
348
Q

what are the definitions of persistent and permanent AF?

A

Persistent: not self-terminating; lasting longer than seven days, or prior cardioversion. Persistent AF may degenerate into permanent AF. Permanent: Long-standing AF (defined as over a year) that is not successfully terminated by cardioversion, when cardioversion is not pursued or has relapsed following termination.

How well did you know this?
1
Not at all
2
3
4
5
Perfectly
349
Q

a patient comes in with a possible PE. Their PE Well’s score is 3, how should they be investigated?

A

do a d-dimer, if positive do an immediate CTPA (if score is 4 or more, do immediate CTPA, is score is less than 4, do d-dimer)

How well did you know this?
1
Not at all
2
3
4
5
Perfectly
350
Q

what are some medications that can cause hyperkalaemia?

A

ACEi, ARBs, NSAIDs, potassium-sparing diuretics (e.g. spironolactone), beta-blockers, heparin, trimethoprim

How well did you know this?
1
Not at all
2
3
4
5
Perfectly
351
Q

with what symptoms would you expect someone with rhuematoid arthritis to typically present with?

A
  • Insidious, symmetrical (but occasionally can be asymmetrical) arthritis lasting >6 weeks. - RA can affect any synovial joint but typically affects the small joints of the hands and the feet. More joints are affected with progression of the disease. - Joint changes: heat and sometimes redness, swelling, pain, stiffness (especially in the early morning or after inactivity), progressive joint destruction and loss of joint function - Morning stiffness lasting >1 hour
How well did you know this?
1
Not at all
2
3
4
5
Perfectly
352
Q

in which form dementia should acetylcholinesterase inhibitors (E.g. rivastigmine, donepezil etc) be actively avoided?

A

in frontotemporal dementia- may worsen confusion

How well did you know this?
1
Not at all
2
3
4
5
Perfectly
353
Q

when should a AAA be monitored yearly (i.e. have repeat aortic ultrasounds on a yearly basis)?

A

when it is 3-4.4 cm in size

How well did you know this?
1
Not at all
2
3
4
5
Perfectly
354
Q

if a direct coombs test of someone with haemolytic anaemia comes back positive, what does this suggest?

A

that the cause of the haemolytic anaemia is immune e.g. haemolytic disease of newborn, blood transfusion reaction

How well did you know this?
1
Not at all
2
3
4
5
Perfectly
355
Q

chest pain which is classically relieved by sitting forward, and made worse by lying down, is associated with which condition?

A

pericarditis

How well did you know this?
1
Not at all
2
3
4
5
Perfectly
356
Q

If BP is found to be high in clinic, what can be offered to confirm it?

A

If between 140/90 and 180/120, offer ambulatory blood pressure monitoring (ABPM) to confirm the diagnosis of hypertension. If ABPM is unsuitable or the person is unable to tolerate it, offer home blood pressure monitoring (HBPM) (note that if >180, you should consider whether they need referral for same-day specialist assessment)

How well did you know this?
1
Not at all
2
3
4
5
Perfectly
357
Q

what is the most common feature of a focal seizure arising from the occipital lobe?

A

floaters/flashes

How well did you know this?
1
Not at all
2
3
4
5
Perfectly
358
Q

what type of breathing commonly occurs in the early stages of cardiac arrest?

A

Agonal breathing (occasional, irregular gasps) is common in the early stages of cardiac arrest and is a sign of cardiac arrest and should not be mistaken for a sign of life

How well did you know this?
1
Not at all
2
3
4
5
Perfectly
359
Q

what are some of the possible triggers for DIC?

A
  • Sepsis/severe infection, major trauma or burns - Some malignancies (acute myelocytic leukemia or metastatic mucin-secreting adenocarcinoma) - Obstetric disorders (amniotic fluid embolism, eclampsia, abruptio placentae, retained dead fetus syndrome) - Severe organ destruction or failure (severe pancreatitis, acute hepatic failure) - Vascular disorders (Kasabach-Merritt syndrome or giant haemangiomas, large aortic aneurysms) - Severe toxic or immunological reactions (blood transfusion reaction or haemolytic reactions, organ transplant rejection, snake bite).
How well did you know this?
1
Not at all
2
3
4
5
Perfectly
360
Q

after a patient with a first presentation of anaphylaxis has been stabilised and is ready for discharge, what should be organised/arranged for long-term management?

A
  • Refer to an allergist or allergy clinic to try to identify the allergen, so that it can be avoided in future. - Organise self-use of pre-loaded pen injections for future attacks (e.g. EpiPen; containing 0.3 mL of 1 in 1000 strength (that is, 300 micrograms) for adults; and for children 0.3 mL of 1 in 2000 (150 micrograms)). This again may be best done in allergy clinics. It is important that the technique for using these auto-injectors should be demonstrated and taught - Give a written self-management plan, information about anaphylaxis and biphasic reactions, and details of the possible signs and symptoms of a severe allergic reaction - Encourage the patient to wear a medical emergency identification bracelet or similar
How well did you know this?
1
Not at all
2
3
4
5
Perfectly
361
Q

what are some of the signs/symptoms of right-sided heart failure?

A
  • peripheral oedema - ascites - hepatomegaly - raised JVP
How well did you know this?
1
Not at all
2
3
4
5
Perfectly
362
Q

what causes ‘cogwheeling’ in Parkinson’s disease?

A

a tremor is superimposed on a limb with increased rigidity

How well did you know this?
1
Not at all
2
3
4
5
Perfectly
363
Q

which condition is commonly associated with acute mesenteric ischaemia?

A

AF (other emboli causing conditions are also associated, but AF is classically)

How well did you know this?
1
Not at all
2
3
4
5
Perfectly
364
Q

the murmur of which valvular disease most commonly radiates to the carotids?

A

murmur of aortic stenosis

How well did you know this?
1
Not at all
2
3
4
5
Perfectly
365
Q

what ECG features would you see in SVT?

A

12 lead ECG: will show tachycardia, narrow QRS complex (<0.12s), may have absent P waves

How well did you know this?
1
Not at all
2
3
4
5
Perfectly
366
Q

in which people should paracetamol dosages generally be halved?

A

people <50kg

How well did you know this?
1
Not at all
2
3
4
5
Perfectly
367
Q

what are some secondary causes of hypertension?

A
  • Renal disease is the most common cause of secondary hypertension. Causes include: glomerulonephritis, pyelonephritis, adult polycystic kidney disease, renal artery stenosis - Endocrine disease such as: Cushing’s syndrome, Conn’s syndrome, thyroid dysfunction, hyperparathyroidism - Drug causes: steroids, NSAIDs, monoamine oxidase inhibitors - Other causes: pregnancy, coarctation of the aorta
How well did you know this?
1
Not at all
2
3
4
5
Perfectly
368
Q

what are rituximab and infliximab examples of and very briefly how do they work?

A
  • they are biological therapies - rituximab= an anti-CD20 monoclonal antibody, results in B-cell depletion - infliximab= a TNF-inhibitor
How well did you know this?
1
Not at all
2
3
4
5
Perfectly
369
Q

what are some of the possible causes of complete heart block?

A

Complete heart block may occur due to: Myocardial fibrosis, previous MI (especially inferior), aortic valve calcification, cardiac surgery/trauma, digoxin toxicity

How well did you know this?
1
Not at all
2
3
4
5
Perfectly
370
Q

is the large or small bowel more likely to be obstructed?

A

80% of bowel obstructions are small bowel

How well did you know this?
1
Not at all
2
3
4
5
Perfectly
371
Q

what is the function of the facial nerve?

A
  • Muscles of facial expression - Taste to anterior 2/3rd of tongue - Lacrimation + salivation
How well did you know this?
1
Not at all
2
3
4
5
Perfectly
372
Q

which genetic condition is most often associated with bicuspid aortic valve?

A

Turner’s syndrome

How well did you know this?
1
Not at all
2
3
4
5
Perfectly
373
Q

what type of anaemia is associated with gallstones?

A

haemolytic anaemia- bilirubin stones can develop in patients with persistent haemolysis

How well did you know this?
1
Not at all
2
3
4
5
Perfectly
374
Q

how is someone with MS who is experiencing an acute relapse usually treated?

A

500mg methylprednisolone for 5/7 with gastroprotection (omeprazole 20mg OD) should always discuss with a specialist neurologist first

How well did you know this?
1
Not at all
2
3
4
5
Perfectly
375
Q

what age group is most affected by appendicitis?

A

most common between ages of 10-20, but can occur at any age

How well did you know this?
1
Not at all
2
3
4
5
Perfectly
376
Q

which investigation is absolutely contraindicated in a patient with a suspected extradural haematoma?

A

Lumbar punctures are absolutely contraindicated for extradural haematomas, as they result in a drop in CSF pressure, which may speed up brain herniation

How well did you know this?
1
Not at all
2
3
4
5
Perfectly
377
Q

what are some of the features of a headache cause by a increased ICP?

A
  • Headache which is worse in the morning/when lying down - Headache may wake from sleep - Headache may be exacerbated by coughing - Nausea and vomiting - May have visual changes - May have a change in mood e.g. more irritable
How well did you know this?
1
Not at all
2
3
4
5
Perfectly
378
Q

what type of haemophilia is more common?

A

type A is more common than type B

How well did you know this?
1
Not at all
2
3
4
5
Perfectly
379
Q

which causative organism is typically associated with pneumonia in alcoholics?

A

Klebsiella pneumoniae

How well did you know this?
1
Not at all
2
3
4
5
Perfectly
380
Q

what would you expect to see in an injury of the superior gluteal nerve?

A
  • paralysis of gluteus medius and minimum resulting in impaired hip abduction - positive Trendelenburg sign (when stood on one leg, pelvis drops on the side of the lifted leg, meaning muscle weakness/paralysis on opposite side)
How well did you know this?
1
Not at all
2
3
4
5
Perfectly
381
Q

what is Klumpke’s palsy (not features) and how is it caused?

A
  • Klmupke’s palsy= injury to the lower trunk of the brachial nerve plexus (C8-T1) - caused by hyperabduction of the arm through trauma or excessive traction on the arm during delivery
How well did you know this?
1
Not at all
2
3
4
5
Perfectly
382
Q

if a patient becomes haemodynamically unstable due to new-onset AF, how should they be managed?

A

emergency electrical cardioversion

How well did you know this?
1
Not at all
2
3
4
5
Perfectly
383
Q

what are some of the features of a tonic seizure?

A

sudden, brief stiffening of muscles of whole body, causing person to become rigid and fall. Recovery is swift but injuries may be sustained

How well did you know this?
1
Not at all
2
3
4
5
Perfectly
384
Q

with what signs/symptoms might someone with varicose veins present with?

A
  • Visually, dilated + tortuous veins are seen - Patient may also complain of leg pain/ache, leg swelling, leg itching - Possible skin changes include: ulceration, venous eczema, lipodermatosclerosis
How well did you know this?
1
Not at all
2
3
4
5
Perfectly
385
Q

what is myasthenia gravis?

A

Myasthenia gravis (MG) is an autoimmune disorder of neuromuscular transmission, resulting from binding of autoantibodies to components of the neuromuscular junction, most commonly the acetylcholine receptor

How well did you know this?
1
Not at all
2
3
4
5
Perfectly
386
Q

what are the reversible causes of cardiac arrest?

A
  • 4 Hs: hypoxia, hypothermia, hypovolaemia, hyperkalaemia (+ hypokalaemia, hypoglycaemia, hypocalcaemia, acidaemia and other metabolic disorders) - 4 Ts: cardiac tamponade, tension pneumothorax, thrombosis (coronary or pulmonary), toxins
How well did you know this?
1
Not at all
2
3
4
5
Perfectly
387
Q

how does reactive arthritis usually present?

A
  • Reactive arthritis usually develops 2-4 weeks after a genitourinary or gastrointestinal infection. About 10% of patients do not have a preceding symptomatic infection. - The onset is most often acute, with malaise, fatigue, and fever. - An asymmetrical, predominantly lower extremity, oligoarthritis is the major presenting symptom. - Low back pain often occurs
How well did you know this?
1
Not at all
2
3
4
5
Perfectly
388
Q

what is a common metabolic abnormality in someone with ACS?

A

hyperglycaemia is common in people admitted to hospital with ACS. Hyperglycaemia at the time of admission with ACS is a powerful predictor of poorer survival and increased risk of complications while in hospital, regardless of whether or not the patient has diabetes.

How well did you know this?
1
Not at all
2
3
4
5
Perfectly
389
Q

describe the murmur heard in mitral regurg

A

The murmur heard on auscultation of the chest is typically a pansystolic murmur described as “blowing”. It is heard best at the apex and radiating into the axilla. S1 may be quiet as a result of incomplete closure of the valve. (remember, left-sided murmurs, including mitral regurg, can be heard louder on expiration)

How well did you know this?
1
Not at all
2
3
4
5
Perfectly
390
Q

what are the two types of thalassaemia?

A

alpha and beta (depending on which polypeptide chains in haemoglobin is affected)

How well did you know this?
1
Not at all
2
3
4
5
Perfectly
391
Q

which medication is by far most responsible for cases of salicylate poisoning?

A

aspirin

How well did you know this?
1
Not at all
2
3
4
5
Perfectly
392
Q

what are some of the causes of small and large bowel obstruction and what are the most common causes?

A

Small bowel: adhesions (~75%) from previous surgeries, strangulated hernia, malignancy, foreign body, pseudobstruction Large bowel: colorectal carcinoma (most commonly), constipation, diverticular stricture, sigmoid or caecal volvulus

How well did you know this?
1
Not at all
2
3
4
5
Perfectly
393
Q

what, briefly, is coeliac disease?

A

An immune-mediated, inflammatory systemic disorder in which the gliadin fraction of gluten provokes a damaging immunological response in the proximal small intestinal mucosa, leading to malabsorption of nutrients Malabsorption is due to the death of enterocytes (absorptive cells) and villous atrophy

How well did you know this?
1
Not at all
2
3
4
5
Perfectly
394
Q

what monitoring should be done when starting someone on an ACE inhibitor?

A

do renal function and U&Es 1-2 weeks after starting ACEi, as can cause renal impairment and hyperkalaemia (should recheck 1-2 weeks after any dose changes)

How well did you know this?
1
Not at all
2
3
4
5
Perfectly
395
Q

with what features would you expect a DVT to present with?

A

Calf warmth/tenderness/swelling/erythema/pain, mild fever, pitting oedema Can progress to PE without DVT being clinically apparent

How well did you know this?
1
Not at all
2
3
4
5
Perfectly
396
Q

who do haemophilias more commonly affect?

A

boys (as X-linked recessive)

How well did you know this?
1
Not at all
2
3
4
5
Perfectly
397
Q

what is the first-line medication used in the management of primary Raynaud’s?

A

first-line medication: calcium channel blockers e.g. nifedipine

How well did you know this?
1
Not at all
2
3
4
5
Perfectly
398
Q

what exactly are diverticula?

A

A diverticulum consists of a herniation of mucosa through the thickened colonic muscle

How well did you know this?
1
Not at all
2
3
4
5
Perfectly
399
Q

what are the most common fragility fractures?

A

Fragility fractures occur most commonly in the spine (vertebrae), hip (proximal femur) and wrist (distal radius). They also occur in the humerus, pelvis etc

How well did you know this?
1
Not at all
2
3
4
5
Perfectly
400
Q

what condition should be screened for on a new diagnosis of type 1 diabetes?

A

coeliac disease

How well did you know this?
1
Not at all
2
3
4
5
Perfectly
401
Q

briefly described the histopathology in Alzheimer’s dementia

A

Accumulation of β-amyloid peptide, a degradation product of amyloid precursor protein, results in progressive neuronal damage, neurofibrillary tangles, increased number of amyloid plaques, and loss of the neurotransmitter acetylcholine Cortical atrophy is apparent in the temporal, frontal, and parietal areas

How well did you know this?
1
Not at all
2
3
4
5
Perfectly
402
Q

what are some of the features of a tonic-clonic seizure?

A

loss of consciousness. Limbs stiffen (tonic), then jerk (clonic). Breathing is shallow or temporarily suspended which may cause cyanosis. May be incontinent. May bite tongue. Usually <2mins, followed by post-ictal confusion, drowsiness, headache, soreness.

How well did you know this?
1
Not at all
2
3
4
5
Perfectly
403
Q

in RA, what must have been tried before a biological therapy (e.g. rituximab) can be considered?

A

at least 2 DMARDs (including methotrexate) must have been tried and been unsuccessful

How well did you know this?
1
Not at all
2
3
4
5
Perfectly
404
Q

how is thalassaemia inherited?

A

autosomal recessive inheritance

How well did you know this?
1
Not at all
2
3
4
5
Perfectly
405
Q

what is the most common cause of secondary Raynaud’s?

A

connective tissue disorders, particularly scleroderma, but can also occur in rheumatoid arthritis, SLE etc

How well did you know this?
1
Not at all
2
3
4
5
Perfectly
406
Q

what are the 2 main possible complications of pericarditis?

A
  • Pericardial effusion (which may lead to tamponade) - Constrictive pericarditis (thickening/scarring of the pericardium interferes with ventricular filling)
How well did you know this?
1
Not at all
2
3
4
5
Perfectly
407
Q

how many types of MND are there, which is the most common, and which carries the worst prognosis?

A

4 types Amyotrophic lateral sclerosis is the most common type Progressive bulbar palsy carries the worst prognosis

How well did you know this?
1
Not at all
2
3
4
5
Perfectly
408
Q

what are the 3 core features of Lewy Body dementia? (of which 2/3 must be present for a diagnosis to be made)

A
  • Fluctuating cognition (attention & alertness) - Spontaneous motor features of Parkinsonism e.g. tremor, rigidity, poverty of facial expression, festinating gait. Occurs in roughly 70% - Visual hallucinations (~70%)
How well did you know this?
1
Not at all
2
3
4
5
Perfectly
409
Q

what is the main investigation used to help confirm polymyalgia rheumatica?

A

ESR, plasma viscosity and/or CRP. Raised inflammatory markers are characteristic laboratory finding in PMR but may be normal (note: USS can also be done if diagnosis is still unclear, typically showing joint effusion/bursitis)

How well did you know this?
1
Not at all
2
3
4
5
Perfectly
410
Q

what is the maximum recommended potassium infusion rate via peripheral lines?

A

10mmol/hour

How well did you know this?
1
Not at all
2
3
4
5
Perfectly
411
Q

what is the most common extra-articular manifestation of ankylosing spondylisis?

A

anterior uveitis (occurs in 20-30% of patients)

How well did you know this?
1
Not at all
2
3
4
5
Perfectly
412
Q

how should a haemodynamically stable patient in SVT be managed?

A

1st line: reflex vagal stimulation e.g. by Valsalva manoeuvre (forced exhalation against a closed airway), immersing the face in ice cold water, or carotid sinus massage 2nd line: IV adenosine (6 milligrams initially, followed by 12 milligrams 1-2 mins later if needed) 3rd line (or if adenosine is contraindicated, e.g. asthmatics): IV verapamil hydrochloride 4th line: direct current cardioversion Recurrent episodes of paroxysmal supraventricular tachycardia can be treated by catheter ablation

How well did you know this?
1
Not at all
2
3
4
5
Perfectly
413
Q

what is the normal size of a QRS complex?

A

0.12s (3 small squares)

How well did you know this?
1
Not at all
2
3
4
5
Perfectly
414
Q

how are osteopenia and osteoporosis differentiated using a DEXA scan?

A
  • Osteopenia (low bone mass): hip BMD between 1 and 2.5 SD below the young adult reference mean (T score less than -1 but above -2.5). - Osteoporosis: hip BMD 2.5 SD or more below the young adult reference mean (T score ≤-2.5).
How well did you know this?
1
Not at all
2
3
4
5
Perfectly
415
Q

what signs might be found on examination of someone with peripheral arterial disease?

A
  • Diminished or absent foot pulses - Pale and/or cold limb/foot with possible hair loss - There may be poorly healing wounds of the extremities (ulceration or gangrene suggests critical limb ischaemia) - may have positive Buerger’s test in critical limb ischaemia
How well did you know this?
1
Not at all
2
3
4
5
Perfectly
416
Q

how are haemophilias inherited?

A

X-linked recessive manner

How well did you know this?
1
Not at all
2
3
4
5
Perfectly
417
Q

what causes reactive arthritis (including examples of organisms and the most common organism)?

A

reactive arthritis occurs due to a prior (roughly 1 to 6 weeks prior) gastrointestinal or urogenital infection: - Post-enteric: the three most commonly associated enteric pathogens are Campylobacter, Salmonella and Shigella species - Post-venereal: Chlamydia species (most commonly: Chlamydia trachomatis) or human immunodeficiency virus (HIV). Chlamydia species are traditionally thought to be the most common cause of ReA

How well did you know this?
1
Not at all
2
3
4
5
Perfectly
418
Q

briefly describe all the different types of heart block (in terms of ECG findings)

A

First-degree heart block: PR interval > 0.2 seconds Second-degree heart block: - type 1 (Mobitz I, Wenckebach): progressive prolongation of the PR interval until a dropped beat occurs - type 2 (Mobitz II): PR interval is constant but the P wave is often not followed by a QRS complex Third-degree (complete) heart block: there is no association between the P waves and QRS complexes

How well did you know this?
1
Not at all
2
3
4
5
Perfectly
419
Q

what would you expect to see on ultrasound of an individual with gallstones? (both cholecystitis and cholangitis)

A

In cholecystitis, would expect to see a distended gallbladder and/or thickened gallbladder wall (+ stones themselves in gallbladder) In cholangitis, would expect to see a dilated common bile duct (+ stones themselves in duct)

How well did you know this?
1
Not at all
2
3
4
5
Perfectly
420
Q

what are some risk factors for infective endocarditis?

A

valvular heart disease, valve replacement (prosthetic valve), structural congenital heart diseases, previous IE, hypertrophic cardiomyopathy, IVDU

How well did you know this?
1
Not at all
2
3
4
5
Perfectly
421
Q

what are some of the possible causes of B12 deficiency?

A
  1. pernicious anaemia (80% of cases) 2. Potential after-effects of surgery e.g. gastrectomy or ileal resection. 3. Bacterial overgrowth or parasitic infestation. 4. HIV infection 5. Dietary deficiency, which may occur in strict vegans but, even in them, it is rare 6. Drug-induced deficiency: metformin, colchicine, PPIs
How well did you know this?
1
Not at all
2
3
4
5
Perfectly
422
Q

which cancers most commonly cause spinal mets (which may lead to spinal cord compression)?

A

breast, lung, prostate are most common thyroid and kidney are not unusual

How well did you know this?
1
Not at all
2
3
4
5
Perfectly
423
Q

what are bouchard’s and heberden’s nodes?

A

Bony swelling and deformity seen in OA due to osteophytes - in the fingers this presents as swelling at the distal interphalangeal joints (Heberden’s nodes) or swelling at the proximal interphalangeal joints (Bouchard’s nodes)

How well did you know this?
1
Not at all
2
3
4
5
Perfectly
424
Q

what are some examples of small, medium and large vessel vassculitides?

A
  • Small-vessel e.g. Henoch-Schönlein purpura, Granulomatosis with polyangiitis (also known as Wegener’s granulomatosis, can also affect medium sized vessels) - Medium-sized vessel e.g. Polyarteritis nodosa, Churg-Strauss syndrome, Kawasaki’s disease - Large vessel e.g. Takayasu’s arteritis, Giant cell arteritis
425
Q

name all 12 cranial nerves in order

A

1: Olfactory 2: Optic 3: Oculomotor 4: Trochlear 5: Trigeminal 6: Abducens 7: Facial 8: Vestibulocochlear 9: Glossopharyngeal 10: Vagus 11: Accessory 12: Hypoglossal

426
Q

how is hydrocephalus managed?

A
  • An external ventricular drain (EVD) is used in acute, severe hydrocephalus and is typically inserted into the right lateral ventricle and drains into a bag at the bedside - A ventriculoperitoneal shunt (VPS) is a long-term CSF diversion technique that drains CSF from the ventricles to the peritoneum - In obstructive hydrocephalus, the treatment may involve surgically treating the obstructing pathology
427
Q

what are the 2 shockable rhythms in a cardiac arrest?

A

ventricular fibrillation pulseless ventricular tachycardia

428
Q

what is given for dual antiplatelet therapy in someone with ACS?

A

Aspirin (300 milligrams orally) and ticagrelor (180mg) (clopidogrel and prasugrel may be used instead of ticagrelor) (note the two doses given are the loading doses, they should be continued at lower doses long-term)

429
Q

what are some hypovolaemic causes of hyponatraemia?

A
  • Vomiting - Diarrhoea - Burns - Diuretic use - Prolonged exercise/sweating - Addison’s disease (insufficient production of cortisol and aldosterone)
430
Q

what is the pterion and what is its clinical significance?

A

The pterion is the region on the skull where the frontal, parietal, temporal, and sphenoid bones join together It is therefore a weak point on the skull, under which the middle meningeal artery runs (and so can be ruptured by injury to this region, which can cause an extradural haematoma)

431
Q

what is the name of the classification system used in heart failure to classify functional ability and what are the different classes?

A

The New York Heart Association’s (NYHA) Classification of Heart Failure has provided a clinically useful, functional classification: - Class I: no symptoms on ordinary physical activity. - Class II: slight limitation of physical activity by symptoms. - Class III: less than ordinary activity leads to symptoms. - Class IV: inability to carry out any activity without symptoms.

432
Q

what is the definition of acute AF? (i.e. what does the acute bit refer to, not the definition of AF)

A

AF with onset within the previous 48 hours.

433
Q

which medications are associated with QT prologation?

A

amiodarone tricyclic antidepressants SSRIs (particularly citalopram) erythromycin haloperidol ondansetron methadone

434
Q

what are some of the symptoms/signs of delirium tremens and when do they appear?

A
  • Tremor, fits, hallucinations, global confusion, severely uncoordinated, autonomic hyperactivity (raised heart rate and blood pressure) - The altered mental state differentiates delirium tremens from normal alcohol withdrawal - Can be fatal if untreated symptoms begin 48-72 hours following last drink
435
Q

what are the features of an allergic (non-anaphylactic) transfusion reaction?

A

urticaria +/- pyrexia +/- itching

436
Q

visual hallucinations are most common in which form of dementia?

A

lewy body dementia

437
Q

how can someone in second degree heart block and who is symptomatic with it be managed?

A

Acutely symptomatic patients with low ventricular rates can be treated with atropine and/or temporary pacemaker insertion. (long-term, insertion of a permanent pacemaker may be required, particularly for mobitz type II block)

438
Q

what can chemotherapy increase the risk of in HL?

A

leukaemia

439
Q

what drug class do donepezil, galantamine and rivastigmine belong to? and what condition are they predominantly used in?

A

Acetlycholinesterase inhibitors Mainly used in the treatment of mild-moderate Alzheimer’s dementia

440
Q

what is the most common cause of peptic ulcers?

A

H.pylori (90% of duodenal ulcers, ~80% of gastric ulcers)

441
Q

what are some of the clinical features of lymphoma (both HL and NHL)?

A

(both types present similarly) - long-standing painless lymphadenopathy (usually in neck) - in HL, may be associated with pain on drinking alcohol (<10% of cases) - Systemic symptoms of drenching night sweats, unexplained fever >38°C, and weight loss of >10% over six months are termed B symptoms and are identified in approximately 25% of patients with HL (usually only seen in advanced stages of NHL) - Hepato/splenomegaly - chest discomfort/cough/dyspnoea (if mediastinal mass)

442
Q

what would you expect the glucose level to be in a lumbar puncture done in: 1. a healthy patient 2. a patient with viral meningitis 3. a patient with bacterial meningitis?

A
  1. 45-100 (2/3rds of serum level) 2. 30-70 (roughly normal) 3. <40 (<40% of serum level)
443
Q

what is the first-line medication used for managing delirium tremens?

A

oral lorazepam

444
Q

what are some common triggers of anaphylaxis?

A
  • Food: peanuts, shellfish, sesame, egg - Venom: bee/wasp sting - Medication: antibiotics, opioids, NSAIDs
445
Q

what specicial test can be conducted to help determine whether a patient has severe peripheral arterial disease/ critical limb ischaemia? and how is it conducted?

A

Buerger’s test is used to assess the adequacy of the arterial supply to the leg. It is performed in two stages: - With the patient supine, elevate both legs to an angle of 45 degrees and hold for one to two minutes. Observe the colour of the feet. Pallor indicates ischaemia. It occurs when the peripheral arterial pressure is inadequate to overcome the effects of gravity. The poorer the arterial supply, the less the angle to which the legs have to be raised for them to become pale. - Then sit the patient up and ask them to hang their legs down over the side of the bed at an angle of 90 degrees. Gravity aids blood flow and colour returns in the ischaemic leg. The skin at first becomes blue, as blood is deoxygenated in its passage through the ischaemic tissue, then returns to a normal pink colour, and then goes beyond to a red-ish colour, due to reactive hyperaemia from post-hypoxic vasodilatation. It is sometimes described as a ‘sunset foot’.

446
Q

how should pseudogout be managed?

A
  • Attacks usually resolve within 10 days - Treatment is symptomatic: RICE, NSAIDs, steroids (oral or injection)
447
Q

what is the most important investigation to conduct when a patient presents acutely with signs/symptoms of a stroke?

A

CT head- can rule out haemorrhagic stroke and therefore determine whether thrombolysis is indicated

448
Q

where should the electrodes be placed in a 12-lead ECG?

A

RA (red): right arm LA (yellow): left arm LL (green): left leg RL (black): right leg V1 (red): 4th intercostal, right sternal edge V2 (yellow): 4th intercostal, left sternal edge V3 (green): half way between V2 and V4 V4 (brown): 5th intercostal, midclavicular V5 (black): anterior axillary line, same plane as V4 V6 (violet): mid-axillary line, same plane as V4

449
Q

what is the most common cause of B12 deficiency?

A

pernicious anaemia

450
Q

which scoring tool can be used to help determine a patient’s risk of bleeding when considering starting an anticoagulant?

A

HAS BLED

451
Q

what is micrographia and what condition is it most commonly associated with?

A

micrographia: where handwriting gets smaller (decreases in amplitude) as the individual writes associated with Parkinson’s disease

452
Q

what are the 5 different types of shock?

A

septic, hypovolaemic, neurogenic, cardiogenic, anaphylactic

453
Q

what MUST you remember when managing hyponatraemia?

A

should be corrected GRADUALLY Osmotic demyelination syndrome (central pontine myelinolysis) can occur due to over-correction of severe hyponatremia. To avoid this, Na+ levels are only raised by 4 to 6 mmol/l in a 24-hour period. Symptoms usually occur after 2 days and are usually irreversible. Dysarthria, dysphagia, paraparesis or quadriparesis, seizures, confusion, and coma. Patients are awake but are unable to move or verbally communicate, also called ‘Locked-in syndrome’

454
Q

which valve condition is associated with a malar flush?

A

mitral stenosis

455
Q

what are the 2 main types of ventricular tachycardia?

A

VT may be monomorphic (typically regular rhythm originating from a single focus with identical or similar QRS complexes, most commonly caused by MI) or polymorphic (may be irregular rhythm, with beat to beat variation in QRS complexes e.g. torsades de pointes)

456
Q

the valsalva manoeuvre is used first-line to try and resolve which arrythmia?

A

supraventricular tachycardia

457
Q

what are some of the possible blood results you would expect to see in an individual with myeloma?

A
  • Impaired renal function - anaemia (normochromic, normocytic) - leukopenia - thrombocytopenia - hypercalcaemia - persistently raised plasma viscosity or erythrocyte sedimentation rate (ESR)
458
Q

what is the other name for 3rd degree heart block?

A

complete heart block

459
Q

what are some of the clinical features of superior vena cava obstruction?

A
  • dyspnoea + cough - swelling of the face, neck and arms - conjunctival and periorbital oedema may be seen - dilated veins over the arms, neck and anterior chest wall - headache: often worse in the mornings - visual disturbance - pulseless jugular venous distension
460
Q

briefly, how is pericarditis managed?

A

A combination of NSAIDs and colchicine is now generally used for first-line for patients with acute idiopathic or viral pericarditis

461
Q

very simplistically, describe the pathophysiology behind complete heart block

A

In 3rd degree heart block, no impulses are passed from the atria to ventricles. The ventricles contract due to pacemaker cells in the ventricles causing a ‘ventricular escape rhythm’. As a result however, there is no coordination between contraction of the atria and ventricles

462
Q

what is the CHA2DS2-VASc assessment tool used for?

A

helps to determine the risk of stroke in a patient with AF

463
Q

what features are needed for a diagnosis of hyperosmolar hyperglycaemia state?

A
  • Hypovolaemia - Marked hyperglycaemia (>30 mmol/L) without significant hyperketonaemia (pH >7.3, bicarbonate >15 mmol/L) - Osmolality >320 mosmol/kg
464
Q

what is the difference between low-output and high-output heart failure and what are some causes of high-output heart failure?

A
  • Low-output: cardiac output id reduced and fails to increase normally with exertion. - High-output: this is rare. In these instances, cardiac output is normal or increased, but the output is insufficient to meet the bodies demand. It is caused by underlying disease processes, such as anaemia, hyperthyroidism, sepsis, pregnancy etc
465
Q

which organism most commonly causes infective endocarditis?

A

Staphylococcus aureus is the most common cause of infective endocarditis. It is particularly common in acute presentations and IVDUs

466
Q

how should a patient diagnosed with a STEMI be managed acutely?

A
  • Oxygen, aspirin + clopidogrel (/ticagrelor), nitrates - slow intravenous injection of diamorphine hydrochloride or morphine should be given for pain + antiemetic such as metoclopramide hydrochloride - Patency of the occluded artery can be restored by percutaneous coronary intervention. This should be done within 90 minutes of diagnosis > Patients undergoing PCI should be given a glycoprotein IIb/IIIa inhibitor to reduce the risk of immediate vascular occlusion + heparin (unfractionated) or a LMWH - In patients who cannot be offered percutaneous coronary intervention within 90 minutes of diagnosis, a thrombolytic drug should be administered along with either heparin (unfractionated) (for maximum 2 days), a low molecular weight heparin (e.g. enoxaparin sodium), or fondaparinux sodium - Patients who do not receive reperfusion therapy (with percutaneous coronary intervention or a thrombolytic) should be treated with either fondaparinux sodium, enoxaparin sodium, or heparin (unfractionated) - Depending on distribution of coronary artery disease, coronary artery bypass grafting (CABG- veins or arteries harvested from elsewhere, often saphenous vein and thoracic artery, and used as grafts where the blockages occur) may be performed. - Beta-blockers and/or ACE inhibitors may also be considered
467
Q

what are some of the features of left-sided heart failure?

A
  • dyspnoea: > Exertional (ask about exercise tolerance) + at rest > Orthopnoea (ask about no. of pillows) and paroxysmal nocturnal dyspnoea - Cough (may be worse at night and associated with pink/frothy sputum) - Fatigue - Signs: cyanosis, tachycardia, displaced apex beat, bibasal crackles (classically- may also be heard as a wheeze)
468
Q

in which instances might rhythm control be considered instead of rate control in AF?

A
  • With new-onset AF (<48h). - Whose AF has a reversible cause (for example a chest infection). - Who have heart failure thought to be primarily caused, or worsened, by AF.
469
Q

describe the murmur heard in aortic stenosis

A

ejection systolic crescendo-decrescendo murmur, heard loudest at the right upper sternal border, and often described as a rough, low-pitch sound. The murmur radiates to the carotids

470
Q

how should status epilepticus be managed in a hospital setting?

A
  • Secure airway + 15L O2 via non-rebreathe - Seizures lasting longer than 5 minutes should be treated urgently with intravenous lorazepam (repeated once after 10 minutes if seizures recur or fail to respond) - If, after initial treatment with benzodiazepines, seizures recur or fail to respond 25 minutes after onset, phenytoin sodium should be used; contact intensive care unit if seizures continue - If these measures fail to control seizures 45 minutes after onset, patient likely will have to be anaesthetised - Throughout the above, if possible, treat/manage cause: > Correct any hypoglycaemia with glucagon IM injection or 20% glucose IV infusion > Parenteral thiamine (e.g. pabrinex) should be considered if alcohol abuse is suspected
471
Q

what is a ‘swan neck deformity’ and ‘Boutonniere’s deformity’ and what condition are they associated with?/

A

seen in RA Swan neck deformity= hyperextension of PIP, flexion of DIP Boutonniére deformity= flexion of PIP, extension at DIP

472
Q

what investigation can be done to confirm a diagnosis of anaphylaxis?

A

Serum mast-cell tryptase can be measured in cases of anaphylaxis, particularly to clarify diagnosis where ambiguity exists. Serum tryptase is the preferred marker for demonstrating mast-cell degranulation and peaks 1 hour after an anaphylactic reaction

473
Q

how would you expect someone with vascular dementia to present?

A
  • Patient will often have a history of cardiovascular disease/ stroke - Characteristically has a sudden onset and a step-wise progression - Presentation varies significantly, as does speed of progression. Possible signs/symptoms include: > Focal neurological abnormalities (e.g. visual disturbances) > Difficulty with attention and concentration > Seizures > Mood and behaviour changes > Early presence of disturbance in gait, unsteadiness and frequent, unprovoked falls > Insight is usually retained until late
474
Q

what investigation is crucial in diagnosing thalassaemia?

A

haemoglobin electrophoresis

475
Q

after a patient has been given a 3rd shock in a cardiac arrest, what medication can be given?

A
  • Give 1mg IV adrenaline. This can be repeated roughly every 3-5 mins (or after alternate shocks) - Give amiodarone 300mg IV
476
Q

what would you expect the opening pressure to be in a lumbar puncture done in: 1. a healthy patient 2. a patient with viral meningitis 3. a patient with bacterial meningitis

A
  1. <180 mm of H2O 2. Normal/slight elevation 3. 200-500 (elevated)
477
Q

how is unstable angina differentiated from an NSTEMI?

A

there is no rise in troponin in unstable angina, there is in NSTEMI

478
Q

how should oral anticoagulants (warfarin or NOAC) be used in the management of a DVT?

A

warfarin or a NOAC should be offered within 24 hours of diagnosis (along with treatment dose LMWH or fondaparinux) and continued for 3 months

479
Q

what are some risk factors for aortic dissection?

A

Hypertension, male, smoking, raised cholesterol, aortic disease/aortic valve disease (or family history of), blunt chest trauma, Marfan’s syndrome, Ehlers Danlos Syndrome

480
Q

what should be given immediately if you suspect a patient of having encephalitis?

A

intravenous aciclovir should be started asap in all cases of suspected encephalitis

481
Q

what would you expect to seen on a blood film of someone with chronic lymphocytic leukaemia?

A

smudge cells (smear cells)

482
Q

when should a paracetamol level ideally be taken in a patient presenting with paracetamol overdose?

A

4 hours post ingestion

483
Q

someone has recently been diagnosed with stable angina and are prescribed a GTN spray for their symptoms. Besides technique, what important safety-netting info should you give them?

A
  • to repeat the dose after 5 minutes if the pain has not gone - to call an emergency ambulance if the pain has not gone 5 minutes after taking a second dose (i.e. 15 mins after onset of pain).
484
Q

what is the main drug of choice in the management of: - supraventricular tachycardias? - ventricular tachycardias?

A

SVT= adenosine VT= amiodarone

485
Q

with what signs and symptoms does appendicitis usually present with?

A

Symptoms: classically periumbilical pain that moves to the RIF and is exacerbated by coughing/movement, nausea, vomiting, anorexia, constipation/diarrhoea General signs: tachycardia, fever, furred tongue, lying still, coughing hurts, halitosis Specific signs: guarding, rebound tenderness (suggests peritonitis), percussion tenderness, Rovsing’s sign (pressure in LIF increases pain in RIF), Psoas sign (pain on extending hip when lying on side), deep tenderness at McBurney’s point (1/3rd distance from anterior superior iliac spine to the umbilicus)

486
Q

what is the most common cause of acute pericarditis?

A

Viral infection is the most common cause. Many viruses can cause pericarditis, including coxsackieviruses, echoviruses, influenza viruses, adenoviruses, rubella, mumps, HIV etc

487
Q

what is Brudzinski’s sign and what condition is it seen in?

A

flexion of neck causes flexion of hips seen in meningitis

488
Q

what cause of haemolytic anaemia also commonly causes leg ulcers?

A

sickle cell anaemia

489
Q

in which condition might it be useful to calculate a FRAX score? and what is the FRAX score?

A

osteoporosis calculates the ten-year probability of a major osteoporotic fracture in people aged 40-90

490
Q

what possible physical signs might you see on examination of someone with advanced rheumatoid arthritis?

A
  • Symmetric hot, tender swelling of PIPs and MCPJs [sparing of DIPs] - Ulnar deviation at MCPJ - Swan neck deformity [hyperextension of PIP, flexion of DIP] - Boutonniére deformity [flexion of PIP, extension at DIP] - Rheumatoid nodules on extensor tendons, elbows, joints
491
Q

with what signs/symptoms might sickle cell disease present with?

A

Anaemia, jaundice, pallor, lethargy, growth restriction/ short stature, general weakness, increased susceptibility to infections, splenomegaly, delayed puberty, sickle cell crises

492
Q

how does Raynaud’s classically present?

A
  • The patient may initially notice pallor of the distal portion of one or more digits (thumb is usually spared). There is usually a clear demarcation line between affected and normal skin. - There then follows a feeling of numbness or pain in the digit(s) accompanied by cyanosis (blueing) of the digit. - Finally, there is a hyperaemic phase where the digit becomes red and feels warm.
493
Q

how might meningitis present?

A

Early: - headache, leg pains, cold hands and feet, abnormal skin colour Later: - Meningism: neck stiffness, photophobia, Kernig’s sign (pain + resistance on passive knee extension with hip fully flexed) - Reduced consciousness, coma - Seizures (~20%) with or without focal CNS signs (~20%) with or without opisthotonos (spasm of muscles causing backward arching of head, neck and spine) - Petechial rash (non-blanching, may only be 1 or 2 spots or none). This is a sign of septicaemia - Brudzinski’s sign (hips flex on bending head forward) - Signs of sepsis: slow cap refill, hypotension, tachycardia, reduced urine output

494
Q

briefly, how should bacterial meningitis be managed?

A
  • Start broad spectrum IV antibiotics immediately (exact ones depend on trust guidelines) e.g. ceftriaxone or cefotaxime. Switch to more specific antibiotic when causative organism is identified - Dexamethasone should be given to children >3 months if suspect bacterial meningitis to reduce the risk of neurological sequelae. - Treat/manage sepsis (BUFALO) - Household members given appropriate prophylactic antibiotic (rifampicin)
495
Q

what is the most common cause of non-traumatic/spontaneous subarachnoid haemorrhage?

A

85% are caused by ruptured berry aneurysm within the circle of Willis

496
Q

in what conditions can BNP levels be reduced (making it more difficult to diagnose heart failure)?

A
  • Obesity. - African-Caribbean origin. - Medication: diuretics, ACE inhibitors, angiotensin-II receptor blockers, beta-blockers, mineralocorticoid receptor antagonist
497
Q

what investigation is diagnostic for coeliac disease?

A

small bowel biopsy

498
Q

briefly, how should Raynaud’s be managed?

A
  • Advice on prevention of attacks e.g. smoking cessation, avoid beta-blockers, wear gloves etc - first-line medication: calcium channel blockers e.g. nifedipine - all patients with suspected secondary Raynaud’s phenomenon should be referred to secondary care
499
Q

of adults who experience migraine, what proportion have migraine with aura?

A

1/3rd

500
Q

optic neuritis is a common presenting feature of which neurological condition?

A

multiple sclerosis

501
Q

which type of organism most commonly causes septic arthritis in prosthetic joints?

A

coagulase-negative staphylococcus spp.

502
Q

what would you expect to see on the ECG of someone having a lateral STEMI and which artery is most likely to be involved?

A

ST elevation in leads I, aVL, V5-6

503
Q

what are some of the red flag symptoms suggestive of cauda equina syndrome?

A
  • Saddle anaesthesia or paraesthesia. - Incontinence - Perianal/perineal sensory loss. - Unexpected laxity of the anal sphincter. - Severe or progressive neurological deficit in the lower extremities
504
Q

what time-frame do you have to give thrombolysis (alteplase) in someone present with symptoms of an ischaemic stroke?

A

only given if onset of symptoms is less than 4 and a half hours ago

505
Q

what type of seizures is carbamazepine used first-line for?

A

partial/focal seizures

506
Q

with what symptoms and signs would you expect cholecystitis to present with?

A
  • Pain frequently begins in the epigastric region and then localizes to RUQ - Pain initially may be colicky (biliary colic) and typically becomes more intense and longer-lasting and eventually becomes constant (as stone becomes lodged) - Nausea, vomiting, fever - Signs of peritoneal irritation may be present, and the pain may radiate to the right shoulder or scapula - Murphy’s sign: lay two fingers over the RUQ. Ask the patient to breathe in. This causes pain and arrest of inspiration as the inflamed GB impinges on your fingers. The sign is only positive if a similar manoeuvre in the left upper quadrant does not cause pain
507
Q

with what signs and symptoms would you expect someone with a subarachnoid haemorrhage to present with?

A

symptoms: sudden severe occipital headache (thunderclap), nausea/vomiting, collapse, seizures Signs: neck stiffness, positive Kernig’s sign, confusional state, intraocular haemorrhage (~15%), complete or partial palsy of the oculomotor nerve

508
Q

what is the most common and second most common valve disease?

A

most common: aortic stenosis second most common: mitral regurg

509
Q

what type of antidepressants should be avoided in Alzheimer’s disease?

A

tricyclic antidepressants (due to anticholinergic effects)

510
Q

briefly, what is atrial flutter?

A

Atrial flutter is a macro-reentrant atrial tachycardia with atrial rates usually above 250 bpm up to 320 bpm The atria beat regularly but at a faster rate than the ventricles

511
Q

what are the different doses of adrenaline, hydrocortisone and chlorphenamine that are used in the management of anaphylaxis (4 different doses depending on age)?

A

<6 months old: - adrenaline: 150 mcg - hydrocortisone: 25mg - chlorphenamine: 250mcg/kg 6 months - 6 years old: - adrenaline: 150mcg - hydrocortisone: 50mg - chlorphenamine: 2.5mg 6-12 years old: - adrenaline: 300mcg - hydrocortisone: 100mg - chlorphenamine: 5mg >12 years old: - adrenaline: 500mcg - hydrocortisone: 200mg - chlorphenamine: 10mg

512
Q

what is the most common subtype of MS?

A

Relapsing-remitting MS: symptoms come and go. Periods of good health or remission (where there is remyelination) are followed by sudden symptoms or relapses (80% of people at onset)

513
Q

what are some neurological features that are suggestive of MND?

A
  • Fasciculations - the absence of sensory signs/symptoms - the mixture of lower motor neuron and upper motor neuron signs - wasting of the small hand muscles/tibialis anterior is common
514
Q

in what type of seizures is sodium valproate used first line for?

A

generalised seizures (assuming that the patient is not a woman of child-bearing age)

515
Q

what are some features you may see in someone with congestive cardiac failure?

A
  • dyspnoea - cough: may be worse at night and associated with pink/frothy sputum - orthopnoea - paroxysmal nocturnal dyspnoea - wheeze (‘cardiac wheeze’) - weight loss (‘cardiac cachexia’): occurs in up to 15% of patients. Remember this may be hidden by weight gained secondary to oedema - bibasal crackles on examination - signs of right-sided heart failure: raised JVP, ankle oedema and hepatomegaly
516
Q

what is the definition of accelerated/malignant hypertension and, very very briefly, how should it be responded to/managed?

A

Accelerated (also known as malignant) hypertension: this is a syndrome characterised by severe hypertension (systolic >200 mm Hg, diastolic >130 mm Hg) accompanied by end-organ damage e.g. encephalopathy, dissection, pulmonary oedema, nephropathy, eclampsia, papilloedema and/or angiopathic haemolytic anaemia. Accelerated hypertension needs urgent (same-day) assessment and immediate treatment to reduce the BP within minutes to hours

517
Q

briefly, what is disseminated intravascular coagulation (DIC)?

A

DIC is an acquired syndrome characterised by activation of coagulation pathways, resulting in formation of intravascular thrombi and depletion of platelets and coagulation factors

518
Q

very briefly, how do you calculate ABPI?

A

divide highest pressure of the foot/ankle by the highest brachial pressure of either arm

519
Q

what would you expect the WBC count to be in a lumber puncture done in: 1. a patient with bacterial meningitis 2. a healthy patient 3. a patient with viral meningitis?

A
  1. >1000 mm^3 2. 0-5 3. 5-500 (mean 80)
520
Q

what is the particular concern with mobitz type 2 second degree heart block?

A

the concern with type 2 second degree heart block is that it can progress rapidly to complete heart block

521
Q

which intervertebral discs are most likely to prolapse?

A

L4-L5 and L5-S1 are the most common levels for disc prolapse

522
Q

what is the most common cause of mitral stenosis?

A

rheumatic fever (by far!)

523
Q

what drug class is memantine and in what condition is it used?

A

memantine is an N-methyl-D-aspartate (NMDA) antagonist recommended by NICE as a second-line option for managing patients with moderate Alzheimer’s disease where AChE inhibitors are not tolerated or are contra-indicated, or in the treatment of severe Alzheimer’s disease

524
Q

how should migraines be managed (general advice, acute management and prophylactic treatment)?

A

General: - avoidance of triggers (which may involve adopting a ‘migraine’ diet) - stress reduction Acute management: - Moderate symptoms: NSAIDs or aspirin +/- anti-emetic (e.g. metoclopramide or prochlorperazine) - Severe symptoms: triptan (e.g. sumatriptan) +/- NSAID +/- anti-emetic Prophylaxis (to be considered in patients experiencing 2 or more migraines a month): - Propranolol is recommended as first-line preventative treatment in patients with episodic or chronic migraine - Topiramate can be given if a beta-blocker is unsuitable, however in women of childbearing potential, advice should be given on the associated risks during pregnancy (teratogenic), the need to use highly effective contraception (as reduces efficacy of hormonal contraceptives) and to seek further information if pregnant or planning a pregnancy. - Amitriptyline can also be considered for patients with episodic or chronic migraine

525
Q

overdose/toxicity of which medication is associated with tinnitus?

A

aspirin (more accurately, tinnitus is seen in salicylate poisoning, the most common cause of which being aspirin)

526
Q

what condition is GCA/temporal arteritis associated with?

A

polymyalgia rheumatica

527
Q

what is the epidemiology for polymyalgia rheumatica?

A

It almost exclusively occurs in people over 50 and is more common in women

528
Q

what are some of the many possible signs and symptoms of MS?

A

Visual: - Optic neuritis, due to demyelination of the optic nerve, causing unilateral visual reduction or loss (which may be painful or painless), is a common presenting feature - Double vision - Nystagmus Sensory changes: - Non-specific tingling or burning sensations - Loss of sensation in the legs ascending to the trunk is caused when spinal nerves of the dorsal or lumbar segments are affected - Lhermitte’s sign: electric shock-like sensations extending down the cervical spine radiating to the limbs - Changes in taste and smell - Changes in balance and hearing - Facial weakness - Fatigue - Symptoms may worsen when body is overheated (e.g. hot bath, exercise etc). This is known as Uhthoff’s phenomenon

529
Q

if alcohol abuse is suspected in status epilepticus, what should be given?

A

Parenteral thiamine (e.g. pabrinex) should be considered if alcohol abuse is suspected

530
Q

how is iron overload managed?

A
  • Regular phlebotomy is first-line management - If not tolerated or contraindicated, chelation therapy can be considered (but is started by a haematologist)
531
Q

which organism is classically responsible for causing an atypical pneumonia in the owners of exotic birds?

A

gram-negative bacteria Chlamydia psittaci

532
Q

briefly, what is the difference between a ‘true’ aneurysm and a ‘false’ aneurysm?

A

True arterial aneurysms involve all three layers of the artery wall (intima, media and adventitia). False aneurysms represent a collection of blood, held around the vessel by a wall of connective tissue. The aneurysm does not involve the vessel wall.

533
Q

injury of which nerve causes a medially winged scapula?

A

long thoracic nerve

534
Q

what would you expect to see on the ECG of someone in typical atrial flutter?

A

negatively directed saw-tooth atrial deflections (f waves) in leads II, III, and aVF, and positive deflections in V1 (if deflections are reversed, this is called reverse typical atrial flutter) with atrial rates of 240 to 320 bpm are seen. 2:1 atrioventricular block is usually present in the typical form, resulting in the characteristic ventricular rate of 150 bpm.

535
Q

which is the single most common presenting feature of multiple sclerosis?

A

optic neuritis due to demyelination of the optic nerve, causing unilateral visual reduction or loss (which may be painful or painless)

536
Q

what is the definition of a fragility fracture?

A

Fragility fractures are fractures that result from mechanical forces that would not ordinarily result in fracture, known as low-level (or ‘low-energy’) trauma, quantified by The World Health Organization (WHO) as forces equivalent to a fall from a standing height or less

537
Q

what medication are can cause/contribute to postural hypotension?

A

antihypertensive medication, alpha-blockers, diuretics, tricyclic antidepressants

538
Q

what are some features of hypoglycaemia?

A
  • Shaking and trembling. - Sweating, pins and needles in the lips and tongue. - Hunger, palpitations. - Headache (occasionally), double vision, difficulty in concentrating. - Slurring of speech, confusion, change of behaviour - Drowsiness/reduced consciousness
539
Q

briefly, how can RA be managed?

A
  • lifestyle: smoking cessation, Mediterranean diet - physio and OT - analgesia: paracetamol, codeine, NSAIDS (+PPI) - corticosteroids: , calcium and vitamin D supplementation + regular DEXA if long-term or high-dose use - DMARDs (e.g. methotrexate, azathioprine): prescribed by specialist with regular monitoring - biological therapies (e.g. infliximab, rituximab): at least 2 DMARDs must have been tried before considering biologic - surgery may be considered if nothing else has been successful
540
Q

how should a suspected DVT be investigated?

A

Conduct 2-level DVT Wells’ score: - If score is 2 or more, refer for proximal leg vein ultrasound scan to be carried out within 4 hours > If unable to do within 4 hours, do d-dimer and start treatment dose heparin - If score is 1 or less, offer D-dimer testing. If this is positive, then refer for leg vein ultrasound

541
Q

what are some of the features of alcohol withdrawal (and what time scale do they appear in)?

A

Minor symptoms (6-12 hours after alcohol): - Insomnia, fatigue, tremors, anxiety, restlessness, nausea and vomiting, sweating, headache, palpitation, alcohol craving Alcoholic hallucinosis (including visual, auditory and tactile hallucinations) can appear 12-24 hours after alcohol has stopped Withdrawal seizures (generalised tonic-clonic seizures) can appear 24-48 hours after alcohol has stopped Alcohol withdrawal delirium or ‘delirium tremens’ can appear 48-72 hours after alcohol has stopped. Symptoms of delirium tremens include: - Tremor, fits, hallucinations, global confusion, severely uncoordinated, autonomic hyperactivity (raised heart rate and blood pressure) - The altered mental state differentiates delirium tremens from normal alcohol withdrawal - Can be fatal if untreated

542
Q

in which ethnic group is sickle cell disease most common?

A

African and Caribbean

543
Q

which leads on a 12-lead ECG represent an inferior view of the heart?

A

lead II, III and aVF

544
Q

what is preload?

A

Preload, also known as the left ventricular end-diastolic pressure (LVEDP), is the amount of ventricular stretch at the end of diastole

545
Q

what is the main risk for Plummer-Vinson syndrome?

A

iron deficiency anaemia

546
Q

what criteria must be met for a stroke to be classified as ‘lacunar syndrome’ (LACS)?

A

One of the following needs to be present for a diagnosis of LACS: - Pure sensory stroke - Pure motor stroke - Senori-motor stroke - Ataxic hemiparesis

547
Q

what are the 2 coronary revascularisation techniques currently used to treat coronary artery disease?

A
  • percutaneous coronary intervention (PCI) - coronary artery bypass graft (CABG) note: The choice of CABG or PCI depends on the distribution of the coronary artery disease, comorbidities and patient preference (complex/multivessel disease is usually CABG)
548
Q

which organism most commonly causes infective endocarditis following prosthetic valve surgery?

A

coagulase-negative Staphylococci such as Staphylococcus epidermidis commonly colonize indwelling lines and are the most cause of endocarditis in patients following prosthetic valve surgery, usually the result of perioperative contamination

549
Q

what skin condition is considered the skin manifestation of coeliac disease?

A

dermatitis herpetiformis

550
Q

what are the different functions of the trigeminal nerve?

A
  • Ophthalmic branch (V1): sensation above lower eyelid + corneal reflex - Maxillary branch (V2): sensation from lower eyelid to upper lip - Mandibular branch (V3): sensation below upper lip + muscles of mastication
551
Q

what kind of antidepressants can cause urinary retention?

A

tricyclic antidepressants e.g. amitriptyline

552
Q

what is the most common cause of TIA?

A

atherothromboemoblism from the carotid (usually at site of bifurcation) is the chief cause

553
Q

what are some of the features of cluster headaches?

A
  • Pain occurs roughly 1-2 times a day, lasting 15 mins – 2hours, for a 4-12 week period - Begin quickly and without warning - pain is very severe and is often described as a sharp, burning or piercing sensation on one side of the head. It’s often felt around the eye and temple - often accompanied by a red, watery eye with lid swelling
554
Q

how does sciatica usually present?

A
  • Shooting pain going down leg (usually one but may be both) from lower back - Onset is often sudden - May or may not have neurological deficit on affected side - When lying on back with leg raised, will often experience shooting pain below knee
555
Q

what does first degree heart block look like on an ECG?

A

Fixed prolonged PR interval (>0.2s/1 large box)

556
Q

what does the ‘hand of benediction’ look like and which nerve is involved?

A

compression at elbow or forearm can cause ‘hand of benediction’= inability to flex thumb, index and middle finger

557
Q

what is the difference between Parkinson’s disease and Parkinsonism?

A

Parkinson’s disease is used to describe the idiopathic syndrome of Parkinsonism i.e. Parkinsonism is a clinical syndrome characterised by tremor, bradykinesia and rigidity. When the exact cause/trigger for these symptoms is unknown (idiopathic), it is termed Parkinson’s disease. However there are other known causes of Parkinsonism, such as drug-induced Parkinsonism.

558
Q

what are some features of superficial thrombophlebitis?

A
  • Tenderness, warmth and erythema following a course of a vein - There may be a cord-like palpable mass - There may be swelling/oedema of surrounding tissue
559
Q

what does a positive Trendelenburg sign look like? (describe the test)

A

place hands on patients superior anterior iliac crests then ask patient to stand on one leg in a positive test, the pelvis drops on the side of the lifted leg, indicating a weakness/paralysis of the hip abducters (gluteus medius and minimus) on the opposite (standing) leg

560
Q

how should alcohol withdrawal be managed?

A
  • Benzodiazepine: chlordiazepoxide or diazepam for 5-7 days in reducing dosages - Thiamine - Oral lorazepam is first line medication for delirium tremens
561
Q

what drug class does allopurinol belong to?

A

xanthine oxidase inhibitor

562
Q

which cranial nerve does Bell’s palsy effect and is it an UMN or LMN lesion?

A

it is a facial nerve palsy (effecting muscles of facial expression) it is a LMN lesion, which can be demonstrated as the patient will be unable to lift their eyebrows on the effected side

563
Q

how much glucose do average healthy adults require?

A

50-100g/day

564
Q

what is the definition of oligoarthritis?

A

arthritis affecting between 2-4 joints during the first 6 months of the disease

565
Q

what is a common way of monitoring the effectiveness of diuretics (e.g. furosemide) in the context of heart failure?

A

regularly weigh patient

566
Q

what is pseudogout?

A

Pseudogout is an acute inflammation of joints caused by the deposition of calcium pyrophosphate (CPP) crystals in articular and periarticular tissue

567
Q

what are some of the potential causes of UMN and LMN lesions of the facial nerve?

A

LMN lesion: Bell’s palsy, polio, otitis media, parotid tumour, Lyme disease, Ramsay Hunt syndrome UMN lesion: tumour, stroke (remember: LMN lesions are not forehead sparing, UMN lesions are)

568
Q

you find a patient to be hyponatraemic. What investigation should be done to help determine aetiology and inform the management plan?

A

BP (hyponatraemia can be classified into hypovolaemic, euvolaemic or hypervolaemic)

569
Q

what is the definition of a subdural haematoma?

A

A subdural haematoma (SDH) is a collection of clotting blood that forms in the subdural space (the space between the dura mater and arachnoid mater).

570
Q

what ECG findings might you see in someone presenting with a tricyclic antidepressant overdose?

A
  • Sinus tachycardia - Wide QRS complex - PR and QT interval prolonged
571
Q

what must B12 combine with for it to be absorbed in the distal ileum?

A

intrinsic factor (produced in the stomach)

572
Q

what is the antidote for paracetmol? (i.e. what is used in paracetamol overdose)

A

acetylcysteine

573
Q

what are the two most common organisms that cause septic arthritis?

A
  • Staphylococcus aureus is the most common causative organism in adults - Neisseria gonorrhoea is common among young, sexually active individuals
574
Q

what is TRALI and what causes it?

A
  • Non-haemolytic transfusion reaction - TRALI is a form of acute respiratory distress due to donor plasma containing antibodies against the patient’s leukocytes - donors are most often multiparous women as they have become alloimmunised
575
Q

how should frontotemporal dementia be managed?

A
  • No specific medication to slow progress - Avoid acetylcholinesterase inhibitors (may worsen confusion) - SSRIs may be useful in managing behavioural symptoms. Antipsychotics may be used if SSRIs are ineffective however must be used with extreme caution (as those with FTD are at increased risk of extrapyramidal side effects) - psychological support
576
Q

what doses of adrenaline, hydrocortisone and chlorphenamine should be used in the management of anaphylaxis in a 14 year old?

A

anyone over 12 years old: - adrenaline: 500mcg - hydrocortisone: 200mg - chlorphenamine: 10mg

577
Q

what is favism? (kinda rogue question)

A

Favism describes the susceptibility to, and clinical presentation of, acute haemolytic crises as a consequence of eating broad beans in a subgroup of patients with glucose-6-phosphate dehydrogenase (G6PD) deficiency

578
Q

what does the 3-6-9 rule refer to?

A

refers to the fact that the small bowel should be <3cm, large bowel <6cm, ceacum <9cm can be used to help determine if there is bowel dilation

579
Q

what is BNP and in what condition is it used to help diagnose?

A

B-type natriuretic peptide: raised levels (BNP >100mg/litre or NT-proBNP level >400 ng/L) indicate myocardial damage and are supportive of a diagnosis of heart failure (but note that BNP can be raised in other conditions so is not specific)

580
Q

what is the most common cause of large bowel obstruction?

A

colorectal carcinoma

581
Q

when do symptoms of sickle cell disease usually start to present and why?

A

during 3-6 months of age as foetal haemoglobin levels fall

582
Q

which joint is most commonly affected in pseudogout?

A

knee

583
Q

how should a tricyclic antidepressant overdose be managed?

A
  • ABCDE - Activated charcoal within first 1-2 hours - Intravenous sodium bicarbonate as an antidote has been shown to be an effective treatment for resolving the metabolic acidosis and cardiovascular complications of TCA poisoning. Should be used if patient is hypotensive, has an arrythmia, or has a prolonged QRS complex (among other indications)
584
Q

what are the classical features of stable angina?

A
  • Constricting discomfort in the front of the chest, neck, shoulders, jaw or arms. - Precipitated by physical exertion. - Relieved by rest or GTN in about five minutes
585
Q

what are some predisposing and precipitating factors for delirium?

A

Predisposing factors: >65, pre-existing cognitive deficit (e.g. dementia, stroke), frailty or multimorbidity, polypharmacy, significant injury (e.g. fractured NOF), visual or hearing impairment Precipitating factors: infection (particularly UTI, sepsis, pneumonia), substance misuse or withdrawal (e.g. alcohol), constipation, hypoxia, hypo/hyperthermia, hepatic or renal impairment, environmental change, pain, post-operation, prescribed medications (e.g. benzodiazepines, analgesics such as morphine, steroids, anticholinergics etc),

586
Q

what is the most common type of primary brain tumour in adults?

A

glioblastomas (poor prognosis, ~1 year)

587
Q

how is haemolytic anaemia diagnosed?/what results help diagnose haemolytic anaemia?

A
  1. Is there increased RBC breakdown? - Normocytic or macrocytic anaemia - Increased serum unconjugated bilirubin - Increased urinary urobilinogen (no urinary conjugated bilirubin) - Increased serum lactic dehydrogenase (LDH) as it is released from RBCs 2. Is there increased RBC production? - Increased reticulocytosis - Increased red cell MCV (due to reticulocytosis, but may be other cause e.g. folate/B12 deficiency)
588
Q

what are some of the clinical features of TACO?

A

dyspnoea, orthopnoea, peripheral and pulmonary oedema, and rapid increase of blood pressure (plus raised JVP)

589
Q

how should hyponatraemia be managed?

A
  • Acute symptomatic hyponatremia is a medical emergency: hypertonic saline to gradually correct the hyponatraemia, with the goal of ensuring that the sodium level does not rise by more than 6 mmol/L in the first six hours. Rapid overcorrection leads to a risk of osmotic demyelination syndrome. - Non-severe hypovolaemic hyponatraemia: IV saline replacement - Normovolaemic hyponatraemia: fluid restriction (500-1000ml per day) - Hypervolaemic hyponatreamia: treat underlying cause (usually CHF, AKI or cirrhosis)
590
Q

when should continuous compressions be done in adult CPR?

A

if the airway is not secure do continuous chest compressions until a secure airway has been established, then do the 30:2 ratio

591
Q

why do you want to do a urine protein electrophoresis in people with suspected myeloma?

A

looks for the presence of Bence Jones’ protein (raised concentration of monoclonal IgA/IgG protein)

592
Q

how is complete heart block managed?

A

It is treated acutely with atropine and temporary pacemaker insertion if required. Atropine is given as 500mcg IV every 3-5 minutes, max dose 3g CHB is the most common ECG indication for permanent pacemaker insertion

593
Q

what is the difference between supraventricular tachycardia and ventricular tachycardia in terms of ECG findings?

A

both show tachycardia on the ECG in SVT, the QRS complex will be narrow (<0.12s) in VT, the QRS complex will be broad (>0.12s)

594
Q

what type of WBCs would you expect to see an increase in, in an LP of someone with bacterial meningitis?

A

significant increase in polymorphs: neutrophils, eosinophils, basophils

595
Q

which valvular disease is most commonly associated with syncope?

A

aortic stenosis

596
Q

how is sickle cell anaemia inherited?

A

autosomal recessive inheritance (just one copy of HbS is called sickle trait)

597
Q

what does the GET SMASHED acronym stand for and what is it’s use?

A

it is used to remember the different causes of pancreatitis G: Gallstones/biliary tract disease (~40%) E: Ethanol/alcohol (~35%) T: Trauma (~1.5%) S: Steroids M: Mumps (or other virus e.g. CMV, measles, rubella) A: Autoimmune S: Scorpion venom H: Hyperlipidaemia, hypothermia, hypercalaemia E: Endoscopic retrograde cholangiopancreatography (ERCP) (~5%) D: Drugs (e.g. thiazides, azathioprine)

598
Q

what signs/symptoms would you expect in someone with a hypoglossal nerve palsy?

A
  • An LMN lesion produces wasting of the ipsilateral side of the tongue, with fasciculation - Tongue deviates towards the side of the lesion - Weakness of tongue muscles on one side (noted when pushing tongue against resistance)
599
Q

what are some conditions associated with berry aneurysms (and therefore SAH)?

A

polycystic kidney disease, coarctation of the aorta, Ehlers-Danlos syndrome, neurofibromatosis type 1

600
Q

what would you expect the protein level to be in a lumbar puncture done in: 1. a patient with viral meningitis 2. a patient with bacterial meningitis 3. a healthy patient

A
  1. 30-150 mg/dL 2. 100-500 3. 15-50
601
Q

shoulder dystocia during delivery is associated by what nerve palsy?

A

Erb’s palsy (injury to upper trunk of brachial nerve plexus)

602
Q

someone is taking an ace inhibitor, a CCB, and a thiazide diuretic for their blood pressure, which is still uncontrolled. Which investigation specifically should be conducted to determine what the next appropriate management option is?

A

should do U&Es to determine potassium If <4.5, add spironolactone. If >4.5, increase dose of thiazide diuretic

603
Q

what are some of the features of a clonic seizure?

A

repeated jerking movements of arms and legs. Awareness is usually impaired. Lasts from seconds to 1-2 mins. Rare type of seizure that most commonly occurs in babies. (can be difficult to distinguish from myoclonic seizures but jerking is more regular and sustained in clonic seizure)

604
Q

with what features (signs+symptoms) would you expect someone with mitral regurg to present with?

A
  • Most people with mild-moderate MR are asymptomatic - Symptoms tend to be due to failure of the left ventricle and dyspnoea on exertion, reduced exercise tolerance and lower limb oedema - Murmur: The murmur heard on auscultation of the chest is typically a pansystolic murmur described as “blowing”. It is heard best at the apex and radiating into the axilla. S1 may be quiet as a result of incomplete closure of the valve. (remember, left-sided murmurs, including mitral regurg, can be heard louder on expiration)
605
Q

what are the 3 clinical syndromes/subtypes of frontotemporal dementia and, briefly, what are the key features of each?

A

There are three main clinical syndromes of FTD: - Behavioural variant frontotemporal dementia. Main features: changes in personality, behaviour, interpersonal and executive skills - Progressive non-fluent aphasia. Main features: loss of language skills - Semantic dementia. Main feature: Loss of semantic memory (e.g. unable to recognise familiar faces/objects)

606
Q

what is the investigation of choice to help confirm a suspected acute diverticulitis case?

A

CT scanning with intravenous, oral or rectal contrast: sensitivities and specificities for CT are significantly better than for contrast enemas. (note: endoscopy is usually avoided in initial assessment of acute diverticulitis due to the risk of perforation)

607
Q

what scoring system can be used in AF to help determine a patient’s risk of stroke?

A

CHA2DS2-VASc assessment tool

608
Q

what is the most important investigation (at least i think?? i dunno) for myasthenia gravis?

A

autoantibodies: around 85-90% of patients have antibodies to acetylcholine receptors. In the remaining patients, about about 40% are positive for anti-muscle-specific tyrosine kinase antibodies

609
Q

how should someone presenting with an opioid overdose be managed?

A
  • ABCDE approach - Give oral activated charcoal, provided the airway can be protected, if a substantial amount has been ingested within two hours. - IV/IM naloxone (0.4-2mg for adults)
610
Q

what is the definition of an extradural haematoma?

A

An extradural haematoma is a collection of blood between the skull and the dura

611
Q

what would you expect to see on the ECG of someone suffering a posterior MI?

A

tall R waves and ST depression in V1-V2

612
Q

how is Parkinson’s disease diagnosed?

A

diagnosis is clinical based off of symptoms (tremor, bradykinesia, rigidity) investigations are only really done to rule out alternative causes

613
Q

what is the most common cause of superior vena cava obstruction?

A

non-small cell ling cancer

614
Q

a ventriculoperitoneal shunt is usually used as long-term management in which condition?

A

hydrocephalus

615
Q

what special test can be done to help determine the site of valvular incompetence in a patient with varicose veins?

A

the trendelenburg test (make sure to know this is completely different from the trendelenburg sign seen in injury of superior gluteal nerve!!)

616
Q

what are some of the most common features of a focal seizure arising from the temporal lobe?

A
  • Preceded by aura - Hallucinations (auditory/gustatory/olfactory), - Automatisms (lip smacking/grabbing/plucking), - Emotional and behavioural changes - Memory disturbance (déjà vu, amnesia)
617
Q

what are some of the clinical features seen in TRALI?

A

nonproductive cough, breathlessness, hypoxia and frothy sputum. Fever and rigors may be present usually occurs within 6 hours of transfusion

618
Q

what is hydrocephalus?

A

Hydrocephalus is defined as a condition in which there is an excessive volume of cerebrospinal (CSF) fluid within the ventricular system of the brain and is caused by an imbalance between CSF production and absorption

619
Q

what is the definition of cardiomopathy?

A

Cardiomyopathy is defined as a ‘myocardial disorder in which heart muscle is structurally and functionally abnormal without coronary artery disease, hypertension, valvular or congenital heart diseases

620
Q

how should an allergic (non-anaphylactic) transfusion be managed?

A

slow transfusion (stop if at all concerned about anaphylaxis), possible IV antihistamine (e.g. chlorphenamine), possible IV corticosteroid (e.g. hydrocortisone), monitor closely to exclude anaphylaxis and inform senior

621
Q

how can the common peroneal nerve be injured and what does this classically cause?

A
  • fracture of the fibular head - compressing the common fibular nerve as it crosses around the neck of the fibula e.g. habitual leg crossing causes foot drop
622
Q

if the vagus nerve is damaged, which side should the uvula deviate towards?

A

deviates to the unaffected side

623
Q

what would you generally expect to see on an FBC of someone with acute myeloid leukaemia?

A

elevation in WBC, neutropenia, thrombocytopenia

624
Q

what is the target time for someone presenting with a STEMI to have primary PCI?

A

ideally should be done within 90 minutes of diagnosis

625
Q

what condition should you think of in an individual with alcohol-induced pain in raised lymph nodes?

A

Hodgkin’s lymphoma (not NHL) (only seen in less than 10% of people with HL)

626
Q

how would you expect a patient with encephalitis to present?

A

Hallmark triad of: fever, headache and altered mental state/psychiatric symptoms May also experience seizures, vomiting, focal features e.g. aphasia etc

627
Q

briefly, what is Erb’s palsy (not features) and how can it be caused?

A
  • Erb’s palsy= injury to the upper trunk of the brachial nerve plexus (C5-6) - can be caused by excessive lateral flexion of neck which may be due to trauma (e.g. falling on neck) or can occur during birth due to shoulder dystocia
628
Q

which patients with suspected aortic stenosis can undergo ECG exercise stress testing?

A

can consider ECG exercise stress test ONLY in asymptomatic patients

629
Q

what are the 2 most common causes of aortic stenosis?

A

-Degenerative calcification (most common cause in older patients > 65 years) - Bicuspid aortic valve (most common cause in younger patients < 65 years)

630
Q

in which condition would you most likely find Bence Jone’s protein on a urine protein electrophoresis?

A

myeloma (Bence Jones’ protein= raised concentration of monoclonal IgA/IgG proteins)

631
Q

what are the 6 Ps of an acutely ischaemic limb?

A
  • Pale - Pulseless - Painful - Paralysed - Paraesthetic (pins and needles) - Perishingly cold
632
Q

what investigation should generally be avoided in cases of suspected acute diverticulitis and why?

A

Because of risk of perforation, endoscopy is generally avoided in initial assessment of the patient with acute diverticulitis. Its use should be restricted to situations when the diagnosis in unclear, to exclude other possible diagnoses

633
Q

how should an anaphylactic transfusion reaction be managed?

A
  • Stop transfusion + call for help - 5L O2, 0.5mg adrenaline IM every 10-15 mins, fluid resuscitation - Consider inhaled bronchodilator (salbutamol neb), corticosteroid (200mg IV hydrocortisone), antihistamine (10mg IV chlorphenamine)
634
Q

what would you expect to see on an ABG of someone with salicylate poisoning (both earlier and later findings)?

A

salicylate overdose leads to a mixed respiratory alkalosis and metabolic acidosis. Early stimulation of the respiratory centre leads to a respiratory alkalosis whilst later the direct acid effects of salicylates (combined with acute renal failure) may lead to an acidosis.

635
Q

what are some of the possible causes of superior vena cava obstruction?

A
  • malignancies e.g. non-small cell lung cancer (most common cause), small cell lung cancer, lymphoma - aortic aneurysm - mediastinal fibrosis - goitre - SVC thrombosis
636
Q

what are the 3 clinical syndromes/subtypes of frontotemporal dementia?

A

There are three main clinical syndromes of FTD: - Behavioural variant frontotemporal dementia. - Progressive non-fluent aphasia. - Semantic dementia.

637
Q

which type of thalassaemia is more common?

A

alpha is more common than beta

638
Q

which cranial nerve controls the extraocular muscle ‘superior oblique’?

A

trochlear nerve (IV)

639
Q

which tool can be used to calculate the ten-year probability of a major osteoporotic fracture in people aged 40-90?

A

FRAX score

640
Q

what are the ‘centor criteria’ used for and what do the criteria consist of?

A

used to help determine whether a case of tonsillitis is due to a bacterial source or not the criteria are: - temp >38 - tender anterior cervical lymphadenopathy - exudate present on tonsils - absence of cough a bacterial infection is likely if at least 3/4 features are present

641
Q

what are some of the many signs which may be noted on examination of someone with infective endocarditis?

A
  • Heart murmur: most commonly of aortic regurgitation - Splinter haemorrhages - Osler’s nodes: small tender red-to-purple nodules on the pulp of the terminal phalanges of the fingers and toes - Janeway lesions: irregular painless erythematous macules on the thenar and hypothenar eminence (usually with acute IE and S. aureus) - Clubbing: only 10% of cases and usually in long-standing subacute IE - Roth’s spots: retinal haemorrhages with pale centres - splenomegaly
642
Q

what features would you expect to see in someone with Klumpke’s palsy?

A
  • total claw hand (persistent flexion of the interphalangeal joints and extension of the metacarpophalangeal joints in the hand) - Sensory loss in the C8 and T1 dermatomes (little finger and medial surface of the forearm and arm) - may have Horner’s syndrome
643
Q

what is the function of the trochlear nerve (cranial nerve IV)?

A

controls the extraocular muscle superior oblique (which is responsible for the down and out movement of the eye)

644
Q

how long should LMWH or fonadparinux be continued for in confirmed DVT?

A

the LMWH or fondaparinux should be continued for at least 5 days or until the international normalised ratio (INR) is 2.0 or above for at least 24 hours, whichever is longer

645
Q

what drugs can cause folate deficiency?

A

alcohol, anticonvulsants, methotrexate, sulfasalazine and trimethoprim but only if high-dose and a prolonged course

646
Q

describe what Osler’s nodes and Janeway lesions look like and in what condition are they seen?

A
  • Osler’s nodes: small tender red-to-purple nodules on the pulp of the terminal phalanges of the fingers and toes - Janeway lesions: irregular painless erythematous macules on the thenar and hypothenar eminence seen in infective endocarditis
647
Q

a patient comes in with a claw hand- hyperextension of the metacarpophalangeal joints and flexion at the distal and proximal interphalangeal joints of the 4th and 5th digits- what nerve is injured?

A

the ulnar (note: a total claw hand, with all fingers flexed, is seen in Klumpke’s palsy)

648
Q

how can you tell the difference between an UMN and LMN of the facial nerve?

A

LMN= patient is unable to raise eyebrows (forehead is paralysed) UMN= patient can raise eyebrows (forehead is spared)

649
Q

what are some of the red flag features suggestive of a spinal fracture?

A
  • Sudden onset of severe central pain in the spine which is relieved by lying down. - Major or minor trauma - Structural deformity of the spine (such as a step from one vertebra to an adjacent vertebra). - Point tenderness over the vertebral body.
650
Q

was is Brown-Séquard’s syndrome and what most commonly causes it?

A

rare neurological condition characterized by a lesion in the spinal cord which results in weakness or paralysis (hemiparaplegia) on one side of the body and a loss of sensation (hemianesthesia) on the opposite side it is usually caused penetrating trauma e.g. stab wound

651
Q

what is the most common cause of an extradural haematoma?

A

EDH is most often caused by trauma to the temple (which may cause a fractured temporal or parietal bone) which damages the middle meningeal artery or vein, resulting in blood collecting between the dura and the skull

652
Q

what are some risk factors for varicose veins?

A

Female sex, age, family history, obesity, standing job, pregnancy, previous VV, previous DVT, long bone fractures

653
Q

what are some of the signs and symptoms of ankylosing spondylitis?

A
  • inflammatory back pain (often improves with exercise, often experience stiffness, pain which wakes them in the night, starts in sacroiliac joint but may be felt as diffuse buttock pain, on examination there is often tenderness at sacroiliac joints, in advanced disease you may see exaggerated thoracic kyphosis) - peripheral enthesitis (seen in 1/3rd of patients) - peripheral arthritis (seen in 1/3rd of patients) - systemic features are common. Fever and weight loss may occur during periods of active disease. Fatigue is also prominent - extra-articular manifestations (e.g. anterior uveitis, aortitis, pulmonary fibrosis)
654
Q

which organisms most commonly cause viral meningitis?

A

enteroviruses, HSV (usually type 2), VZV

655
Q

how should hypoglcaemia be managed?

A

If conscious: - Initially glucose 10–20 g is given by mouth either in liquid form or as granulated sugar or sugar lumps. If necessary, this may be repeated after 10–15 minutes. - Or can give buccal glucose gel If unconscious/reduced consciousness: - Sc or IM glucagon - Or 20% glucose IV Long-term: review hypoglycaemic medications

656
Q

what is the epidemiology like for ankylosing spondylitis?

A

peak onset is between 20-30 years of age and is 3 times more common in men

657
Q

injury of which nerve causes wrist drop?

A

radial nerve

658
Q

when are troponin levels most commonly taken/repeated?

A

at 6 and 12 hours post onset of symptoms (levels usually peak at 24/48 hours and then begin to fall from roughly 5 days)

659
Q

what is the best investigation for varicose veins?

A

duplex ultrasound

660
Q

very briefly, how should a patient be managed if they are found to have acute pancreatitis secondary to gallstones?

A

If gallstones are thought to be the only cause, then do cholecystectomy. If gallstones and cholangitis are suspected, then do endoscopic retrograde cholangiopancreatography (ERCP)

661
Q

what 4 main symptoms are associated with GCA?

A
  • Severe temporal (or occipital) headache (>85% of cases) - Scalp tenderness (may be pronounced so that lying head on pillow is painful) - Jaw claudication (>50%) - Visual disturbances
662
Q

what are some of the features of an absence seizure?

A

brief (<10s), characterised by staring, loss of expression, unresponsiveness and stopping mid-activity. Will usually recover immediately with no memory of the seizure, and continue their activity. Presents in childhood and often grown out of by puberty

663
Q

what is the most common contraindication for sodium valproate?

A

it is contraindicated in women of child-bearing age (as it highly teratogenic)

664
Q

when should a V/Q scan be offered instead of a CTPA in suspected PE?

A

patients who have an allergy to contrast media, or who have renal impairment, or whose risk from irradiation is high

665
Q

damage/compression of which nerve causes the ‘hand of benediction’?

A

compression of median nerve (but at elbow or forearm, rather than carpal tunnel)

666
Q

how should a TIA be managed, both immediately and long-term?

A
  • Immediate management in suspected TIA: 300mg aspirin OD until confirmed + urgent assessment within 24 hours - Long-term: 75mg clopidogrel OD, control risk factors (hypertension, hypercholesterolaemia etc)
667
Q

what is koilonychia a sign of and what does it look like?

A

iron deficiency anaemia spoon-shaped nails with longitudinal ridging

668
Q

what is the role of zolindronic acid in the management of myeloma?

A

Zoledronic acid is given to prevent and manage osteoporosis and fragility fractures as these are a large cause of morbidity and mortality, particularly in the elderly.

669
Q

what are some causes of type 2 respiratory failure?

A
  • Pulmonary disease: COPD, severe asthma, pulmonary oedema - Reduced respiratory drive: sedative drugs (e.g. opioids), CNS tumour/trauma - Neuromuscular disease: cervical cord lesion, myasthenia gravis, Guillain-Barre syndrome
670
Q

how should tension-type-headaches be managed?

A
  • General: deal with any underlying triggers e.g. stress - Medication: > Ibuprofen is first line medication (paracetamol if can’t take NSAIDs) > In chronic TTH, medications should be avoided due to risk of medication-induced headaches > Amitriptyline is commonly used for prophylaxis (although not NICE recommended)
671
Q

what are some of the possible signs of haemolytic anaemia?

A
  • Signs of anaemia: general pallor and pale conjunctivae. Tachycardia, tachypnoea and hypotension if severe. - Mild jaundice may occur due to haemolysis. - Splenomegaly: occurs with some causes - eg, hereditary spherocytosis. It may indicate an underlying condition such as CLL, lymphoma or SLE. - Leg ulcers may occur in some causes of haemolytic anaemia - eg, sickle cell anaemia
672
Q

‘saddle-shaped’ ST elevation is associated with what condition?

A

pericarditis

673
Q

what condition is Buerger’s test used for?

A

peripheral arterial disease (specifically in more severe disease or when you are concerned about possible critical limb ischaemia)

674
Q

how do bisphosphonates work?

A

act by inhibiting the action of osteoclasts, the cells responsible for bone resorption

675
Q

what 3 features, if present, indicate an intravascular rather than extravascular cause for a haemolytic anaemia?

A
  • Increased plasma haemoglobin - Methaemoglobinaemia (produced when haem binds with albumin) - Haemoglobinuria
676
Q

‘saw-tooth’ p waves are seen on the ECG of patients in what arrythmia?

A

atrial flutter

677
Q

which artery is most commonly involved in extradural haematomas?

A

middle meningeal artery

678
Q

what is the definition of first degree heart block?

A

PR interval >0.2s

679
Q

what does Cushing’s triad/reflex consist of and why does it occur?

A

bradycardia, hypertension and deep/irregular breathing occurs when intracranial pressure is at a critically high level and often indicates imminent brain herniation

680
Q

what are some of the risk factors for lymphomas?

A

Both: Ebstein-Barr virus, immunocompromised (including HIV). NHL: - Chromosomal translocations - Environmental factors: pesticides, herbicides, solvents, hair dye, chemotherapy and radiation exposure etc - Autoimmune disorders e.g. Hashimotos’s thyroiditis

681
Q

what are some causes of type 1 respiratory failure?

A

COPD, pneumonia, asthma, pulmonary oedema, pneumothorax, PE, bronchiectasis, acute respiratory distress syndrome

682
Q

how would you expect an aortic dissection to present?

A
  • Sudden severe chest pain (may be described as ‘ripping’ pain): > location of pain is associated with location of dissection: anterior pain suggests ascending aorta; back pain suggests descending - hypertension - aortic regurgitation (murmur) - syncope - neurological symptoms (e.g. acute paraplegia, upper or lower limb ischaemic neuropathy)
683
Q

what criteria can you use to diagnose infective endocarditis?

A

Modified Duke’s criteria

684
Q

what are the 2 main methods of rhythm control in AF and what is the timescale for when they can be offered?

A

methods: electrical cardioversion or pharmacological cardioversion (with amiodarone) if onset of AF is <48h ago, then either method can be used. If onset of AF is >48h ago, then electrical cardioversion if preferred and should be done after the patient has been anticoagulated for at least 3 weeks

685
Q

what medication should always be avoided in individuals with migraine with aura?

A

combined oral contraceptive pill

686
Q

what is the ‘DAS28’ tool, which condition is it used in, and why?

A
  • used in rheumatoid arthritis - is a measure of disease activity in RA - helps monitor progress of RA
687
Q

what are some of the features of pericarditis?

A
  • Sudden, sharp/stabbing, constant chest pain which can also be pleuritic. Classically it is relieved by sitting forward and made worse by lying down - Pericardial friction rub can be heard in roughly 1/3rd of patients. It is described as high-pitched or squeaky and is heard best at the left sternal edge with the patient leaning forward at end-expiration - Fever, myalgia
688
Q

in which condition would you expect to see a ‘rain-drop skull’ on x-ray?

A

multiple myeloma

689
Q

what are the 2 most common causes of megaloblastic, macrocytic anaemia?

A

B12 and folate deficiency

690
Q

what are gouty tophi?

A

Gouty tophi are nodular masses of monosodium urate crystals deposited in the soft tissues of the body. They are a late complication of hyperuricaemia and develop in more than half of patients with untreated gout

691
Q

what 2 scores does a DEXA provide and what do they represent?

A
  • T score and Z score T-score is a comparison of a person’s bone density with that of a healthy 30-year-old of the same sex. The Z-score is a comparison of a person’s bone density with that of an average person of the same age and sex
692
Q

briefly, what is multiple sclerosis?

A

Multiple sclerosis is chronic cell-mediated autoimmune disorder characterised by demyelination in the central nervous system

693
Q

what causes appendicitis?

A

Likely the end result of a primary obstruction of the lumen of the appendix. Causative agents include: - Calcified faecal deposits known as faecoliths (most common cause) - Tumour - Bezoars (mass of indigestible material) - Trauma - Intestinal worms

694
Q

how should someone with stable angina be managed?

A

General: - Lifestyle changes: diet, exercise, smoking etc - Help patients recognise precipitating factors e.g. exercise, and encourage them to take GTN before these activities Pharmacological: - Prescribe sublingual glyceryl trinitrate (GTN) for the rapid relief of symptoms of angina. Advice to people: - Offer either a beta blocker or a calcium channel blocker as first-line treatment for stable angina. Decide which drug to use based on comorbidities, contraindications and the person’s preference - Consider aspirin 75 mg daily for people with stable angina, taking into account the risk of bleeding and comorbidities - Offer statin Coronary revascularisation (CABG or PCI) may be required in those at high risk and those who have failure to be controlled by medical therapy

695
Q

if a patient has superficial thrombophlebitis, what is an important investigation to consider?

A

diagnosis of superficial thrombophlebitis is superficial, but a doppler should be done to rule out concomitant deep vein thrombosis (DVT)

696
Q

what are some of the features (signs and symptoms) of DKA?

A

Polyuria, polydipsia, vomiting, dehydration, weight loss, weakness, lethargy, abdominal pain, altered mental state (if severe), Kussmaul respiration (deep hyperventilation), slow cap refill, tachycardia, hypotension

697
Q

what conditions does acute coronary syndrome consist of?

A
  • STEMI - NSTEMI -unstable angina
698
Q

injury to which nerve causes foot drop?

A

common peroneal nerve

699
Q

what scoring system is used to determine the most appropriate initial investigation in suspected DVT?

A

2-level DVT Wells’ score

700
Q

what are the names of the bony swellings/deformities classically seen in the hands of individuals with OA?

A

Bouchard’s (PIP) and Heberden’s (DIP) nodes

701
Q

what is the correct technique for recording lying-standing BP?

A

proper technique is to have patient lie down for at least 5 mins and measure BP. Ask patient to stand and measure BP again after 1 minute and 3 minutes of standing

702
Q

which conditions may cause a false ABPI reading and why?

A

diabetes (most commonly) and chronic renal failure as these conditions can cause calcified vessels, resulting in a falsely elevated reading (suspect in anyone with ABPI >1.2)

703
Q

what are some of the investigations for suspected lymhoma?

A
  • Bloods film: Reed-Sternberg cells (multinucleated giant cells) in Hodgkin’s -Bloods: FBC, ESR (>70 - poor prognosis), lactate dehydrogenase (LDH), HIV, LFTs, U&Es - CXR (mediastinal expansion) - CT scan of thorax and abdomen (staging) - Lymph node (preferably excisional) and bone marrow biopsies
704
Q

A patient presents with wasting of the left side of his tongue. His tongue is also deviating towards the left. Which cranial nerve is affected and on which side is the lesion?

A

lesion of the left hypoglossal nerve (tongue deviates towards side of lesion and their is wasting on the ipsilateral side)

705
Q

what is the most common cause of cardiogenic shock?

A

MI (particularly anterior)

706
Q

what is a ‘normal’ diameter for the abdominal aorta and what is a AAA defined as?

A

The ‘normal’ diameter of the abdominal aorta is approximately 2 cm; it increases with age. An abdominal aneurysm is usually defined as an aortic diameter of 3 cm or greater

707
Q

what are some of the migraine prophylaxis options?

A

Propranolol is recommended as first-line preventative treatment in patients with episodic or chronic migraine Topiramate can be given if a beta-blocker is unsuitable, however in women of childbearing potential, advice should be given on the associated risks during pregnancy (teratogenic), the need to use highly effective contraception (as reduces efficacy of hormonal contraceptives) and to seek further information if pregnant or planning a pregnancy. Amitriptyline can also be considered for patients with episodic or chronic migraine Note: preventative treatment should be tried for at least 3 months at the maximum tolerated dose, before deciding whether or not it is effective

708
Q

a 48 year old caucasian man is diagnosed with stage 1 hypertension. It improves only slightly with lifestyle changes. What is the drug should he be offered first?

A

ACE inhibitor e.g. ramipril (offered to those <55 and who are not of afro-caribbean origin)

709
Q

what are some of the ways that heart failure can be classified?

A
  • acute or chronic - left, right or congestive - systolic or diastolic - low-output or high-output - reduced or preserved ejection fraction d
710
Q

what would you expect in the results of a lumbar puncture in a patient with bacterial meningitis?

A

appearance: Cloudy/turbid opening pressure: 200-500 (elevated) WBC count: >1000 protein: 100-500 glucose: <40 (<40% of serum level)

711
Q

what is the gold standard investigation for gout?

A

microscopy of synovial fluid aspirate from joint or tophi showing urate crystals

712
Q

how is myasthenia gravis managed?

A
  • long-acting anticholinesterase inhibitors e.g. pyridostigmine - immunosuppression: prednisolone initially - thymectomy
713
Q

what are generalised seizures and what are the 6 different types?

A

Primary generalised seizures: simultaneous onset of electrical discharge throughout cortex (both hemispheres) types: - abscence seizures (‘petit mal’) - tonic-clonic seizures (‘grand mal’) - myoclonic seizures - clonic seizures - tonic seizures - atonic seizures

714
Q

how is polymyalgia rheumatica managed?

A
  • glucocorticosteroids are the only known effective treatment. Non-steroidal anti-inflammatory agents are of little value for the management of this disease - Treatment is usually with prednisolone, starting at a higher dose (15mg) and gradually reducing to a maintenance dose of 1mg - must review in a few weeks and then follow up at 3-monthly features
715
Q

what would you expect to see in a simple partial seizure?

A

awareness is unimpaired, usually <1min, with focal motor (e.g. jerking limb), sensory (e.g. numbness, tingling), autonomic (e.g. nausea) or psychic (e.g. hallucinations) symptoms. No post-ictal symptoms

716
Q

which joint is most commonly affected by septic arthritis?

A

knee (prosthetic joints are also at greater risk)

717
Q

what is the definition of thrombocytopenia?

A

Thrombocytopenia means a reduction in the platelet count below the normal lower limit, which is usually defined as 150 x 109/L

718
Q

if someone has both B12 and folate deficiency, which should be treated first?

A

it is essential to start treating the B12 deficiency before starting folate or the latter may aggravate the B12 deficiency and precipitate subacute combined degeneration of the cord

719
Q

how would you expect a sixth nerve (abducens) palsy/lesion to present?

A

Inability to look laterally. The eye is deviated medially because of unopposed action of the medial rectus muscle

720
Q

what are some of the possible causes of ischaemic stroke?

A

thrombosis in situ, atherothromboembolism (e.g. from carotid arteries), cardiac emboli (due to AF, endocarditis)

721
Q

what are the diagnostic criteria for DKA?

A
  • Ketonaemia (3 mmol/L and over), or significant ketonuria (more than 2+ on standard urine sticks). - Blood glucose over 11 mmol/L or known diabetes mellitus (the degree of hyperglycaemia is not a reliable indicator of DKA and the blood glucose may rarely be normal or only slightly elevated in DKA). - Bicarbonate below 15 mmol/L and/or venous pH less than 7.3
722
Q

which antibody is more associated with providing long-term immunity?

A

IgG (presence of these antibodies indicate the patient has immunity)

723
Q

what would you expect to see in a complex partial seizure?

A

awareness is impaired. Vary greatly depending on where they start and spread (most commonly arise in temporal lobe). People may have unusual or repetitive behaviours e.g. chewing, fidgeting etc. Usually 30s-3mins. Post-ictal confusion common

724
Q

how should TACO be managed?

A

stop transfusion, 15L O2, sit upright, consider furosemide 40mg IV

725
Q

what are some of the clinical features of hyperosmolar hyperglycaemic state?

A
  • General: fatigue, lethargy, nausea and vomiting - Neurological: altered level of consciousness, headaches, papilloedema, weakness, seizures may be present in up to 25% of cases - Haematological: hyperviscosity (may result in myocardial infarctions, stroke and peripheral arterial thrombosis) - Cardiovascular: dehydration (dry mouth, decreased skin turgor), hypotension, tachycardia
726
Q

what are some of the possible clinical features of macrocytic anaemia?

A

Possible symptoms: Shortness of breath on exertion, fatigue, palpitations, exacerbation of angina, complaining of looking pale Possible signs: Pallor (look at the nail beds and tongue), a bounding pulse, systolic pulmonary flow murmur

727
Q

what would you see in the ECG of a patient in complete heart block?

A

ECG: will show dissociation between P waves and ORS complexes and bradycardia

728
Q

what is pancytopenia?

A

Pancytopenia is a reduction in the number of red blood cells, white blood cells, and platelets in the peripheral blood below the lower limits of the age-adjusted normal range for healthy people. It is therefore the combination of anaemia, leukopenia, and thrombocytopenia

729
Q

very briefly, how is thalassaemia managed?

A

with repeat blood transfusion (which can cause iron overload)

730
Q

what are the pharmacological management steps in hypertension? (i.e. what do you start with first, then what is added etc)

A

Step 1: - <55 years old: offer an ACEi (rampiril) or an ARB (losartan) if not tolerated (do renal function and U&Es 1-2 weeks after starting ACEi, as can cause renal impairment and hyperkalaemia) - >55 years old or afro-caribbean origin of any age: offer a CCB (amlodipine) Step 2: - Add the alternative drug above Step 3: - Add thiazide-diuretic (e.g. indapamide or bendroflumethiazide) Step 4: - If BP still >140/80 at this stage, considered to be treatment resistant hypertension - Do U&Es to determine potassium. If <4.5, add spironolactone. If >4.5, increase dose of thiazide diuretic. If these options are not tolerated, consider adding a beta-blocker (or obtain specialist advice)

731
Q

how is coeliac disease managed?

A

All products containing wheat, rye and barely are removed from diet (closely involve dietician). Lifelong dietary changes are the only known effective treatment

732
Q

what are the 2 different types of respiratory failure?

A
  • Type 1 respiratory failure- also known as hypoxaemic respiratory failure- is characterised by an arterial oxygen tension (PaO2) of <8 kPa (60 mm Hg) with normal or low arterial carbon dioxide tension (PaCO2). - Type 2 respiratory failure- also known as hypercapnic respiratory failure- is the presence of a PaCO2 >6 kPa (45 mm Hg) and PaO2 <8 kPa
733
Q

what may precipitate and episode of status epilepticus?

A

Potential precipitants include drug withdrawal, intercurrent illness, metabolic disturbance (e.g. hypoglycaemia), cerebrovascular event and alcohol intoxication or withdrawal

734
Q

how should a massive PE (i.e. haemodynamically unstable patient) be managed?

A

start on unfractionated heparin then thrombolyse as soon as possible

735
Q

how can h.pylori be tested for?

A

carbon-13 urea breath test, stool antigen test, blood antibody test (less common)

736
Q

describe the sensitivity and specificity of a d-dimer test for PE

A

D-dimer has high sensitivity but low specificity, so excludes PE if normal but doesn’t confirm if positive

737
Q

what does ERCP stand for and what is it used for?

A

Endoscopic retrograde cholangiopancreatography (ERCP) is used to diagnose common bile duct stones and can be used for stone extraction

738
Q

what is the leading cause of sudden cardiac death in young athletes?

A

hypertrophic obstructive cardiomyopathy

739
Q

how does haemoglobinuria present and what condition is it associated with?

A

presents as dark urine associated haemolytic anaemia (intravascular causes)

740
Q

what are some of the contraindications for performing a lumbar puncture?

A
  • Reduced consciousness (GCS<9 or a drop of 3 points or more) - Signs of raised ICP (coma, high BP + bradycardia, papilloedema) - Focal neurology - Bleeding tendency/coagulation abnormalities - Local superficial infection at lumbar puncture site - Extensive or spreading purpura
741
Q

what would you expect to see on the ECG of someone in heart failure?

A

May show evidence of ventricular hypertrophy (e.g. left/right axis deviation and increased QRS amplitude) ECGs are almost always abnormal in some way in patients with heart failure (in fact, a normal ECG should make you doubt a HF diagnosis!). Abnormalities include arrythmias (particularly AF), ST segment and T wave changes

742
Q

how long should symptoms have been present for a confident clinical diagnosis of dementia to be made?

A

symptoms and impairments should have been evident for at least 6 months for a confident clinical diagnosis of dementia to be made

743
Q

what tool can be used to help determine severity in pancreatitis?

A

Glasgow Prognostic Score

744
Q

which antibody is the first to appear in response to an antigen?

A

IgM (so if IgM positive and IgG negative, it suggests a recent infection)

745
Q

what are some possible features of hyperkalaemia?

A

Symptoms are nonspecific and include weakness, fatigue, nausea SOB, palpitations or chest pains Physical examination is unlikely to suggest the diagnosis but may show bradycardia (heart block), tachypnoea, muscle weakness, depressed or absent reflexes

746
Q

what is angular cheilitis a sign of and what does it look like?

A

iron defiency anaemia ulcers at corner of mouth

747
Q

briefly describe the epidemiology of multiple sclerosis

A

MS is more common in women (3x more common than men) and in Caucasians. Incidence increases as you move further from the equator and is most commonly diagnosed between the ages of 20-40. Of those with a first-degree relative with MS, 2% will go on to develop the condition

748
Q

how should confirmed AAAs be monitored?

A

Offer surveillance with aortic ultrasound to people with an asymptomatic AAA: - every year if the AAA is 3.0–4.4 cm. - every 3 months if the AAA is 4.5–5.4 cm (surgery is advised if >5.5cm)

749
Q

with what signs and symptoms would you expect cholangitis to present with?

A
  • Charcot’s triad: fever, jaundice, RUQ pain - Rigors - Altered mental state
750
Q

describe the classical Parkinsonian tremor

A
  • most marked at rest, 3-5 Hz - worse when stressed or tired, improves with voluntary movement (e.g. picking up mug) - typically ‘pill-rolling’, i.e. in the thumb and index finger - usually begins on one side of body and progresses to other side
751
Q

what are some of the common features of migraine?

A

Typically the headaches last between 4 and 72 hours and have at least two of the following features: - Unilateral. - Pulsating. - Moderate or severe intensity of pain. - Aggravated by, or resulting in the avoidance of, routine physical activity. In addition, there is at least one of: - Nausea and vomiting during migraine attacks. These are common symptoms that affect at least 60% of patients suffering from migraines. (note that GI disturbances are very common among children affected by migraine) - Photophobia and phonophobia, which are also very common

752
Q

peptic ulcers are particularly associated with which syndrome?

A

Zollinger-Ellison syndrome

753
Q

what type of WBCs would you expect to see an increase in, in an LP of someone with viral meningitis?

A

moderate increase in lymphocytes

754
Q

hypotension, muffled heart sounds and jugular venous distention are the classical features of which condition?

A

cardiac tamponade

755
Q

when is surgery usually offered in patients with AAAs?

A

usually considered/offered when AAA is 5.5cm in size or larger

756
Q

what criteria must be met for a stroke to be classified as ‘posterior circulation syndrome’ (POCS)?

A

one of the following need to be present for a diagnosis of POCS: - Cranial nerve palsy and a contralateral motor/sensory deficit - Bilateral motor/sensory deficit -Conjugate eye movement disorder (e.g. horizontal gaze palsy) - Cerebellar dysfunction (e.g. vertigo, nystagmus, ataxia) Isolated homonymous hemianopia

757
Q

what medication class is the first-line treatment in chronic myeloid leukaemia?

A

tyrosine kinase inhibitors (e.g. imatinib)

758
Q

what is a possible complication of abrupt withdrawal of anti-Parkinsonian medications?

A

neuroleptic malignant syndrome

759
Q

what are the 2 different classification systems for aortic dissection and what are the different classifications within these systems?

A

The Stanford Classification in common use classifies the dissections into type A and type B: - Type A involves ascending aorta (DeBakey types I and II). - Type B does not involve the ascending aorta (DeBakey type III). The DeBakey Classification classifies the dissections into: - Type I: aorta, aortic arch, and descending aorta. - Type II: ascending aorta only. - Type III: descending aorta distal to left subclavian.

760
Q

what are some causes of subarachnoid haemorrhage?

A
  • The most common cause of SAH is head injury and this is called traumatic SAH. In the absence of trauma, SAH is termed spontaneous SAH (below refers to spontaneous SAH) - Approximately 85% of spontaneous SAH are caused by ruptured berry aneurysm within the circle of Willis - Approximately 15% are caused by arteriovenous malformations - Other causes: encephalitis, tumour, vasculitis
761
Q

a ‘petit mal’ refers to which kind of seizure?

A

absence seizures

762
Q

what are some of the most common causes of chronic heart failure?

A

Common causes of chronic heart failure include: - Coronary artery disease - Hypertension - Valvular disease - Myocarditis.

763
Q

why is it particularly important that ventricular tachycardia is managed quickly?

A

It has the potential to precipitate ventricular fibrillation

764
Q

what hereditary condition is associated with bilateral vestibular schwannomas (acoustic neuromas)?

A

neurofibromatosis type 2

765
Q

what are the 2 different types of second degree heart block and what do they look like on an ECG?

A

Type 1 (Mobitz I, Wenckebach): progressive prolongation of the PR interval until a dropped beat occurs (P wave present but no QRS as atrial impulse fails to be conducted to ventricles) Type 2 (Mobitz II): PR interval is constant but the P wave is often not followed by a QRS complex. There may be a regular pattern to the number of atrial impulses that do actually lead to ventricular activation e.g. every second or third atrial impulse may ‘get through’, termed 2:1 and 3:1 block respectively

766
Q

how is torsades de pointes (a polymorphic VT) usually managed?

A

Intravenous magnesium is the drug of choice for torsades de pointes

767
Q

how are peptic ulcers managed?

A
  • General: Stop NSAIDs, lifestyle changes (e.g. smoking cessation) - If H.pylori positive, H.pylori eradication therapy: > A PPI (e.g. omeprazole) and > Clarithromycin and > Amoxicillin or metronidazole - If H.pylori negative, give PPI until ulcer is healed
768
Q

what are the management options for varicose veins?

A

Conservative management: rest, elevation, lifestyle changes e.g. weight loss (although minimal evidence) In symptomatic VVs, endovenous or open surgery is considered: - Endovenous options include radiofrequency ablation (also called endothermal ablation), laser therapy or foam sclerotherapy (note that endovenous options are preferred over surgical) - Surgical options include avulsions (small incisions are made over each varicosity and that part of the vein is excised using a vein hook or forceps) or stripping (a wire, plastic or metal rod is passed through the lumen of the saphenous vein and pulled until the entire vein is stripped out of the leg)

769
Q

how should GCA be managed?

A
  • Treat immediately with high-dose cortico-steroids: > 40mg prednisolone daily (to be continued for 1-2 years) + urgent ophthalmologist review > If visual symptoms present: 60mg prednisolone one-off dose + seen by ophthalmologist same day - Start aspirin 75mg daily - Start PPI (for gastroprotection) and consider osteoporosis prophylaxis (with bisphosphonates e.g. alendronic acid) due to long term steroid use
770
Q

what might you see on an ECG of someone with a PE?

A

may be normal or show various anomalies, including the classical S1Q3T3 pattern (deep S wave in 1, large Q waves in 3 and inverted T waves in 3), RBBB, AF etc. Sinus tachy is the most common finding A large PE can show ECG features of acute cardiac ischaemia (e.g. ST depression)

771
Q

what is a reciprocal change on an ECG and what does it help diagnose?

A

Reciprocal change is defined as ST-segment depression occurring on an ECG which also has ST-segment elevation in at least 2 leads in a single anatomic segment it supports a diagnosis of STEMI

772
Q

what can cause folate deficiency?

A
  • Dietary deficiency. - Malabsorption. - Increased demands including haemolysis, leukaemia and rapid cell turnover as may occur in some skin diseases. - Increased urinary excretion occurs in heart failure, acute hepatitis and dialysis. - Drug-induced deficiency includes alcohol, anticonvulsants, methotrexate, sulfasalazine and trimethoprim but only if high-dose and a prolonged course
773
Q

explain the theory behind the concern regarding giving COPD patients high-concentration supplemental oxygen

A

in normal circumstances, respiratory rate is generally controlled by the partial pressure of carbon dioxide. However COPD patients can develop chronically elevated arterial CO2 levels, and as a result, their hypercapnic drive becomes blunted, and their hypoxic drive begins to control their respiration. Therefore if they are given high-concentration oxygen, this would cause a drop in their hypoxic drive and thus reduce their repiratory rate

774
Q

what is a common side effect of taking an ACE inhibitor and what should be done if the patient develops the side effect?

A

cough can switch to an ARB (e.g. losartan) if becomes troublesome

775
Q

briefly, how should diverticulitis be managed?

A
  • Admit to hospital if suffering from complications, pain not adequately managed by paracetamol, is dehydrated or at risk of being etc - Analgesia (avoid NSAIDs and opiates which may cause constipation if possible) - Oral or IV abx (depending on severity) - 15-30% with acute diverticulitis will require surgery for complications
776
Q

very briefly, how is acute mesenteric ischaemia managed?

A

initial resuscitation then urgent surgery is usually required.

777
Q

what are some of the features of a myoclonic seizure?

A

brief but intense jerk of limb, face or trunk. Consciousness not impaired but may feel confused/drowsy if several occur over short period. May be mistaken for clumsiness

778
Q

what are some of the signs/symptoms of tricyclic antidepressnant overdose?

A
  • Milder symptoms: tachycardia, drowsiness, dry mouth, nausea and vomiting, urinary retention, confusion, agitation, headache, dilated pupils - More severe: hypotension, cardiac rhythm disturbances, hallucinations, seizures, convulsions, coma
779
Q

with what symptoms would you expect someone in second degree heart block to present with?

A
  • some patients will be asymptomatic, others may experience light-headedness, dizziness, shortness of breath and syncope - Type II block may cause symptoms in the form of Stokes-Adams attacks, where episodes of syncope are caused by significant slowing of the ventricular rate
780
Q

briefly how is an aortic dissection managed?

A
  • Analgesia - Transfer to ICU or HDU - Manage hypertension (usually using beta-blockers, aim for 100-120mmHg) - Surgical intervention may involve the placing of stents or grafts to the aorta but accurate assessment is essential first, as there may be entry, re-entry and multiple tears. Type A dissections are surgically managed, uncomplicated type B dissections are usually medically managed
781
Q

when should someone be referred for a proximal leg vein ultrasound scan to be carried out within 4 hours in suspected DVT?

A

if they have a DVT Well’s score of 2 or more

782
Q

how should someone diagnose with acute heart failure be managed?/

A
  • Manage any underlying cause e.g. arrythmia, ischaemia etc - IV diuretic (furosemide): monitor renal function, urine output and weight to assess effectiveness - Consider NIV if severe dyspnoea + acidaemia - Once stabilised: Start ACE-I (ramipril)/ARB (valsartan) + β-blocker (bisoprolol) - Discharge with specialist follow-up in 2 weeks
783
Q

what are the 2 most important investigations in pericarditis and why?

A

ECG: - The changes in pericarditis are often global/widespread, as opposed to the ‘territories’ seen in ischaemic events - ‘saddle-shaped’ ST elevation - PR depression: most specific ECG marker for pericarditis Echocardiography: necessary to see if there is a pericardial effusion (which may in turn cause tamponade)

784
Q

briefly, how should septic shock be managed?

A
  • Begin BUFALO ASAP: blood cultures (prior to starting abx), monitor urine output hourly, fluid resuscitation, begin broad spectrum antibiotics, monitor lactate, start on 15L oxygen via non-rebreathe - In septic shock, you may use a vasopressor to manage the hypotension (e.g. noradrenaline, vasopressin)
785
Q

with what signs/symptoms would you expect someone with carpal tunnel syndrome to present?

A
  • numbness/tingling/pain in median nerve distribution - weakness in hand grip/opposition of thumb - wasting of thenar eminence - positive Phalen’s/Tinel’s test (Positive Phalen test: flexing the wrist for 60 seconds causes pain or paraesthesia in the median nerve distribution. Positive Tinel’s sign: tapping lightly over the median nerve at the wrist causes a distal paraesthesia in the median nerve distribution)
786
Q

what is the most appropriate investigation in a patient with suspected AAA?

A

abdo ultrasound

787
Q

what is the first line medication in mild-moderate Alzheimer’s dementia? (including examples)

A

Acetylcholinesterase (AChE) inhibitors e.g. donepezil, galantamine, rivastigmine

788
Q

what does it mean if a patient is diagnosed with ‘sero-negative’ rheumatoid arthritis?

A

that they have tested negative for the 2 most commonly associated antibodies: rheumatoid factor and anti-CCP

789
Q

verapamil is used as a prophylactic drug in which neurological condition?

A

cluster headaches

790
Q

what symptoms might someone in first degree heart block have?

A

largely asymptomatic and found incidentally (or rarely can progress to other forms of heart block) (or may have symptoms of underlying cause e.g. myocarditis)

791
Q

what symptoms may someone in SVT experience?

A

Symptoms may include: dizziness, palpitations, chest discomfort, fatigue

792
Q

what are the classical ECG findings in someone with pericarditis?

A
  • The changes in pericarditis are often global/widespread, as opposed to the ‘territories’ seen in ischaemic events - ‘saddle-shaped’ ST elevation - PR depression: most specific ECG marker for pericarditis
793
Q

what happens to preload and afterload in hypovolaemic shock?

A

preload: decreases afterload (systemic vascular resistance): increases

794
Q

what is Todd’s paresis?

A

a transient focal neurological deficit after an epileptic seizure. It usually presents as a localised to one side of the body but may also affect speech and vision. It is an important differential for stroke!

795
Q

what investigations should you consider in someone with suspected salicylate poisoning?

A
  • Plasma salicylate concentrations (although concentration is a poor indicator of severity, so should use clinical picture to guide management) - Bloods: U&Es, FBC, renal function, glucose, coag screen, LFTs - Toxicology screen (urine + bloods) - ECG - ABG: salicylate overdose leads to a mixed respiratory alkalosis and metabolic acidosis. Early stimulation of the respiratory centre leads to a respiratory alkalosis whilst later the direct acid effects of salicylates (combined with acute renal failure) may lead to an acidosis.
796
Q

what is the investigation of choice in a suspected subdural haematoma?

A

non-contrast CT head

797
Q

how should DKA be managed?

A
  • ABCDE - IV fluids: > if <90mmHg systolic, give 500ml bolus 0.9% NaCl > if >90mmHg systolic, give 1L NaCl over 1 hour (give then next two 1L infusions over 2 hours, and the fourth 1L infusion over 4 hours) > if potassium is 3.5-5.5, can add up to 40mmol potassium per 1L of NaCl, but can only be given starting from the SECOND bag of fluid (i.e. over 2 hours, so rate is max 20mmol/hr). If potassium <3.5, patient requires immediate senior review. - IV insulin infusion: soluble insulin should be diluted (and mixed thoroughly) with sodium chloride 0.9% intravenous infusion to a concentration of 1 unit/mL; infuse at a fixed rate of 0.1 units/kg/hour. - Monitor blood-ketone and blood-glucose concentrations hourly and adjust the insulin infusion rate accordingly - Once blood-glucose concentration falls below 14 mmol/litre, glucose 10% should be given by intravenous infusion - Continue insulin infusion until blood-ketone concentration is below 0.3 mmol/litre, blood pH is above 7.3 and the patient is able to eat and drink; ideally give subcutaneous fast-acting insulin and a meal, and stop the insulin infusion 1 hour later - Consider catheter if not passed urine by 1 hour and consider NG tube if vomiting or drowsy
798
Q

is there a screening programme for AAAs?

A

yerp

799
Q

what are some of the clinical features of a patient with a parcetamol overdose?

A
  • Often asymptomatic for first 24 hours, or non-specific abdominal symptoms (e.g. nausea/vomiting) - Hepatic necrosis begins to develop after 24 hours (elevated transaminases, right upper quadrant pain and jaundice) and can progress to acute liver failure. - Patients may also develop: encephalopathy, oliguria, hypoglycaemia, renal failure (usually occurs around day three), lactic acidosis.
800
Q

what are some risk factors for supraventricular tachycardia?

A

Risk factors include: previous MI, congenital heart disease, rheumatic heart disease, pericarditis, pneumonia, current alcohol intoxication, digoxin toxicity

801
Q

the Stanford classification system is used in which condition? and what does ‘type A’ and ‘type B’ refer to?

A

used to classify aortic dissections - Type A involves ascending aorta - Type B does not involve the ascending aorta (note: DeBakey is another classification system)

802
Q

roughly how should cholecystitis be managed?

A
  • Fluids, analgesia (NSAIDs) - Abx: (e.g. ciprofloxacin) oral or IV depending on severity - Laparoscopic cholecystectomy (or cholecystostomy, where a stoma is created in the gallbladder, followed by elective cholecystectomy in severe/emergency cases)
803
Q

how should a febrile non-haemolytic transfusion reaction be managed?

A

Slow transfusion down + give paracetamol 1g stop transfusion if symptoms continue + gain senior help

804
Q

which condition are associated with smudge cells (also known as smear cells) seen on a blood film?

A

chronic lymphocytic leukaemia

805
Q

if a haemorrhagic stroke has been ruled out, how should you initially treat a patient with an ischaemic stroke?

A

Thrombolysis with alteplase if onset of symptoms <4.5 hours ago 300mg aspirin (anti-platelet)

806
Q

what is an aortic dissection?

A

Aortic dissection is defined as disruption of the medial layer of the wall of the aorta provoked by intramural bleeding, resulting in separation of the aortic wall layers and subsequent formation of a true lumen and a false lumen with or without communication.

807
Q

how should second degree heart block be managed?

A

Patients with second-degree AV block should be referred for cardiological assessment (after adjusting the dose of or discontinuing any culprit medications), where more detailed investigations such as 24-hour ECG monitoring, electrophysiological studies, cardiac imaging and cardiac catheterisation may be carried out. Acutely symptomatic patients with low ventricular rates can be treated with atropine and/or temporary pacemaker insertion. Treatment by insertion of a permanent cardiac pacemaker may be required, particularly for Mobitz type II AV block

808
Q

what are the 3 different types of sickle cell crisis?

A
  1. Vaso-occlusion crises: this is the most common type of crisis in which obstruction of the microcirculation is caused by sickled red blood cells, causing ischaemia. This occlusion can cause mild to severe pain. Larger vessels may also be involved (which may cause complications such as stroke) 2. Aplastic crises: temporary cessation of erythropoiesis, causing severe anaemia. Usually precipitated by infection with parvovirus B19 (slapped cheek syndrome). Often required transfusion 3. Sequestration crises: sudden splenic or hepatic enlargement, abdominal pain and circulatory collapse from accumulation of sickled cells in spleen
809
Q

what investigations might you consider in someone with status epilepticus?

A
  • Toxicology screen - VBG/U&Es: Severe electrolyte imbalance, hypoglycaemia, and hyperglycaemia are treatable and reversible causes of seizures and SE - FBC: abnormal FBC results may shed light on the cause of SE - ECG: Arrhythmias or cardiac ischaemia may be the result of prolonged SE - Consider EEG: may be helpful when evaluating for non-convulsive SE or monitoring response to treatment - Obs
810
Q

what is enthesitis and which 2 rheumatological conditions is it most commonly associated with?

A

inflammation of the entheses, the sites where tendons or ligaments insert into the bone most commonly associated with ankylosing spondylisis and psoriatic arthritis

811
Q

what are some causes of aortic regurgitation? (note: there are 2 categories into which causes can be grouped)

A

Valvular abnormalities: - Rheumatic fever (most common cause in developing countries) - Bicuspid aortic valve (most common congenital cause + one of the most common causes in developed countries) - Infective endocarditis - connective tissue diseases e.g. RA/SLE aortic root dilation: - aortic dissection - spondylarthropathies (e.g. ankylosing spondylitis) - hypertension - aortitis secondary to syphilis - Marfan’s, Ehler-Danlos syndrome

812
Q

what are the 3 different subtypes/classifications of multiple sclerosis and how do they differ?

A
  • Relapsing-remitting MS: symptoms come and go. Periods of good health or remission (where there is remyelination) are followed by sudden symptoms or relapses (80% of people at onset) - Secondary progressive MS: follows on from relapsing-remitting MS. There are gradually more or worsening symptoms with fewer remissions (about 50% of those with relapsing-remitting MS develop secondary progressive MS during the first ten years of their illness) - Primary progressive MS: from the beginning, symptoms gradually develop and worsen over time (10-15% of people at onset)
813
Q

at what age does coeliac disease usually present?

A

any!

814
Q

what is the function of the accessory nerve?

A

motor function to sternocleidomastoid and trapezius

815
Q

what investigation should you consider in someone with suspected subarachnoid haemorrhage but in whom a CT head is normal? and when should this investifation be done?

A

lumbar puncture to detect xanthochromia (diagnostic) should be done at least 12 hours after onset of symptoms

816
Q

what signs and symptoms would you expect a patient with diverticulitis to present with?

A

Symptoms: - Generally LLQ pain, which may be constant or intermittent and may be associated with change in bowel movements - Fever and tachycardia are usually present (hypotension and shock are uncommon) - Anorexia, nausea and vomiting may occur On examination: - Localised tenderness and occasionally a palpable mass - Often reduced bowel sounds - Rectal examination may reveal tenderness or a mass

817
Q

an individual on allopurinol develops gout, what should be done?

A

treat acute attack (NSAIDs/colchicine) but DON’T stop allopurinol issue with allopurinol is that it should not be started during an acute attack but can be continued through an acute attack

818
Q

what is afterload and what is it dependant on?

A

Afterload is the pressure the heart must work against to eject blood during systole, and so is determined by systemic vascular resistance

819
Q

what are some risk factors for fragility fractures?

A
  • osteoporosis - Other conditions affecting bone strength, such as acromegaly or osteogenesis imperfecta. - advancing age - Female - Caucasian - corticosteroid therapy - Cushing’s syndrome - alcohol - smoking
820
Q

how should an adult (or child >12) in anaphylaxis be managed?

A
  • ABCDE approach - Airway: secure airway (call anaesthetist if necessary), 15L oxygen via non-rebreathe - Obtain IV access - Raise legs to help restore circulation - IM adrenaline: > Given in the anterolateral aspect of the middle third of the thigh > Adult and child (12-17) dose: 500 micrograms (see table for other doses) > Should be repeated after 5 minutes if there is no clinical improvement - Fluid bolus (500ml Hartmanns or 0.9% saline) - After initial resuscitation: > Chlorphenamine 10 mg intravenously > Hydrocortisone 200 mg intravenously
821
Q

what are some of the risk factors for diverticular disease?

A

The main risk factors are age over 50 years and low dietary fibre. Obesity is an important risk factor in young people

822
Q

how would you expect a lesion of the ulnar nerve to present?

A
  • ‘claw hand’- hyperextension of the metacarpophalangeal joints and flexion at the distal and proximal interphalangeal joints of the 4th and 5th digits - loss of sensation over the palmar aspect of the little finger and half of the ring finger - wasting of hypothenar eminence
823
Q

how does osteoporosis usually present?

A

asymptomatic before presenting with a fragility fracture

824
Q

what is the definition of orthostatic/postural hypotension?

A

Orthostatic hypotension is defined as a fall in systolic blood pressure of at least 20 mmHg (at least 30 mmHg in patients with hypertension) and/or a fall in diastolic blood pressure of at least 10 mmHg within 3 minutes of standing

825
Q

what is the definition of essential hypertension?

A

Essential hypertension is defined as blood pressure (BP) ≥140/90 mmHg, with no secondary cause identified It is the most common type of hypertension

826
Q

a patient presents to the GP with one eyelid drooping, and the eye itself is dilated and looking in a down and out direction. What is the cranial nerve affected?

A

third nerve (oculomotor) palsy

827
Q

what are some of the many possible features of Parkinson’s disease?

A

Bradykinesia: - poverty of movement also seen, sometimes referred to as hypokinesia - short, shuffling steps with reduced arm swinging (reduction in arm swinging usually seen before change in walking) - difficulty in initiating movement (e.g. difficulty rising from sitting- this is usually a later finding) - progressive reduction in the amplitude of repetitive movements e.g. asking the patient to repeatedly oppose middle finger and thumb - May also develop micrographia, where handwriting gets smaller (decreases in amplitude) as the individual writes Tremor: - most marked at rest, 3-5 Hz - worse when stressed or tired, improves with voluntary movement (e.g. picking up mug) - typically ‘pill-rolling’, i.e. in the thumb and index finger - usually begins on one side of body and progresses to other side Rigidity: - lead pipe - cogwheel: due to superimposed tremor - Postural instabliity (usually seen in mid-to-late stage disease) - In later stages, gait is characterised by small shuffling steps with unsteadiness on turning (taking several steps to turn) and difficulty in stopping (‘festination’) - Reduction in facial expressions (often noted by partners) - Psychiatric conditions: depression (most common), anxiety, psychosis, dementia - REM sleep disturbances (individuals may not lose tone of muscles during REM sleep, causing them to act out dreams) - Constipation

828
Q

what is most commonly the first manifestation of ankylosing spondylitis seen on a pelvic x-ray?

A

sacroilitis

829
Q

what are some of the classical features of heart failure on a chest x-ray?

A

A - alveolar oedema (bat wing opacities) B - Kerley B lines (These are thin lines 1-2 cm in length in the periphery of the lung(s). They represent thickened subpleural interlobular septa and are usually seen at the lung bases) C – cardiomegaly (cardiothoracic ratio >50%) D - dilated upper lobe vessels E - pleural effusion

830
Q

what would you expect to see on the ECG of someone having an anteroseptal STEMI and which artery is most likely to be involved?

A

ST elevation in leads V1-V4 left anterior descending artery is the most likely occluded vessel

831
Q

what are some of the possible extra-articular features of rheumatoid arthritis? (note: there are many!)

A
  • Eyes: secondary Sjögren’s syndrome, scleritis and episcleritis. - Skin: leg ulcers especially in Felty’s syndrome - Rheumatoid nodules: these are common, and may occur in the eyes, may be subcutaneous, and may be in the lung, heart and occasionally the vocal cords. - Neurological: peripheral nerve entrapment - Respiratory system: pleural involvement, pulmonary fibrosis - Cardiovascular system: pericardial involvement, valvulitis and myocardial fibrosis, immune complex vasculitis. - Liver: mild hepatomegaly and abnormal transaminases are common. - Other: thyroid disorders, osteoporosis, depression, splenomegaly.
832
Q

which 2 genetic conditions are particular risk factors for aortic dissection?

A

Marfan’s syndrome Ehlers Danlos syndrome

833
Q

what is the difference between: diverticulosis, diverticular disease and diverticulitis?

A

Diverticulosis= the presence of diverticula which are asymptomatic. Diverticular disease= diverticula associated with symptoms. Diverticulitis= evidence of diverticular inflammation (fever, tachycardia) with or without localised symptoms and signs.

834
Q

what condition is defined as a deficiency of factor VIII?

A

haemophilia type A

835
Q

when is surgery usually offered in patients with AAAs and what are the two options?

A

Offered when AAA is 5.5 cm or bigger in size Open repair: involves exposure of the abdominal aorta, aortic and iliac clamping and replacement of the aneurysmal segment with a prosthetic graft Endovascular repair (EVAR): involves introducing a stent-graft system through the femoral arteries, which relines the aneurysm, diverts blood flow through the endograft and allows the aneurysm to thrombose (endovascular is preferred method due to reduced risk of complications/ faster recovery but requires patients to have lifelong imaging to make sure there are no issues with the stent graft)

836
Q

you come across an unconscious patient. on assessment, the patient is unresponsive but you can hear occasional irregular gasps, with the interval between each gasp gradually increasing, how should you proceed?

A

continue with rest of assessment (i.e. check for pulse) and begin BLS this is an example of agonal breathing, which is common in the early stages of cardiac arrest and should not be mistaken for a sign of life (it is not true breathing, but a brainstem reflex)

837
Q

briefly, where are vessels usually harvested from when doing a coronary artery bypass graft?

A

Veins may be harvested from the saphenous vein in the legs but arteries are also used and these give better long-term outcomes, particularly the internal mammary artery.

838
Q

what does the epidemiology of myeloma look like?

A

median age at presentation= 70, more common in Afro-Caribbean’s than Caucasians and more common in men

839
Q

what is the general management of someone with haemolytic anaemia?

A
  1. Administer folic acid because active haemolysis may cause folate deficiency. 2. Discontinue medications that may have precipitated or aggravated haemolysis (penicillins, quinine, L-dopa etc) + general supportive measures
840
Q

what is the the best indicator of severity of liver failure in the context of paracetamol overdose?

A

prothrombin time

841
Q

what proportion of stokes are ischaemic?

A

roughly 85%

842
Q

what are some causes of right-sided heart failure?

A
  • primary causes: right-sided valvular disease (e.g. pulmonary stenosis, severe tricuspid regurg), right ventricular infarction - secondary causes: left-sided heart failure, cor pulmonale (where pulmonary disease, such as pulmonary hypertension, COPD, or interstitial lung disease, causes failure of the right ventricle)
843
Q

describe the kind of gait associated with later stages of Parkinson’s disease?

A

gait is characterised by small shuffling steps with unsteadiness on turning (taking several steps to turn) and difficulty in stopping (‘festination’). There is also reduced arm swinging

844
Q

what might you expect to see on an ECG of a patient with hyperkalaemia?

A
  • Tall tented T waves - can be difficult to determine (>5mm in limb leads, >10mm in chest leads) - Widening of the QRS. - Flattened, or loss of, P wave. (note that ECG may be normal even in severe hyperkalaemia so while ECG changes warrants urgent treatment, lack of ECG changes does not mean patient should not be treated)
845
Q

what are some ‘special tests’ which are recommended by NICE to be performed to help assess falls risk in the elderly?

A
  • Timed Up and Go Test: request that the patient rise from a chair without the support of their arms, walk three metres, turn round and sit down again. A walking aid can be used if required. Completion of the test without unsteadiness or difficulty suggests a low risk of falling. - Turn 180° Test: request that the patient stand up and step around until they are facing the opposite direction. If more than four steps are required to do this, further assessment is indicated
846
Q

what does the murmur of aortic regurgitation sound like and how is it best listened for?

A

early diastolic murmur, best heard in the aortic area, with the patient sitting forward and in expiration

847
Q

what must be given with levodopa and why when treating Parkinson’s disease?

A

It is given with a peripheral dopa-decarboxylase inhibitor (benserazide or carbidopa), which prevents peripheral conversion of levodopa to dopamine, which helps to reduce side-effects including nausea and vomiting, and also increases the amount of levodopa that reaches the CNS (dopa-decarboxylase inhibitors do not cross the blood-brain barrier, unlike levodopa) Benserazide + levodopa= co-beneldopa (brand name: Madopar) Carbidopa + levodopa= co-careldopa (brand name: Sinemet)

848
Q

what are some risk factors for AAA?

A
  • Family history - Tobacco smoking - Male sex. - Increasing age. - Hypertension. - Chronic obstructive pulmonary disease. - Hyperlipidaemia. - For a small number there may be a specific cause e.g. trauma, infection Note: diabetes is protective
849
Q

what is the pathophysiology behind AF?

A

The normal regular electrical impulses generated by the SA node in the right atrium of the hear are overwhelmed by disorganised electrical impulses usually originating in the roots of the pulmonary veins. This leads to irregular conduction of ventricular impulses that generate the heartbeat

850
Q

what are the 3 main management methods for osteoporosis?

A
  • Lifestyle advice: smoking cessation, alcohol moderation, dietary advice, exercise - Supplements: calcium and vitamin D supplementation - Medication: bisphosphonates (e.g. alendronic acid)
851
Q

what are some medications which are associated with falls in the elderly?

A

Medications associated with postural hypotension: nitrates, diuretics, anticholinergic medications, antidepressants, L-DOPA, beta-blockers, ACEi Medications associated with falls due to other mechanisms: benzodiazepines, antipsychotics, opioids, anticonvulsants, codeine, digoxin

852
Q

a 47 year old black man is diagnosed with hypertension, which is not controlled with lifestyle measures. What medication should he be offered first-line?

A

calcium channel blocker e.g. amlodipine (>55 years old or afro-caribbean origin of any age: offer a CCB (amlodipine) )

853
Q

what is very important to bear in mind when managing someone with both B12 and folate deficiency and why?

A

it is essential to start treating the B12 deficiency before starting folate or the latter may aggravate the B12 deficiency and precipitate subacute combined degeneration of the cord

854
Q

what are some of the clinical features of an anaphylactic transfusion reaction?

A

tachycardia, reduced BP, bronchospasm/acute dyspnoea, cyanosis, oedema

855
Q

what safety-netting advice should be given to someone who has been diagnosed with polymyalgia rheumatica?/

A

give advice on GCA symptoms to look out for (headache, visual changes, jaw claudication, scalp tenderness) as 15% will develop GCA during treatment for PMR

856
Q

which blood test in particular should you consider doing in individuals with suspected heart failure?

A

B-type natriuretic peptide: raised levels (BNP >100mg/litre or NT-proBNP level >400 ng/L) indicate myocardial damage and are supportive of a diagnosis of heart failure

857
Q

what investigations should you consider if you are suspecting acute pancreatitis in a patient?

A
  • Serum amylase 3x or more than the normal is traditional way of diagnosing (although lipase levels are more sensitive and specific) - FBC, U&Es, glucose, CRP etc indicate prognosis - Abdominal and erect chest x-ray - Ultrasound (to detect gallstones) - ABG (to detect hypoxaemia)
858
Q

which is the only DMARD which does not require regular blood tests?

A

hydroxychloroquine (all the others require blood tests, such as FBC and LFTs due to risk of myelosuppression and liver cirrhosis)

859
Q

in what situations might it be useful to measure serum tryptase?

A

in suspected anaphylaxis cases (or if there is any ambiguity regarding the diagnosis) serum tryptase indicates degranulation of mast cells and so is seen in anaphylaxis.

860
Q

in basic terms, what is myeloma?

A

malignant proliferation of plasma cells

861
Q

how should vascular dementia be managed?

A
  • There are no specific pharmacological treatments approved for the treatment of vascular dementia - Acetylcholinesterase inhibitors and NMDA antagonists are only recommended if there is suspected comorbid Alzheimer’s dementia - Medication may be given to help with risk factors e.g. antihypertensive medication - psychological support
862
Q

which 2 coagulation tests would you expect to be prolonged in DIC?

A

prothrombin time (PT) Activated partial thromboplastin time (APTT)

863
Q

how should pulseless ventricular tachycardia be managed?

A

immediate defibrillation

864
Q

what is Rovsing’s sign and which condition is it classically seen in?

A

Pressure in LIF increases pain in RIF Classically seen in appendicitis

865
Q

what would you expect to see on an FBC of someone with DIC?

A

thromboctytopenia

866
Q

what is Lhermitte’s sign and in which condition is it seen?

A

Lhermitte’s sign: electric shock-like sensations extending down the cervical spine radiating to the limbs Seen in MS

867
Q

what are some potential risk factors for ventricular tachycardia?

A
  • VT is often a symptom of coronary heart disease or structural heart disease. - VT can be triggered by electrolyte deficiencies e.g. hypokalaemia, hypocalcaemia, hypomagnesaemia. - Use of sympathomimetic agents (e.g. caffeine or cocaine) may stimulate VT in vulnerable hearts
868
Q

in addition to pharmacological treatment and help with lifestyle changes, what else should be offered to individuals diagnosed with heart failure?

A
  • Annual flu vaccine - One-off pneumococcal vaccine
869
Q

how should folate deficiency be managed (including dietary advice)?

A
  • Dietary advice: nuts, green vegetables, brown rice and liver - Folic acid 5mg OD for 4 months (higher dose in malabsorption states)
870
Q

which 2 troponins are used predominantly when investigating for possible cardiac ischaemia?

A

troponin I and T

871
Q

what are DMARDs and what are some examples?

A
  • stand for: Disease-modifying antirheumatic drugs - a group of drugs which alter the outcome or course of inflammatory conditions - Examples: methotrexate, azathioprine, hydroxychloroquine, sulfasalazine
872
Q

which 2 investigations are most important when suspecting infective endocarditis?

A
  • Blood cultures: It is generally recommended that 3 sets of blood cultures be obtained at least 1 hour apart prior to initiating antibiotic therapy to ensure greatest yield - Echocardiography: must be performed as soon as possible (ideally within 24 hours) in all patients with suspected IE
873
Q

what are some potential complications of pancreatitis?

A

Pancreatic complications: necrosis, abscess formation, pancreatic ascites, chronic pancreatitis Other organ involvement: AKI, intestinal obstruction, sepsis, acute respiratory distress syndrome, pulmonary oedema, shock, disseminated intravascular coagulopathy

874
Q

what are some possible causes of first degree heart block?

A

may be normal in athletes (increased vagal tone) myocarditis acute MI (particularly inferior) Lyme disease SLE electrolyte disturbances medications (e.g. calcium channel blockers, beta-blockers etc)

875
Q

what are some of the acquired causes of haemolytic anaemia? (there are many)

A
  1. Immune: - Isoimmune: haemolytic disease of newborn, blood transfusion reaction. 2. Autoimmune: - Warm antibody type: idiopathic, systemic lupus erythematosus (SLE), lymphoma, chronic lymphatic leukaemia (CLL) - Cold antibody type: cold haemagglutinin disease, paroxysmal cold haemoglobinuria, Mycoplasma pneumoniae, lymphoma, infectious mononucleosis or other viral infections - Drug-related: drug absorbed on to red cell surface (eg, penicillins, cephalosporins) or immune complex mediated (eg, sulfonamides, sulfasalazine). 3. Non-immune: trauma (cardiac haemolysis, microangiopathic anaemia (found in patients with disseminated intravascular coagulation or haemolytic uraemic syndrome and thrombotic thrombocytopenic purpura), infection (malaria, sepsis), hypersplenism, membrane disorders, paroxysmal nocturnal haemoglobinuria, liver disease.
876
Q

what information is vital to give female patients being started on topiramate?

A

in women of childbearing potential, advice should be given on the associated risks during pregnancy (teratogenic), the need to use highly effective contraception (as reduces efficacy of hormonal contraceptives) and to seek further information if pregnant or planning a pregnancy.

877
Q

you suspect a patient of having Raynaud’s. What factors/features can help you determine whether it is more likely to be primary or secondary?

A

Primary: usually affects younger people (<30s), more commonly women. Symptoms are usually bilateral and there isn’t evidence of an underlying condition. They usually have a normal ESR and negative antineutrophil antibodies Secondary: less common than primary. Affects older individuals who may have symptoms of an underlying condition (e.g. skin tightness). They usually have a raised ESR and positive antineutrophil antibodies.

878
Q

how should patients presenting with a staggered paracetamol overdose be managed?

A

given acetylcysteine straight away (i.e. you can’t plot the paracetamol level on the chart to determine whether acetylcysteine should be given, so you justs give it)

879
Q

how should paracetamol overdose be managed?

A
  • ABCDE - All patients who have a timed plasma paracetamol level plotted on or above the line drawn between 100 mg/L at 4 hours and 15 mg/L at 15 hours after ingestion, should receive acetylcysteine. Infusion should be given in 5% glucose solution. If given within 8 hours of ingestion, acetylcysteine is almost 100% effective in preventing liver damage - If there is any doubt about the timing of the ingestion (including a staggered overdose over one hour or more), acetylcysteine should be given without delay
880
Q

what investigations should be done if you believe a patient may have coeliac disease? (3)

A
  • Specific antibodies: Total immunoglobulin A (IgA) and IgA tissue transglutaminase (tTG) as the first choice. Endomysial antibodies (EMA) if IgA-tTG is unavailable - FBC: 50% of patients will have iron or folate deficiency anaemia - Biopsy is needed to confirm diagnosis
881
Q

what would you expect to see in the blood film (not FBC) of someone with B12 or folate deficiency?

A

hypersegmented neutrophils + oval macrocytes

882
Q

what classification system is commonly used to classify ischaemic strokes?

A

Oxford/Bamford Classification System

883
Q

who should be screened for coeliac disease? (4)

A

Should screen individuals at greater risk, such as those with: - Type 1 diabetes (at the time of diagnosis). - Autoimmune thyroid disease (at the time of diagnosis). - Irritable bowel syndrome (IBS) - First-degree relatives with coeliac disease

884
Q

where might pain radiate to in cholecystitis?

A

the shoulder tip (due to peritoneal irritation)

885
Q

briefly, how should sciatica be managed?

A
  • Analgesia (e.g. NSAID or opioid if necessary) - Encourage to keep active (don’t rest for long periods of time) - Usually gets better in 4-6 weeks
886
Q

briefly, how are cluster headaches managed (both acute attacks + prophylaxis)?

A
  • Acute attack: 100% oxygen, subcut triptan - Prophylaxis: verapamil
887
Q

what is the most common type of gallstone?

A

cholesterol stone (develop when the bile contains too much cholesterol and not enough bile salts)

888
Q

what is the most common cause of small bowel obstruction?

A

adhesions from prior surgeries

889
Q

if correctly managed, what is the prognosis like for TRALI?

A

improves over 2-4 days in over 80% cases with adequate ITU management and respiratory support

890
Q

how should someone presenting with an NSTEMI or unstable angina be managed (acutely)?

A

All patients: - Oxygen if evidence of hypoxia, pulmonary oedema or continuing myocardial ischaemia - Nitrates (GTN or IV isosorbide mononitrate) to relieve ischaemic pain (morphine can also be given if nitrates do not control pain) - Aspirin (300 milligrams orally) and ticagrelor (180mg) (clopidogrel and prasugrel are also commonly used) for dual antiplatelet effect. Patients should also receive either heparin (unfractionated), a low molecular weight heparin, or fondaparinux sodium - Beta-blockers (if not contraindicated) which should be continued indefinitely If patient is to undergo PCI or angiography: - The glycoprotein IIb/IIIa inhibitors eptifibatide (in combination with heparin (unfractionated) and aspirin) and tirofiban (in combination with heparin (unfractionated), aspirin, and clopidogrel) can be used for unstable angina or for NSTEMI in patients at a high risk of myocardial infarction and angiography is scheduled for within 96 hours

891
Q

with what signs and symptoms would you expect acute pancreatitis to present?

A

Symptoms: - Most commonly, presents as severe upper abdominal pain of sudden onset with vomiting. - Pain is focused in the left upper quadrant of the epigastrium and penetrates to the back. Occasionally, it encircles the abdomen. - Pain tends to decrease steadily over 72 hours Signs: - Tachycardia and dehydration common - Epigastric or generalised abdominal tenderness, often with rigidity - Jaundice may be present in patients with common bile duct stones or, to a lesser degree, in those with alcohol-induced disease - Bowel sounds are usually present in the early phase. Paralytic ileus, causing absent bowel sounds can last for >4 days and is a useful marker of disease severity - Hypoxaemia is characteristic of acute pancreatitis - In severe cases: gross hypotension, pyrexia, tachypnoea, acute ascites, pleural effusions, periumbilical (Cullen’s sign) or flank (Grey Turner’s sign) bruising.

892
Q

very briefly, what investigations should you consider in someone with suspected heart failure?

A
  • ECG - bloods (including BNP) - CXR - echocardiogram
893
Q

what does deposition of calcium pyrophosphate crystals in joints cause?

A

pseudogout

894
Q

what are some of the causes of a subdural haematoma?

A

Blunt head trauma is the usual mechanism of injury but spontaneous SDH can arise as a consequence of clotting disorder, arteriovenous malformations/aneurysms or other conditions

895
Q

which nerve is compressed in carpal tunnel syndrome?

A

median

896
Q

how is sickle cell anaemia generally managed?

A
  • Advice: minimise exposure to cold, dehydration, excessive exercise, undue stress etc - Prophylaxis: due to increased susceptibility to infection, children should be fully immunised and receive daily oral penicillin. - Blood transfusions: transfusion therapy is a key intervention in decreasing morbidity and mortality in patients with sickle cell disease - Bone marrow transplantation: may be offered to the most severely affected children
897
Q

what are some of the (many) causes of second degree heart block?

A

valvular/structural heart disease, MI/coronary heart disease, SLE, ankylosing spondylitis, Lyme disease, rheumatic fever, endocarditis, myocarditis, electrolyte disturbances, medications (digoxin, beta-blockers, calcium channel blockers etc).

898
Q

what are the 2 main management steps in chronic AF?

A
  • rate control e.g. beta-blocker, rate-limiting calcium channel blocker or digoxin - thromboprophylaxis e.g. NOAC or warfarin
899
Q

with what symptoms does coeliac disease usually present with?

A

• The clinical features of coeliac disease can be highly variable and include: mild, non-specific GI symptoms, anaemia (iron and/or folate) and growth failure

900
Q

what should be given to household contacts of someone with bacterial meningitis?

A

should be given prophylactic antibiotic (rifampicin)

901
Q

very very briefly, how is MND managed?

A
  • Riluzole (a neuroprotective glutamate-release inhibitor) is the only drug of proven disease-modifying efficacy (can help prolong lifespan by 2-4 months) - Otherwise treatment is largely supportive
902
Q

what are the 3 main characteristic features of Parkinson’s disease?

A
  • tremor - bradykinesia - rigidity (postural instability is sometimes included as a characteristic features as well)
903
Q

which signs/symptoms make up the ‘opioid overdose triad’?

A
  • Pinpoint pupils - Unconsciousness - Respiratory depression
904
Q

what is by far the most common causative organism for encephalitis?

A

HSV-1 responsible for 95% of cases in adults

905
Q

what are some of the possible clinical features of DIC?

A
  • Signs/symptoms of underlying condition - Bleeding, usually from at least 3 sites (e.g. nose, ear, GI tract, venepuncture/cannula site) - Confusion, disorientation, delirium, coma - Fever - Skin signs e.g. petechiae, acral cyanosis, skin necrosis - Signs of circulatory collapse e.g. tachycardia, hypotension etc
906
Q

what picture would you expect to see on an FBC of someone with iron-deficiency anaemia?

A

hypochromic microcytic anaemia

907
Q

briefly descrobe the AAA screening programme

A

The NHS AAA Screening Programme invites men, when they turn 65, for a scan of their aorta. If normal, no other scans will be offered

908
Q

what are the 2 features of AF seen on an ECG?

A

Absent P waves Irregularly irregular rhythm

909
Q

which inflammatory marker is usually raised in GCA?

A

ESR (CRP is often raised as well but ESR is more commonly raised)

910
Q

when should prophylactic treatment for migraines be considered?

A

to be considered in patients experiencing 2 or more migraines a month

911
Q

you hear a mid-late diastolic murmur which sounds low-pitched and rumbling on auscultation. What is the likely cause of this murmur and how could you position the patient to hear it better?

A

mitral stenosis listen with bell at 5th intercostal mid-clavicular with patient in left lateral position

912
Q

naloxone is used in the management of what kind of overdose?

A

opioid overdose

913
Q

what features would you expect to see in someone with Erb’s palsy?

A
  • flexed wrist with an extended forearm and internally rotated and adducted arm (waiter’s tip posture) - weakness in numerous upper limb muscle groups - sensory loss in the C5 and C6 dermatomes (thumb and lateral surface of the forearm and arm)
914
Q

what is the main symptom of mesenteric ischaemia?

A

abdominal pain that is typically severe, of sudden onset and out-of-keeping with physical exam findings

915
Q

what is the main issue associated with levodopa in the treatment of Parkinson’s disease?

A

Long-term levodopa treatment is associated with adverse motor effects that limit its use A major issue is the wearing-off effect, where patients symptoms worsen towards the end of the dosage interval. Increasing the dosage/frequency may overcome this, however patients may then start to get dyskinesia at the beginning of a dosage. When this occurs together, it is called the on-off effect.

916
Q

what specific examination can be used to help diagnose ankylosing spondylitis?

A

Schober’s test

917
Q

how should atrial flutter (acute, chronic and haemodynamically unstable) be managed?

A

If haemodynamically unstable: electrical cardioversion Acute atrial flutter: very similar management to AF (except medication is usually less effective in atrial flutter) > Rate control with beta-blocker or calcium channel blocker > Anticoagulation Chronic/recurrent atrial flutter: catheter ablation

918
Q

a patient is diangosed with acute heart failure and is stabilised. What medication should they be put on for the long-term?

A

Start ACE-I (ramipril)/ARB (valsartan) + β-blocker (bisoprolol) (may need other meds depending on other risk factors e.g. NOAC if in AF)

919
Q

what would you expect to hear on auscultation of someone with mitral stenosis?

A
  • Loud first heart sound with an opening snap in early diastole (due to stiffened valve) - Mid-late diastolic murmur, low-pitched and rumbling in nature, best heard with bell in left lateral position
920
Q

which movement is usually the first to be limited in hip OA?

A

internal rotation

921
Q

besides from Parkinson’s disease, what are some other causes of Parkinsonism?

A
  • drug-induced e.g. antipsychotics, metoclopramide - progressive supranuclear palsy - multiple system atrophy - Wilson’s disease - post-encephalitis - dementia pugilistica (secondary to chronic head trauma e.g. boxing) - toxins: carbon monoxide, MPTP
922
Q

what are the main investigations for DIC? (blood tests and what they would show)

A
  • FBC: thrombocytopenia - D-dimer: elevated - Fibrinogen: usually low - Prothrombin time (PT) and activated partial thromboplastin time (APTT): both prolonged
923
Q

what is the management of ventricular fibrillation?

A

Immediate defibrillation and advanced life support

924
Q

what is McBurney’s point and what is it’s clinical significance?

A

location: 1/3rd distance from anterior superior iliac spine to the umbilicus deep tenderness at this point is suggestive of appendicitis

925
Q

describe how IV fluids should be given in DKA management

A
  • if <90mmHg systolic, give 500ml bolus 0.9% NaCl - if >90mmHg systolic, give 1L NaCl over 1 hour (give then next two 1L infusions over 2 hours, and the fourth 1L infusion over 4 hours) - if potassium is 3.5-5.5, can add up to 40mmol potassium per 1L of NaCl, but can only be given starting from the SECOND bag of fluid (i.e. over 2 hours, so rate is max 20mmol/hr). If potassium <3.5, patient requires immediate senior review.
926
Q

briefly, what is gout?

A

defined as arthritis due to deposition of monosodium urate (MSU) crystals within joints causing acute inflammation and eventual tissue damage

927
Q

what are some common triggers for migraines?

A
  • tiredness, stress - alcohol - combined oral contraceptive pill - lack of food or dehydration - cheese, chocolate, red wines, citrus fruits - Menstruation (migraine occurs regularly 1-2 days before menstruation- known as a menstrual migraine) - bright lights
928
Q

which vasculitis is most commonly associated with being c-ANCA positive?

A

granulomatosis with polyangiitis (Wegener’s granaulomatosis)

929
Q

with what signs and symptoms would you expect someone with mitral stenosis to present with?

A

Symptoms: - Patient may be asymptomatic for a number of years - Breathlessness: on exertion, orthopnoea, PND - Atrial fibrillation: palpitations due to atrial fibrillation may be the presenting feature. Signs: - Malar flush - Raised JVP - AF - Signs of right ventricular failure including hepatomegaly, ascites and peripheral oedema - On auscultation: > Loud first heart sound with an opening snap in early diastole (due to stiffened valve) > Mid-late diastolic murmur, low-pitched and rumbling in nature, best heard with bell in left lateral position

930
Q

what is the HAS BLED score used for and what score is it commonly compared against?

A

it is used to help determine a patient’s bleeding risk on anticoagulation. it is often compared to the CHA2DS2-VASc score to determine the suitability of anticoagulating a patient with AF

931
Q

what are some causes of LBBB?

A
  • New LBBB should raise suspicion of MI (particularly if patient presents with chest pain!) - Hypertension - aortic stenosis - cardiomyopathy - rare: idiopathic fibrosis, digoxin toxicity, hyperkalaemia
932
Q

how should a haemodynamically unstable patient in VT be managed?

A
  • direct current cardioversion - IV amiodarone 2nd line (300mg over 1 hour)
933
Q

how is appendicitis managed in the vast majority of cases?

A

Open surgery or laparoscopic appendicectomy

934
Q

with what symptoms do peptic ulcers typically present?

A
  • GU: Epigastric pain, related to eating and may be relieved by antacids - DU: Epigastric pain, typically before meals or at night, relieved by eating or drinking milk - Nausea (both)
935
Q

what is the most common type of peptic ulcer?

A

duodenal (2-3 times more common than gastric)

936
Q

with what symptoms/signs might someone with vestibulocochlear nerve palsy present with?

A
  • Hearing loss - Tinnitus - Balance disturbance/vertigo
937
Q

what causes a fourth heart sound and what conditions is it associated with?

A

The fourth heart sound is produced by an increase in stiffness of the left ventricle due to scar tissue formation. This may be a manifestation of coronary heart disease. A fourth heart sound can also be caused by a greatly thickened left ventricular wall such as with essential hypertension or aortic stenosis

938
Q

what is the most important investigation in suspected spinal cord compression?

A

MRI spine

939
Q

briefly, what is the technique involved in PCI?

A

The technique involves gaining arterial access via the femoral, radial or brachial arteries. Under fluoroscopy, a guide wire is passed into the coronary artery and across the stenosis; the balloon or stent catheter is passed over it, and the lesion dilated and/or stented

940
Q

which 2 pancreatic enzymes can be used to help diagnose pancreatitis?

A

serum amylase and lipase

941
Q

how should sinus bradycardia be managed?

A

500mcg IV atropine

942
Q

what are some causes of white-out of a hemithorax, where the trachae is pulled towards the side of the white-out?

A
  • pneumonectomy
  • complete lung collapse
  • pulmonary hypoplasia
943
Q

what are some causes of a white-out of a hemithorax on a CXR where the trachea is central?

A
  • consolidation
  • pulmonary oedema
  • mesothelioma
944
Q

what are some causes of white-out of a hemithorax on a cxr where the trachea is pushed away from the white-out?

A
  • massive pleural effusion
  • diaphragmatic hernia
  • large thoracic mass
945
Q

in what instances might ferritin levels be unreliable?

A

ferritin levels can be elevated when inflammation or co-existing conditions such as liver disease, malignancy or hyperthyroidism are present thus giving spurious readings

946
Q

when is the combined oral contraceptive pill and HRT usually stopped prior to surgery?

A

4 weeks prior to surgery

947
Q

what should you consider prescribing alongside an opioid?

A
  • REGULAR laxative (usually movicol)
  • PRN antiemetic (usually cyclizine)
  • consider PRN naloxone
948
Q

what is vitamin B1 more commonly called?

A

thiamine

949
Q

briefly, what is Ramsay-Hunt syndrome?

A

shingles affecting the facial nerve

950
Q

polycystic kidney disease is associated with which valvular disease?

A

mitral valve prolapse

951
Q

with what features would you expect someone with type 1 diabetes to present?

A
  • Patients with all types of diabetes may present with polyuria, polydipsia, lethargy, boils, pruritus vulvae or with frequent, recurrent or prolonged infections.
  • Patients with type 1 diabetes may also present with weight loss, dehydration, ketonuria and hyperventilation. Presentation of type 1 diabetes tends to be acute with a short duration of symptoms
952
Q

how is diabetes usually diagnosed?

A

If symptomatic: -

  • Random plasma glucose >11 mmol/L
  • May also confirm by measuring fasting glucose (no food/drink except water for 8 hours) as >7 mmol/L

(note: if asymptomatic, above results must be seen on 2 separate occasions)

HbA1c greater than or equal to 48 mmol/mol (6.5%)

953
Q

what is the main clinical importance/use of C-peptide?

A

can be used to help differentiate type 1 from type 2 diabetes

C-peptide is made alongside insulin so is low in type 1 diabetics

954
Q

besides from insulin, which medication used in diabetes management is most commonly associated with hypoglycaemia?

A

Sulfonylureas (e.g. gliclazide)

955
Q

what is the most signficant draw-back of sulfonylureas (e.g. gliclazide)?

A

can cause hypoglycaemia

956
Q

What is Charcot’s arthropathy?

A
  • A Charcot joint is also commonly referred to as a neuropathic joint. It describes a joint which has become badly disrupted and damaged secondary to a loss of sensation (most commonly in diabetes)
  • The Charcot foot is characterised by bone and joint degeneration which can lead to a devastating deformity. It usually presents as a hot swollen foot after minor trauma
957
Q

which group is most likely to develop Charcot’s foot? and how does it typically present?

A

diabetics (as is due to damage secondary to peripheral neuropathy)

It usually presents as a hot swollen foot after minor trauma

958
Q

which thyroid hormone is inactive?

A

T4 (thyroxine). It is converted to T3 outside of the thyroid gland (particularly in the liver)

959
Q

what is the most common cause of hyperthyroidism?

A

Grave’s disease

960
Q

briefly describe the aetiology behind Grave’s disease

A

Graves’ disease is an autoimmune condition. The aetiology of thyroid hormone overproduction is stimulation of the thyroid by TSH receptor antibodies

961
Q

bilateral exopthalmos is associated with what condition?

A

Grave’s disease

962
Q

what are some of the signs and symptoms (+ specific features) you might expect someone with Grave’s disease to present with?

A

Hyperthyroidism features:

  • Symptoms: weight loss, diarrhoea, heat intolerance, sweating, palpitations, tremor, irritability, oligomenorrhoea (infrequent periods)
  • Signs: tachycardia, warm moist skin, fine tremor, palmar erythema, brisk reflexes, diffuse goitre, hair loss

Features specific to Grave’s disease:

  • Bilateral exophthalmos
  • Opthalmoplegia (paralysis of eye muscles)
  • Pretibial myxoedema
  • thyroid acropachy, a triad of: digital clubbing, soft tissue swelling of the hands and feet and periosteal new bone formation
963
Q

pretibial myxoedema is a dermatological manifestation of which condition?

A

Grave’s disease

964
Q

what are some features specific to Grave’s disease (i.e. help to differentiate from other causes of hyperthyroidism)?

A
    • Bilateral exophthalmos
    • Opthalmoplegia (paralysis of eye muscles)
    • Pretibial myxoedema
    • thyroid acropachy, a triad of: digital clubbing, soft tissue swelling of the hands and feet and periosteal new bone formation
965
Q

which antibodies are most associated with Grave’s disease?

A

TSH-receptor antibodies

966
Q

how should Grave’s disease be managed?

A
  • All patients with new-onset thyrotoxicosis should be referred for assessment in secondary care, to establish the cause and to agree on a management plan
  • Antithyroid drugs (e.g. carbimazole)
  • Symptomatic treatment e.g. beta-blocker
  • Radioactive iodine or surgery may also be considered in some cases
967
Q

when starting a patient on carbimazole, what important safe guarding info should you give them?

A

carbimazole can cause agranulocytosis so warn patients to come for FBC if they develop a sore throat, etc!!!

968
Q

what is the most significant side effect/complication of carbimazole?

A

can cause agranulocytosis (so warn patients to come for FBC if they develop a sore throat, etc!!!)

969
Q

what is the most common cause of hypothyroidism, both in developed countries and worldwide?

A

in developed countries: Hashimoto’s thyroiditis

worldwide: Iodine deficiency

970
Q

what are some of the signs and symptoms of hypothyroidism?

A
  • Often has insidious onset
  • Symptoms: lethargy, cold intolerance, weight gain, constipation, menorrhagia, hoarse voice, dry skin, hair loss (including outer portion of eyebrows), poor memory and difficulty concentrating, reduced libido
  • Signs: bradycardia, delayed reflex relaxation, cold peripheries, round puffy face + hands + feet (myxoedema), carpal tunnel syndrome
971
Q

what would the TFTs show in:

  1. hyperthyroidism
  2. subclinical hyperthyroidism
A
  1. In hyperthyroidism, TSH will be decreased, while T4/T3 will be elevated
  2. In subclinical hyperthyroidism, TSH will be decreased while T4/T3 will be normal
972
Q

what would the TFTs show in:

  1. primary hypothryoidism
  2. subclinical hypothyroidism
A
  1. Primary hypothyroidism: TSH elevated, T4/T3 reduced
  2. Subclinical hypothyroidism: TSH elevated, T4/3 normal
973
Q

briefly, what is the effect of parathyroid hormone on the body’s electrolytes?

A

High levels of PTH cause serum calcium levels to increase and serum phosphate levels to fall.

974
Q

what is the difference between primary and secondary hyperparathyroidism?

A
  • Primary: one parathyroid gland (or more) produces excess PTH.
  • Secondary: there is increased secretion of PTH in response to low calcium because of kidney, liver, or bowel disease
975
Q

what is the most common cause of primary hyperparathyroidism?

A

A single parathyroid gland adenoma (85% of cases), which causes excess PTH is produced

976
Q

what are the 2 most common causes of hypercalcaemia?

A

90% of cases are due to either primary hyperparathryoidism or malignancy

977
Q

pepperpot skull (also called salt and pepper sign) is a characteristic X-ray finding of which condition?

A

primary hyperparathyroidism

978
Q

what electrolyte disturbances would you expect to see in primary hyperparathryoidism? (2)

A

raised calcium, low phosphate

979
Q

how should you confirm death?

A
  • Step 1: Establish pre-conditions:
  • Was a DNAR in place
  • Was a DNAR not in place but CPR was felt to be of no clinical benefit
  • Was CPR attempted but unsuccessful
  • Step 2: confirm cardio-respiratory arrest for five continuous minutes (the breakdown of these minutes can vary e.g. 2 mins on cardiac activity, 3 mins on respiratory activity):
  • No evidence of cardiac activity by palpation of central pulse and cardiac auscultation
  • No evidence of respiratory activity on clinical observation and auscultation
  • Step 3: confirm absence of neurological function:
  • Pupillary responses to light are absent
  • Corneal reflexes are absent
  • No motor responses to supra-orbital pressure
980
Q

when is time of death noted?

A

after death has been confirmed by a suitably trained healthcare professional

981
Q

how should a death certificate be filled out?

A
  • In section Ia, mention the immediate terminal event that led to death, and in Ib, and Ic list the underlying conditions, which might in certain cases be chronic, leading to death. Thus, the diseases mentioned in Ib should have lead to/directly caused Ia, while diseases mentioned in Ic should have lead to/directly caused Ib
  • In section II state all the diseases, injuries, and conditions that contributed to death but were not part of Ia, Ib, and Ic, for example diabetes. You shouldn’t, however, put the patient’s entire PMH
982
Q

briefly, what is Addison’s disease?

A

Addison’s disease is the most common cause of primary adrenal insufficiency, where autoimmue destruction of the adrenal glands results in not enough steroid hormones (cortisol and aldosterone in particular) being produced.

983
Q

what are some of the causes of primary and secondary adrenocrotical insufficiency (particularly the most common cause of both)?

A

Primary insufficiency:

  • Addison’s disease (85%): autoimmune destruction of the adrenal glands
  • Infections, particularly TB
  • Infarction e.g. in antiphospholipid syndrome

Secondary insufficiency:

  • Exogenous steroid use (most common)
  • Adrenal adenomas

(note that there are many more causes of both primary and secondary)

984
Q

what are some of the signs and symptoms of Addison’s disease (primary adrenocortical insufficiency)?

A
  • Symptoms: lethargy, weakness, anorexia, nausea & vomiting, weight loss, ‘salt-craving’
  • Signs: hyperpigmentation (especially palmar creases)*, vitiligo, loss of pubic hair in women, hypotension, hypoglycaemia, hyponatraemia and hyperkalaemia may be seen
985
Q

what are some of the features of an Addiosonian crisis?

A
  • May be precipitated by infection, surgery or trauma on a background of chronic adrenocortical insufficiency (e.g. Addison’s disease, which may be diagnosed or undiagnosed)
  • features include hypotension, hypovolaemic shock (which may present as collapse), acute abdominal pain, low-grade fever and vomiting
  • can be life-threatening
986
Q

what are some of the main investigations for suspected adrenocortical insufficiency?

A
  • ACTH (also known as corticotropin) - when measured together with cortisol (which is usually reduced) allows differentiation of primary vs secondary insufficiency:
  • Levels are raised in primary insufficiency.
  • Levels are low or low normal in secondary insufficiency.
  • U&Es/ABG due to electrolyte disturbances:
  • hyperkalaemia
  • hyponatraemia
  • hypoglycaemia
  • metabolic acidosis
987
Q

briefly, how should Addison’s disease be managed?

A

Patients who have Addison’s disease are usually given both glucocorticoid and mineralocorticoid replacement therapy. This usually means that patients take a combination of hydrocortisone and fludrocortisone

988
Q

a patient with Addison’s disease is due to undergo a minor surgery. What MUST be done?

A

increase their glucocorticosteroid (e.g. hydrocortisone) dose

(during minor illness or minor surgery, glucocorticoid doses may be increased up to three times their normal dose to avoid adrenal crisis, and up to ten times for major illness or major surgery)

989
Q

how do you calculate pack years?

A

average number of packs per day (where 1 pack=20 cigarettes) x the number of years they have smoked them for

e.g. 2 packs a day for 10 years= 2x10=20

990
Q

which cancers are particularly associated with hypercalcaemia?

A

Hypercalcaemia can be associated with most tumours, but is commonly seen in patients with breast cancer, squamous cell carcinomas (lung cancer) and myeloma

991
Q

what are some of the features of hypercalcaemia?

A

‘bones, stones, abdo moans and psychiatric groans’

  • Bones: bone pains, arthritis, osteoporosis
  • Stones: renal stones, polyuria, nocturia, dehydration/thirst
  • Abdo moans: abdominal pains, nausea and vomiting, constipation
  • Psychiatric groans: confusion, lethargy, drowsiness, depression, coma
992
Q

how should hypercalcaemia be managed?

A
  • Rehydration with IV saline, as forceful diuresis promotes calcium excretion. Exact rate depends on severity of hypercalcaemia
  • IV bisphosphonates (after rehydration): Pamidronate or Zolendronic Acid (inhibit osteoclastic bone resorption)
  • Other medication to consider: calcitonin and corticosteroids
993
Q

what are some possible signs of hyperlipidaemia?

A
  • Corneal arcus (white/grey ring around iris)
  • Xanthelasma (yellow flat plaques, usually around the eyes, due to cholesterol deposits under the skin)
  • Tendon xanthoma (slowly enlarging subcutaneous nodules related to a tendon or ligament, usually on hands and feet, due to deposits of cholesterol)
994
Q

which statin is usually used first line in patients with hypercholestrolaemia?

A

atrovastatin

995
Q

what are some causes of upper GI bleeds?

A
  • 2 most common causes: peptic ulcers and oesophageal/gastric varices
  • Other causes include: oesophagitis, gastritis, malignancy, Mallory Weiss tear
996
Q

what 2 main scoring tools are used in acute upper GI bleeds?

A
  • Glasgow-Blatchford bleeding score (GBS): a screening tool to assess the likelihood that a person with an acute upper gastrointestinal bleeding (UGIB) will need to have medical intervention such as a blood transfusion or endoscopic intervention
  • Complete Rockall Score: estimates mortality in patients with active upper GI bleed who have had endoscopy
997
Q

you suspect the cause of an acute upper GI bleed to be varices. What should the patient recieve prior to endoscopy (besides from initial resuscitation)?

A

patients should receive terlipressin and prophylactic antibiotics prior to endoscopy

998
Q

which medications are most commonly associated with constipation?

A

opioid analgesics (e.g. morphine, codeine), anticholinergics (tricyclics, phenothiazines), iron

999
Q

what is Barrett’s oesophagus?

A

metaplasia of the lower oesophageal mucosa, with the usual squamous epithelium being replaced by columnar epithelium.

1000
Q

Barrett’s oesophagus is a precursor for which specific type of oesophageal cancer?

A

adenocarcinoma

1001
Q

what are the main blood findings which are seen in hypoparathyroidism?

A

low serum calcium, high serum phosphate, low PTH

1002
Q

briefly, what is the relation between vitamin d and calcium?

A

Vitamin D promotes the efficient absorption of calcium from the intestine (so lack of vitamin D can lead to low calcium levels)

1003
Q

what is the most common type of thyroid cancer?

A

Papillary thyroid carcinoma

1004
Q

which medications are most associated with thyroid enlargement?

A

amiodarone and lithium

1005
Q

what features of a goitre would make you consider an urgent referral to secondary care?

A

Urgently refer (for an appointment within 2 weeks) to a thyroid surgeon or endocrinologist people with:

  • An unexplained thyroid lump (e.g. TFTs normal, no clinical evidence of hyper/hypothyroidism).
  • Unexplained voice changes (for example hoarseness) associated with a goitre.
  • Cervical lymphadenopathy (usually deep cervical or supraclavicular region) associated with a thyroid mass.
  • A painless thyroid mass, rapidly increasing in size over a period of weeks (rare; usually associated with anaplastic thyroid cancer or thyroid lymphoma
1006
Q

what are some of the main clinical features (signs + symptoms) of a phaeochromocytoma?

A
  • hypertension (around 90% of cases, may be sustained)
  • headaches
  • palpitations
  • sweating
  • anxiety
1007
Q

what is the difference between Cushing’s syndrome and Cushing’s disease?

A
  • Cushing syndrome is the clinical manifestation of pathological hypercortisolism from any cause
  • Cushing’s disease, which is hypercortisolism caused by an adrenocorticotropic hormone (ACTH)-secreting pituitary adenoma, is the most common cause of Cushing’s syndrome, and is responsible for 70% to 80% of cases
1008
Q

what causes Cushing’s disease?

A

hypercortisolism caused by an adrenocorticotropic hormone (ACTH)-secreting pituitary adenoma

1009
Q

what are some of the main features (signs + symptoms) of Cushing’s syndrome?

A

Patients often display weight gain with central obesity, facial rounding and plethora, proximal muscle weakness, and thinning of the skin. They also develop metabolic complications including diabetes mellitus, dyslipidaemia, metabolic bone disease, and hypertension

1010
Q

what is Conn’s syndrome?

A

hyperaldosteronism due to an adrenal adenoma

1011
Q

what is hirsutism and what is the most common cause of it?

A

Hirsutism is a condition seen in women that is associated with development of androgen-dependent terminal hair (coarse, pigmented) following a male-pattern distribution (face, chest, abdomen, back)

the most common cause is polycystic ovary syndrome

1012
Q

what is diabetes insipidus, what are the main features, and what is the main investigation?

A

Diabetes insipidus (DI) is a metabolic disorder characterised by an absolute or relative inability to concentrate urine due to either a deficiency of antidiuretic hormone, ADH, (central DI) or an insensitivity to antidiuretic hormone (nephrogenic DI).

main features are polydipsia, polyuria and formation of inappropriately hypotonic (dilute) urine

main investigation is urine and serum osmolarity (in DI you would have a high plasma osmolality, low urine osmolality)

1013
Q

what electrolyte disturbance is associated with SIADH?

A

hyponatraemia (as water is not being excreted so you get dilutional hyponatraemia)

1014
Q

what is the most common type of hiatur hernia?

A

sliding hiatus hernias account for 95% of hiatus hernias, the gastroesophageal junction moves above the diaphragm

1015
Q

how does atelectatis usually present?

A

it should be suspected in the presentation of dyspnoea and hypoxaemia around 72 hours postoperatively

1016
Q

what is the more commonly used name for infectous mononucleosis and what is the most common causative organism for it?

A

glandular fever

majority of cases are cause by Epstein-Barr virus (EBV)

1017
Q

how does infectious mononucleosis (glandular fever) usually present?

A
  • Classical triad: fever, pharyngitis (sore throat) and lymphadenopathy
  • Other common features: malaise, splenomegaly (50%) and hepatomegaly (10%)
  • Other possible features: maculopapular rash, jaundice, nausea, myalgia
1018
Q

what condition is the Monospot test used to help diagnose?

A

infectious mononucleosis (glandular fever).

it is a blood test which detects heterophile antibodies

1019
Q

what specific advice should be given to someone with suspected or confirmed infectious mononucleosis (glandular fever)?

A

avoid strenuous physical activity (particularly contact sports) due to risk of splenic rupture

1020
Q

what medication should be avoided when managing someone with infectious mononucleosis (glandular fever)?

A

aspirin in children: risk of Reye’s syndrome

ampicillin or amoxicillin: may cause florid maculopapular rash in those infected with EBV

1021
Q

what are some risk factors for c.diff infection?

A
  • Broad spectrum antibiotics (particularly if prolonged use or multiple use): clindamycin, cephalosporins (e.g. ceftriaxone, cefotaxime), carbapenems (e.g. meropenem), co-amoxiclav, ampicillin, amoxicillin
  • Increasing age
  • Immunocompromised
  • Presence of NG tube
1022
Q

how does c.diff infection usually present?

A
  • Symptoms usually appear 5-10 days follow antibiotic use
  • Most affected individuals experience watery diarrhoea (varies from self-limiting to severe and debilitating) ± blood-stained stools, abdominal cramps, fever (especially so in severe cases), rigors ± sepsis
1023
Q

how should c.diff infections be managed?

A
  • Confirmed cases must be reported to Public Health England
  • General measures:
  • Correct fluid loses and electrolyte disturbances
  • Cease possible causative antibiotics
  • Medication:
  • For first episode of mild to moderate infection, give oral metronidazole for 10-14 days
  • For second or subsequent episode of infection, severe infection, infection not responding to metronidazole, or in patients intolerant of metronidazole, give oral vancomycin for 10-14 days
1024
Q

what are some of the features of measles?

A
  • Prodrome:
  • This lasts 2-4 days with fever, cough, runny nose, mild conjunctivitis and diarrhoea.
  • Koplik’s spots are pathognomonic and appear on the buccal mucosa as small, red spots, each with a bluish-white speck (sometimes compared with a grain of rice) in the centre
  • Features strongly suggestive of measles:
  • Rash: This is first seen on the forehead and neck and spreads, involving the trunk and finally the limbs, over 3-4 days. It may become confluent in some areas. The rash then fades after 3-4 days in the order of its appearance.
  • High fever
  • With a rash and a fever, often at least one of the following are present: cough, coryza, conjunctivitis
  • Other features: eye swelling, photophobia
1025
Q

which organism causes whooping cough?

A

bordetella pertussis

1026
Q

how does malaria present?

A
  • Symptoms: fever, chills, rigors, headache, cough, myalgia, GI upset
  • Signs: fever, splenomegaly, hepatomegaly, jaundice
  • Features of severe disease: SOB, fits, impaired consciousness, AKI
1027
Q

how might pancreatic cancer present?

A
  • Note that early symptoms are often vague and non-specific
  • More than two thirds occur in the head of the pancreas and classically present with painless, progressive, obstructive jaundice
  • Abdominal pain: typically located in the epigastric region, radiating through to the back. Back pain is typically dull and worse when supine and eased by sitting forward.
  • Unexplained weight loss, anorexia.
  • Steatorrhoea due to malabsorption
  • Palpable gallbladder: Courvoisier’s sign (a palpable gallbladder in the presence of painless jaundice) occurs in fewer than 25% of patients
  • Haematemesis, melaena or iron-deficiency anaemia
1028
Q
A
1029
Q

when should you consider prescribing antibiotics in suspected acute otits media?

A
  • Symptoms lasting more than 4 days or not improving
  • Systemically unwell but not requiring admission
  • Immunocompromise or high risk of complications secondary to significant heart, lung, kidney, liver, or neuromuscular disease
  • Younger than 2 years with bilateral otitis media
  • Otitis media with perforation and/or discharge in the canal
1030
Q

swimming is particularly associated with what kind of ear infection?

A

otitis externa

1031
Q

which type of eye sight is more associated with acute angle closure glaucoma, long-sightedness (hyperopia) or short-sightedness (myopia)?

A

long-sightedness (hyperopia)

1032
Q

what are the signs and symptoms of acute angle closure glaucoma?

A

Symptoms:

  • Severe and rapidly progressive pain: may be ocular or headache
  • Blurred vision/ reduced visual acuity
  • Coloured haloes around lights
  • Systemic malaise - nausea and vomiting are common and may be the main presenting feature in some patients

On examination: Examination shows a red eye in the form of a ciliary flush: the redness is more marked around the periphery of the cornea. There is a hazy cornea and a non-reactive (or minimally reactive) mid-dilated pupil

1033
Q
A
1034
Q

how does testicular torsion usually present (including examination findings)?

A
  • Sudden severe pain in one testis, often coming on during physical activity/sports
  • Nausea and vomiting are common
  • Often a history of brief periods of similar pain (during which the torsion corrected itself)
  • Examination:
  • Red scrotal skin
  • Swollen, tender testis retracted upwards
  • Absent cremasteric reflex: gentle pinching/stroking of inner thigh should cause the cremaster muscle to contract and pull up the ipsilateral testicle toward the inguinal canal
1035
Q

how should tetsticular torsion be managed?

A
  • Immediate referral to urology/ surgery (should withhold food and provide analgesia while waiting for referral)
  • Surgery: bilateral orchiopexy- fixation of testes in the scrotum (risk of recurrence in other testicle is high so orchiopexy should be bilateral). Should be done as soon as possible and at least within 24 hours to save the teste.
  • A baby born with testicular torsion should have the affected testis removed (because it is always non-viable) and orchidopexy of the other side (because bilateral torsion is common)
1036
Q

what are some of the signs/symptoms and examination findings of someone with an ectopic pregnancy?

A

Signs/symptoms:

  • Lower abdominal pain (may be colicky in nature to start, then becomes constant)
  • Abnormal vaginal bleeding (often dark in colour with or without clots)
  • Amenorrhoea of 4-10 weeks
  • Dizziness, fainting, syncope
  • Shoulder tip tenderness (suggests intraperitoneal bleeding)
  • May have urinary or bowel symptoms
  • Ruptured ectopic may present with symptoms of shock

Examination:

  • Pelvic or abdominal tenderness (possible rebound tenderness)
  • Adnexal tenderness and cervical motion tenderness
  • Closed cervical os
  • Tachycardia and hypotension suggest shock
1037
Q

what is the most common cause of genital warts?

A

Over 95% of genital warts are caused by infection with HPV types 6 and 11

1038
Q

what are the features of prostatitis (including examination findings)?

A
  • Fever, malaise, arthralgia and myalgia.
  • Urinary frequency, urgency, dysuria, nocturia, hesitancy and incomplete voiding.
  • Low back pain, low abdominal pain, perineal pain and pain in the urethra. In chronic prostatitis the most consistent finding is that of chronic pelvic pain
  • -* On examination:
  • Tender boggy prostate gland
1039
Q

brielfly, what is the difference between phimosis and paraphimosis?

A

phimosis is where you cannot retract the foreskin over the glans penis.

paraphimosis is where the foreskin is retracted and cannot be brought back over the glans penis

1040
Q
A
1041
Q

what are breast fibroadenomas and how are they managed?

A

These are benign tumours that are common in young women, with incidence peaking at 20-24 years of age. They present as firm, non-tender, highly mobile palpable lumps

They are often treated with surgical excision but this may not be necessary if they are small and the diagnosis is confirmed

1042
Q

what features can help you differentiate fibrocystic disease from breast cancer?

A

To help differentiate from breast cancer, fibrocystic disease is most often bilaterally symmetrical (whereas breast cancer usually affects one breast). Patients may also experience pain/tenderness that fluctuates with the menstrual cycle in fibrocystic disease

1043
Q

what is the difference between indirect and direct inguinal hernias and which are more common?

A
  • indirect hernia (these are more common): a protrusion through the internal inguinal ring passes along the inguinal canal through the abdominal wall, running laterally to the inferior epigastric vessels
  • Direct: the hernia protrudes directly through a weakness in the posterior wall of the inguinal canal, running medially to the inferior epigastric vessels
1044
Q

how are inguinal hernias managed?

A
  • most patients, as long as they are medically fit, undergo surgery to repair due to risk of strangulation
1045
Q

out of inguinal and femoral hernias, which are more likely to strangulate?

A

femoral hernias

(note that both should be surgically managed ideally, but this is more important for femoral hernias and needs to be done sooner)

1046
Q
A
1047
Q

what is the most common cause of portal hypertension?

A

cirrhosis

1048
Q

what is the main cause of oesophageal or gastric varices?

A

portal hypertension

1049
Q

what is the most significant complication of portal hypertension?

A

oesophageal/gastric varices

1050
Q

what proportion of patients with hepatitis C will go on to develop chronic hep C?

A

About 75% of patients infected with hepatitis C will develop chronic disease (roughly 20-30% of these will go on to develop cirrhosis after 20 years)

1051
Q

which types of hepatitis are most likely to cause cirrhosis?

A

hep B and C

1052
Q
A
1053
Q

where does colorectal cancer most commonly metastesise to?

A

The most common site for metastatic spread is the liver. Other sites (eg, the lungs, brain and bone) are unusual in the absence of liver metastases.

1054
Q

what stool test should be conducted when you suspect a patient may have inflammatory bowel disease or IBS?

A

Faecal calprotectin:

Calprotectin is a small calcium-binding protein. The concentration of calprotectin in faeces has been shown to correlate well with the severity of intestinal inflammation[6].

Faecal calprotectin testing is recommended as an option when considering the differential diagnosis of IBD or irritable bowel syndrome

1055
Q

how is a sigmoid volvulus usually diagnosed?

A
  • Diagnosis is usually made from abdo x-ray: shows a single grossly dilated sigmoid loop commonly reaching the xiphisternum- called coffee bean sign
1056
Q

osteomalacia in children is known as?

A

Rickets

1057
Q
A
1058
Q

Von Willebrand Disease and Factor V Leiden, which increases the risk of bleeding and which increases the risk of clotting?

A
  • Von Willebrand’s disease is the most common inherited bleeding disorder.
  • Factor V Leiden is the most common cause of inherited thrombophilia.